You are on page 1of 318

SAMPLE

PAPERS

[SESSION: 2022-23]
CLASS IX

KENDRIYA VIDYALAYA SANGATHAN::: RANCHI REGION


UNDER THE GUIDANCE OF

HON’BLE SHRI D P PATEL


DEPUTY COMMISSIONER
KVS RO, RANCHI

&

RESPECTED SHRI SURESH SINGH


ASSISTANT COMMISSIONER
KVS RO, RANCHI
LIST OF TEACHERS WHO PREPARED SAMPLE PAPERS
S.N. Name of KV Name of TGT Sc. Group Name Group Leader
1 Barkakana Smt. Zeba Waris Bhabha
2 Bokaro Thermal Sh. Indra Dev Hansda Bhabha
Ms. Zeba Waris
3 Sahibganj Ms. Shweta Suman Bhabha
4 Latehar Sh. Arun Kumar Bhabha
5 Garhwa Mr. Ashutosh Lal Birbal Sahni
6 Ranchi, Hinoo ( 1S) Mrs.Kumari Sunita Birbal Sahni Mr. Ashutosh Lal
7 Hazaribagh Kanchan Kunari Birbal Sahni
8 Bhurkunda Pramila Kumari Chandrashekhar
9 Dumka Ms Madhulika Kumari Chandrashekhar Ms. Pramila Kumari
10 Lohardaga Sweta Verma Chandrashekhar
11 Madhupur Ms Mini Kumari Kalam
12 Chakradharpur Tulika giri Kalam Dr. Vineeta Parmar
13 Patratu Dr. VINEETA PARMAR Kalam
14 Bokaro-1 1. Sh. Pawan Kumar Oraon
2. Sh. Shankar Prasad
3.Smt. Salina Kumari Meghnad Ms. Jyoti Prasad
15 Maithon Dam 1. MS JYOTI PRASAD
2. MS. SHARDA BHARTI Meghnad
16 Khunti MISS PURNIMA KUMARI PC Roy
17 Meghahatuburu 1.PREM CHANDRA PAN
2. SUSHMA DAS PC Roy
Ms. Purnima Kumari
18 Singharsi Smt. Soni Kumari PC Roy
19 Ranchi, Dipatoli Smt. N.M.Tirkey
Smt. Ragini Singh PC Roy
20 Dhanbad-1 Smt.Kiran Kumari Raman
21 Gomoh Ms. Pallavi Kumari Raman Ms. Kiran Kumari
22 Ranchi, CRPF Smt. Sangeeta Sinni Raman
23 Dhanbad-2 SHILPI BILLAIYA Ramanujan
Ms. Shilpi Billaiya
24 Jamtara MRS. RAKHI SINGH Ramanujan
25 Chandrapura Kumud Prashar Sayendra
26 Gumla Ms Jyoti Kumari Sayendra Ms. Sushma Dubey
27 Ramgarh Cantt 1. Mrs. Sumitra Kumari
2. Mrs. Sushma Dubey Sayendra
28 Giridih Ms. Rajni kumari Verma Vikram Sarabhai
29 Tatanagar Dr. Deepa Kaushal Vikram Sarabhai Mrs. Jiren Tuti
30 Surda Mrs. Jiren Tuti Vikram Sarabhai
31 Coordinator Manish Kumar Prabhat, Principal, KV Madhupur
KENDRIYA VIDYALAYA SANGATHAN
RANCHI RAGION (2022-23)
CLASS 9 SCIENCE
[MLL]
BLUE PRINT
MAX. MARKS: 80 TIME : 3 HRS
S. CHAPTERS MCQ A-R SA SA LA CBQ SUB
No. (1M) (1M) (2M) (3M) (5M) (4M) TOTAL
1 MATTER IN OUR 1(1) 1(2)OR 1(3) 3(6)
SURROUNDING
2 IS MATTER AROUND 1(1) 1(2) 1(3) 3(6)
US PURE
3 ATOMS AND 1(1) 1(1) 2(4) 4(6)
MOLECULES
4 STRUCTURE OF 3(3) 1(4) 4(7)
ATOM
5 CELL:BASIC UNIT OF 2(2) 1(2) 1(3) 1(4) 5(11)
LIFE
6 TISSUES 2(2) 1(1) 1(3) 1(5) 5(11)
7 MOTION 1(1) 1(3) 2(4)
8 FORCE AND LAWS OF 1(1) 2(4) 3(5)
MOTION
9 GRAVITATION 1(1) 1(5) 2(5)
10 FLOATATION 1(1) 1(3) 2(4)
11 WORK ENERGY AND 1(1) 1(1) 1(3) 3(5)
POWER
12 SOUND 1(1)or 1(4) 2(4)
13 IMPROVEMENT IN 1(1) 1(5) 2(6)
FOOD RESOURCES
39(80)
KENDRIYA VIDYALAYA SANGATHAN
RANCHI REGION (2022-23)
MLL
CLASS : 9 SUBJECT :
SCIENCE
MAX. MARKS : 80 TIME : 3 HRS
GENERAL INSTRUCTIONS:
i. This question paper consists of 39 questions in 5 sections.
ii. All questions are compulsory. However, an internal choice is provided in some
questions. A student is expected to attempt only one of these questions.
iii. Section A consists of 20 objective type questions carrying 1 mark each.
iv. Section B consists of 6 Very Short questions carrying 02 marks each. Answers to
these questions should in the range of 30 to 50 words.
v. Section C consists of 7 Short Answer type questions carrying 03 marks each. Answers
to these questions should in the range of 50 to 80 words.
vi. Section D consists of 3 Long Answer type questions carrying 05 marks each. Answer
to these questions should be in the range of 80 to 120 words.
vii. Section E consists of 3 source-based/case-based units of assessment of 04 marks
each with sub-parts.

S. SECTION A Marks
NO.
1. The two major gases present in the air are 1
a) Nitrogen and Carbon dioxide.
b) Nitrogen and Oxygen
c) Hydrogen and Oxygen
d) Nitrogen and Hydrogen

2. Chromosomes are made up of 1


a) RNA
b) DNA
c) DNA and protein
d) Protein

3. Which of the following gives both direction and magnitude? 1


a) Unit Scalar
b) Scalar
c) Unit Vector
d) Vector

4. Who is known as the 'Father of white revolution' in India? 1


a) Mrs. Indira Gandhi
b) Shri Jai Prakash Narain
c) Dr.V. Kurian
d) Prof M.S. Swaminathan

5. Select the incorrect sentence 1


a) Cartilage is a form of connective tissue
b) Tendons are non-fibrous tissue and fragile
c) Two bones are connected with ligament
d) Blood has matrix containing proteins, salts, and hormones

6. Which of the following cell functions will stop, if its ribosomes are 1
destroyed?
a) Formation of complex sugars
b) Lipid metabolism
c) Protein synthesis
d) ATP synthesis
7. Match the following with the correct response: 1
(1) Nitrate (A) P3−
(2) Phosphide (B) PO43−
(3) Nitrite (C) NO2−
(4) Phosphate (D) NO3−
a) 1-B, 2-D, 3-A, 4-C
b) 1-C, 2-B, 3-D, 4-A
c) 1-D, 2-A, 3-C, 4-B
d) 1-A, 2-C, 3-B, 4-D

8. Which of the following sets includes simple permanent tissues? 1


a) Collenchyma, parenchyma, Sclerenchyma
b) Pholem, xylem, collenchyma
c) Sclerenchyma, phloem, collenchyma
d) Parenchyma, phloem, sclerenchyma
9. If the weight of Harsh is 150N. What is his mass? (g = 10 m/s2) 1
a) 12 kg
b) 90 kg
c) 15 kg
d) 100 kg
10. The slope of the x-t graph is a measure of 1

a) velocity = 2ms-1
b) acceleration = 2ms-2
c) acceleration =12ms-2
d) velocity =12ms-2

11. Argentum is the Latin name of: 1


a) Tungsten
b) Silver
c) Sodium
d) Antimony

12. Contractile proteins are present in ________ tissue. 1


a) areolar
b) adipose
c) nervous
d) muscular

13. Match the following with the correct response: 1


(a) Robert Brown (i) Cell
(b) Purkinje (ii) Nucleus
(c) Robert Hooke (iii) Cell theory
(d) Schleiden (iv) Protoplasm
a) (a) - (iii), (b) - (ii), (c) - (iv), (d) - (i)
b) (a) - (ii), (b) - (iv), (c) - (i), (d) - (iii)
c) (a) - (iv), (b) - (i), (c) - (iii), (d) - (ii)
d) (a) - (i), (b) - (iii), (c) - (ii), (d) - (iv)

14. Match the following with the correct response: 1


(1) Ozone (A) Monoatomic
(2) Helium (B) Tetraatomic
(3) Oxygen (C) Diatomic
(4) Phosphorous (D) Triatomic
a) 1-C, 2-B, 3-D, 4-A
b) 1-D, 2-A, 3-C, 4-B
c) 1-A, 2-C, 3-B, 4-D
d) 1-B, 2-D, 3-A, 4-C

15. Match the following with the correct response. 1


(a) A cloud of smoke in the air (i) Sol
(b) Oil drops in water (ii) Emulsions
(c) Whipped cream (iii) Aerosols
(d) Paints (iv) Foam
a) (a) - (i), (b) - (iii), (c) - (ii), (d) - (iv)
b) (a) - (ii), (b) - (iv), (c) - (i), (d) - (iii)
c) (a) - (iii), (b) - (ii), (c) - (iv), (d) - (i)
d) (a) - (iv), (b) - (i), (c) - (iii), (d) - (ii)

16. Match the following with the correct response. 1


(1) Green manure (A) Earthworms

(2) Vermicomposting (B) Hydrogen

(3) Micronutrient (C) Chlorine

(4) Macronutrient (D) Cluster bean


a) 1-B, 2-D, 3-A, 4-C
b) 1-D, 2-A, 3-C, 4-B
c) 1-C, 2-B, 3-D, 4-A

17. Assertion (A): A bus moving due north takes a turn and starts moving 1
towards the east with the same speed. There will be no change in the
velocity of the bus.
Reason (R): Velocity is a vector quantity.

a) Both A and R are true and R is the correct explanation of A.


b) Both A and R are true but R is not the correct explanation of A.
c) A is true but R is false.
d) A is false but R is true.

18. Assertion (A): Gaseous state of ammonia is not regarded as vapours. 1


Reason (R): As the volume of a substance increases, its density increases.

a) Both A and R are true and R is the correct explanation of A.


b) Both A and R are true but R is not the correct explanation of A.
c) A is true but R is false.
d) A is false but R is true.

19. Assertion (A): Squamous epithelial tissue consists of cube-like cells. 1


Reason (R): They form the delicate lining of cavities (mouth, esophagus,
nose, etc.) and blood vessels.

a) Both A and R are true and R is the correct explanation of A.


b) Both A and R are true but R is not the correct explanation of A.
c) A is true but R is false.
d) A is false but R is true.
20. Assertion (A): The mass of the total number of protons and neutrons is a 1
measure of the approximate mass of an atom.
Reason (R): The mass of an electron is negligible.

a) Both A and R are true and R is the correct explanation of A.


b) Both A and R are true but R is not the correct explanation of A.
c) A is true but R is false.
d) A is false but R is true.

SECTION B
21. A force of 10 N displaces a body by a distance of 2 m at an angle 60o to its 2
own direction. Find the amount of work done.
OR
An object of mass 40 kg is raised to a height of 5 m above the ground. What
is its potential energy? If the object is allowed to fall, find its kinetic energy
when it is half way down.

22. Why do we feel comfortable under a fan when we are perspiring? 2

23. A girl is sitting in the middle of a park of dimension 12 m × 12 m. On the left 2


side of it there is a building adjoining the park and on the right side of the
park, there is a road adjoining the park. A sound is produced on the road by
a cracker. Is it possible for the girl to hear the echo of this sound? Explain
your answer.

24. Name A, B, C, D, E and F in the following diagram showing change in its 2


state.

25. While driving vehicle how does the use of safety belts prevent accidents? To 2
show that a body remains at rest unless acted upon by an unbalanced force,
mention one situation from everyday life.
OR
How much momentum will a dumb-bell of mass 10 kg transfer to the floor if
it falls from a height of 80 cm? Take its downward acceleration to be 10 ms-
2
.

26. Calculate the number of neutrons present in the nucleus of an element 'X' 2
which is represented as 1531X.
SECTION C
27. Study the given below diagram and answer the following questions: 3

i. Identify the application of ultrasound in the above diagram.


ii. Explain the working principle of this medical procedure.
iii. What is the range of frequencies associated with ultrasound?

28. Composition of the nuclei of two atomic species X and Y are given as under: 3

Give the mass numbers of X and Y. What is the relation between the two
species?
29. The velocity of a body in motion is recorded every second as shown- 3
time (s) 0 1 2 3 4 5 6 7 8 9 10
velocity (m/s) 60 54 48 42 36 30 24 18 12 6
Calculate the -
a. acceleration
b. distance travelled and draw the graph.
OR
A cheetah is the fastest land animal and can achieve a peak velocity of 100
kmh-1 up to distances less than 500 m . If a cheetah spots his prey at a
distance of 100 m, what is the minimum time it will take to get its prey, if
the average velocity attained by it is 90 kmh-1?

30. Ram's family was worried about heavy electricity bills to be paid. Their 3
neighbour Mohan suggested some easy and effective steps to reduce the
same. Next month's bill came as a relief to Ram, as the consumption of
electricity had reduced by 50 units and so had the bill.
c. In what other aspects of life can this situation help?
d. What is the unit of energy?
e. Write any three steps that you think Mohan might have
suggested to Ram.
31. The velocity-time graph of an object of mass m = 50 g is shown in the figure. 3
Observe the graph carefully and answer the following questions.
i. Calculate the force on the object in time interval 0 to 3 s.
ii. Calculate the force on the object in the time interval 6 to 10
s.
iii. Is there any time interval in which no force acts on the
object? Justify your answer.

32. What is nucleoid? How it is different from the nucleus of eukaryotic cell? 3
OR
Differentiate between diffusion and osmosis. What is its importance?

33. Study the following figure and answer the following questions: 34

iv. Identify the type of tissue shown in the given figure. Write
the labellings - A, B, C, D.
v. Is the given type of tissue in the figure is flexible or not? Give
a reason for your answer.
vi. What are the functions of the tissue shown in the given
figure?

SECTION D
34. A stone is dropped from the edge of a roof. 5
i. How long does it take to fall 4.9 m?
ii. How fast does it move at the end of that fall?
iii. How fast does it move at the end of 7.9 m?
iv. What is its acceleration after 1s and after 2 s?
OR
i. Write the formula to find the magnitude of the gravitational
force between the earth and an object on the earth's surface.
ii. Derive how does the value of gravitational force F between two
objects change when
i. distance between them is reduced to half and
ii. mass of an object is increased four times.

35. The labelled diagram of endoplasmic reticulum is shown below: 5

Using the above diagram, answer the following questions:


i. Why RER looks rough under a microscope?
ii. Why the ER is involved in membrane biogenesis?
iii. Which type of endoplasmic reticulum is mainly concerned
with the synthesis of proteins?
iv. What is the structure of RER?
v. Which type of endoplasmic reticulum helps in detoxifying
many poisons in the liver of vertebrates?
OR
i. Describe the role played by the lysosomes. Why are they termed
as suicidal bags? How do they perform their function?
ii. What happens to the dry raisins, when placed in plain water for
some time? State the reason for whatever is observed. What
would happen if these raisins are then placed in concentrated
salt solution?

36. Classify each of the following as a physical or a chemical change. Give 5


reasons.
i. Drying of a shirt in the sun.
ii. Rising of hot air over a radiator.
iii. Burning of kerosene in a lantern.
iv. Change in the colour of black tea on adding lemon juice to it.
v. Churning of milk cream to get butter.

SECTION E
37. Read the text carefully and answer the questions: 4
Sound bounces off a solid or a liquid like a rubber ball bounce off a wall.
Like the light, sound gets reflected at the surface of a solid or liquid and
follows the same laws of reflection.
The directions in which the sound is incident and is reflected make equal
angles with the normal to the reflecting surface at the point of incidence,
and the three are in the same plane. If we clap near a suitable reflecting
object such as a tall building or a mountain, we will hear the same sound
again a little later. This sound that we hear is called an echo. The sensation
of sound persists in our brain for about 0.1 s. To hear a distinct echo the
time interval between the original sound and the reflected one must be at
least 0.1s. Hence, for hearing distinct echoes, the minimum distance of the
obstacle from the source of sound must be 17.2 m. This distance will change
with the temperature of air. Another phenomenon of reflection of sound is
reverberation.
A sound created in a big hall will persist by repeated reflection from the
walls until it is reduced to a value where it is no longer audible. The
repeated reflection that results in this persistence of sound is called
reverberation. Excessive reverberation is highly undesirable.
(i) Which of the following is true related to the reflection of sound?
(a) Directions in which the sound is incident and is reflected make equal
angles with the normal to the reflecting surface at the point of incidence.
(b) Incident wave reflected wave and normal lies in the same plane
(c) Both a and b are true.
(d) None of these
(ii) For hearing distinct echoes, the minimum distance of the obstacle from
the source of sound must be
(a) 10m
(b) 20m
(c) 17.2m
(d) None of these
(iii) Excessive reverberation is
(a) Desirable phenomenon
(b) Undesirable phenomenon
(c) Does not exist
(d) None of these
(iv) A person makes a sound near an obstacle and heard the echo after 1 s.
What is the distance of the obstacle from the person if the speed of the
sound, v is taken as 346 m/s?

38. Read the text carefully and answer the questions: 4


Cropping pattern refers to the proportion of land under cultivation of
different crops at different points of time. Intercropping is the
practice of growing more than one crop on the same field at the same
time in a definite row pattern. It is much superior to mixed cropping.
Intercropping can incorporate crop rotation. The technique makes the
farmers busy throughout the year. Productivity is increased. Soil
erosion is prevented. Pests and weeds remain under control.

i. What is Inter-cropping?
ii. Why intercropping is superior to all other means of
cropping?
iii. Which method of cropping prevents soil erosion by
checking soil creep after harvesting a crop?

OR

Modern cropping is based on machines from sowing to


harvesting. Which is the best method of cropping for this?
39. Read the text carefully and answer the questions: 4
Sulfuric acid or sulphuric acid, known in antiquity as oil of vitriol, is a
mineral acid composed of the elements sulphur, oxygen and
hydrogen, with the molecular formula H2SO4. It is a colourless,
odourless and viscous liquid that is miscible with water.

i. How many oxygen atoms are there in a molecule of


sulphuric acid?
ii. Write a physical property of sulphuric acid.
iii. Whether the central atom in sulphuric acid belongs to a
metal or a non-metal? Identify it.

OR

Calculate the number of atoms in one molecule of H2SO4?


What type of molecule is it – monoatomic or polyatomic?
ANSWER KEY
KENDRIYA VIDYALAYA SANGATHAN
RANCHI REGION (2022-23)
CLASS : 9 SUBJECT : SCIENCE
MAX. MARKS : 80 TIME : 3 HRS
S.NO. ANSWERS (SECTION A) MARKS
1. (b) Nitrogen and Oxygen 1
2. (c) DNA and protein 1
3. (d) Vector 1
4. (c) Dr.V. Kurian 1
5. (b) Tendons are non-fibrous tissue and fragile 1
6. (c) Protein synthesis 1
7. (c) 1-D, 2-A, 3-C, 4-B 1
8. (a) Collenchyma, parenchyma, Sclerenchyma 1
9. (c) 15 kg 1
-1
10. (a) velocity = 2ms 1
11. (b) Silver 1
12. (d) muscular 1
13. (b) (a) - (ii), (b) - (iv), (c) - (i), (d) - (iii) 1
14. (b) 1-D, 2-A, 3-C, 4-B 1
15. (c) (a) - (iii), (b) - (ii), (c) - (iv), (d) - (i) 1
16. (b) 1-D, 2-A, 3-C, 4-B 1
17. (d) A is false but R is true. 1
18. (b) Both A and R are true but R is not the correct explanation of A. 1
19. (d) A is false but R is true. 1
20. (a) Both A and R are true and R is the correct explanation of A. 1
SECTION B
21. 10 J ; 2
By definition: Work = Force ×displacement in the direction of force = Fs
cosθ
OR
(PE) = mgh
Decrease in P.E. = Increase in K.E.
1000 J
22. The sweat is readily evaporated from the body by the air from the fan. 2
Evaporation of sweat from skin causes cooling sensation. As a result,
we feel comfortable under a fan.
23. If the time gap between the original sound and reflected sound 2
received by the listener is around 0.1s, only then the echo can be
heard.
Velocity of sound in air = 344 m/s.
Time interval = 0.1 s
Therefore, The minimum distance travelled by the reflected sound
wave for the distinctly listening to the echo = Velocity of the
sound × Time interval = 344 × 0.1 = 34.5 m
Since, the girl is sitting in middle of a park of dimension 12 m ×12 m,
Therefore; The distance travelled by the sound reflected from the
building and then reaching to the girl = (6 + 6) = 12 m, which is much
smaller than the required distance. Therefore, Girl will not hear
any echo of sound.
24. A – Fusion (Heating – Melting) 2
B – Vaporisation
C – Cooling – Condensation (Liquefaction)
D – Cooling – Freezing (Solidification)
E – Sublimation
F – Solidification or sublimation.
25. First law of motion-make the forward motion slower during accidents. 2
Example: Pedaling of bicycle.(And explanation)
OR
υ2=u2+2as
the momentum transferred to the floor = mυ
40kgms−1
26. From the given information:- 2
A = Mass number of element 'X' = 31
Z = Atomic number of element 'X' = 15
Number of neutrons = Mass number - Atomic number = A - Z = 31 - 15
= 16
The number of neutrons present in the nucleus of element 'X' is 16.
SECTION C
27. i.Ultrasound is used to monitor the development of the foetus inside 3
the mother's womb.
ii.The ultrasound scanner transmits ultrasound into the mother's body
and receives echoes formed by the reflection of ultrasound from
inside. The ultrasound echoes form a picture of a developing baby on a
monitor which helps the doctor to keep a track of the developing baby.
Thus ultrasonography is used for the examination of the foetus during
pregnancy to detect any growth and abnormalities.
iii.The sound above 20,000Hz is regarded as ultrasound
28. The mass number of X = Number of protons + Number of neutrons = 6 3
+ 6 = 12
The mass number of Y = Number of protons + Number of neutrons = 6
+ 8 = 14
Since the number of protons (6) in the two species is the same and the
atomic mass of the two species is different (12 and 14), the given
atomic species are isotopes of the same element (with atomic number
Z = 6).
29. a. a=−6m/s2 3
b. Distance = S= 300m
OR
time to catch the prey = 4S
30. a.Promotes saving habits and judicial uses of resources and save 3
resources
b.Joule.
c.
i.Switch off the power where it is not need.
ii.Use of CFL light.
iii.To check the proper wiring as to avoid leakage of power.

31. i. Force, F1 = ma = 2 N 3
ii.Force, F2 = ma2 = -1.5 N.
iii. Acceleration= '0' m/s2. Therefore, F= '0' N.
32. In prokaryotic cell, genetic material in not surrounded by membrane. 3
This undefined region where genetic material is present, is known as
nucleoid.
In eukaryotic cell, genetic material is surrounded by nuclear
membrane.
OR
OSMOSIS DIFFUSION
It involves the movement of It involves the movement of
solvent molecules solute molecules
Molecules move from a lower Molecules move from higher
concentration of solute to a concentration of lute to a
higher concentration of lower concentration of
solute solute
It occurs only across a semi- It does not require semi-
permeable membrane permeable membrane
Example: Shrinking of Potato
Example: Spreading of ink
slice when kept in
when a drop of it is put in a
concentrated sucrose
glass of water.
solution
Importance – diffusion and osmosis are important for the transport of
substances across the cell membrane.
33. i.The tissue shown is collenchyma tissue. 3
The labelling of the collenchyma tissue is as follows:
A. - Wall thickenings
B. - Nucleus
C. - Vacuole
D. - Cell wall
ii.Yes, the collenchyma tissue is flexible. This is so because collenchyma
cells don't have lignin in their cell wall.
iii.The function of collenchyma tissue are as follows:
a. Collenchyma tissue provides flexibility to the
plant.
b. It also provides mechanical support to plants.
SECTION D
34. I. t=1 s 5
ii. v = 9.8 m/s
iii. v=12.44 m/s
The stone will be moving with a speed of 12.44 m/s at the end of 7.9 m.
iv. During the free fall the acceleration produced in a body remains
constant.
So, acceleration after 1 s = 9.8 m/s2
Acceleration after 2 s = 9.8 m/s2
OR
i. Formula to find the magnitude of gravitational force:
F = GMmR2.
ii.a. F' = GMm(R2)2 = 4GMmR2 = 4F
ii.b. F' = GM(4m)R2 = 4F
35. i.RER looks rough under a microscope because it has ribosomes 5
attached to its surface.
ii.The RER synthesises proteins and SER synthesises fat molecules or
lipids out of which some of the proteins and lipids help in building the
cell membrane. Hence, ER is involved in membrane biogenesis.
iii.RER is mainly concerned with the synthesis of proteins as ribosomes
are attached to its membranes which are the sites of protein synthesis.
iv.The rough endoplasmic reticulum (RER) is a large network of
membrane-bound tubes and sheets which looks like long tubules or
round or oblong bags (vesicles) having ribosomes attached to its
membrane.
v. SER plays a crucial role in detoxifying many poisons and drugs in the
liver of vertebrates.
OR
i. a. Lysosomes are membrane-bound sacs filled with digestive
enzymes. These enzymes are made by the rough endoplasmic
reticulum.
i.b. Lysosomes compose a kind of waste disposal system of the cell.
During the disturbance in cellular metabolism, e.g. when a cell gets
damaged, lysosomes present in the cell may burst and the enzymes
digest the damaged cell. Hence, lysosomes are called as ‘suicidal bags’
of a cell.
i.e. Lysosomes break up the foreign materials entering into the cell,
such as bacteria or food into small pieces.
ii. The dry raisins, when placed in plain water for some time will swell
up due to endosmosis. If these raisins are again placed in a
concentrated salt solution, they will shrink, due to exosmosis.
36. It is a physical change because moisture in the shirt is converted from 5
its liquid state to gaseous state because of the heat of the Sun.
It is a physical change because water in the radiator is converted from a
liquid state to gaseous state.
It is a chemical change because combustion of kerosene occurs and
new products are formed.
It is a chemical change because there is a reaction between citric acid
present in lemon and the compounds of the tea resulting in the
formation of new products.
It is a physical change because the cream suspended in milk is
separated by churning (centrifugation).
SECTION E
37. i. (c) Both a and b are true. 4
ii. (c) 17.2m
iii.(b) Undesirable phenomenon
iv. 173 m.
38. i. The various cropping seasons in India are Rabi crop, (1+1+2)
Kharif crop and Zaid crop.
ii. Rabi crops are sown during the winter season which
requires less water.
Kharif crop is sown during the summer/rainy
season which requires abundant water.
iii. Farmers are suggested to grow drought-resistant crops
that can mature early. Along with this farmers are
advised to use manure for their fields as it increases the
water-holding capacity of the soil.
OR
There is a short season between Kharif and Rabi season in the months
of March to July. The crops that grow in this season are Zaid crops.
These crops are grown on irrigated lands and do not have to wait for
monsoons. Some examples of Zaid types of crops are pumpkin,
cucumber, and bitter gourd.
39. i. 4 O-atoms (1+1+2)
ii. It is a colourless, odourless and viscous liquid that is miscible
with water (any one physical property)

iii. In H2SO4; the central atom is S (Sulphur), which is a non-


metal.

OR

iii. There are seven atoms in one molecule of sulphuric acid. It is


a polyatomic molecule.
KENDRIYA VIDYALAYA SANGATHJAN RANCHI REGION CLASS IX
SUB- SCIENCE FM 80
Time allowed: 3 hours
General instructions:
(i) The question paper consists of 39 questions in section 5
(ii) All questions are compulsory. However, an internal choice is
provided in some questions. A student is expected to attempt ony one
of these question
(iii) Section A consists of 20 objective questions carrying 01 mark
each.
(iv) Section B consists of 6 Very Short questions carrying 02 marks
each. Answer to these question should in the range of 30 to 50 words
(v) Section C consists of 7 Short Answer type questions carrying 03
marks each. Answer to these questions should be in the range of 50
to 80 words
(vi) Section D consists of three long answer type questions carrying
05 marks each. Answer to these questions should be in the range of
80 to 120 words.
(vii) Section E consists of 3 source based/case-based units of
assessment of 04 marks each with sub-parts.
Section A (1 mark)

Q1. When heat is constantly supplied by a gas burner to boiling water, then the
temperature of water, during vapourisation
(a) rises slowly. (b) does not rise at all.
(c) rises until steam is produced. (d) rises and then becomes constant.
Q2. Which of the following is homogeneous in nature?
(i) Ice (ii) Wood
(iii) Soil (iv) Air
Q3. Modern atomic symbols are based on the method proposed by
(a) Dalton (b) Berzelius
(c) Bohr (d) Mendeleev
Q4. Rutherford’s alpha particle scattering experiment was responsible for the
discovery of:
(a) Atomic Nucleus (b) Electron
(c) Proton (d) Neutron
Q5. An atom with 3 proton and 4 neutron will have a valency of
(i) 3 (ii) 7 (c) 1 (d)4

Q6. Isotopes of an element have


(i) the same physical properties (ii) different chemical properties
(iii) different number of neutrons (iv) different atomic numbers
Q7. Which out of the following is not a function of the vacuole?
(a) Storage (b) Providing turgidity and rigidity to cell
(c) Waste excretion (d) Locomotion
Q8. Which of the following are covered by a single membrane?
(a) Mitochondria (b) Vacuole
(c) Lysosome (d) Plastid
Q9.The image shows the stem of a plant.

Which type of meristematic tissue is present at the labeled part ‘X’?


(a) lateral meristem (b) intercalary meristem
(c) apical meristem (d) both apical and lateral meristem
Q10. Flexibility in plants is due to
(a) collenchyma (b) sclerenchyma
(c) parenchyma (d) chlorenchyma
Q11. A vector quantity has
(a) direction (b) magnitude
(c) both magnitude and direction (d) none of these
Q12. Which of the given example can be illustrated using the Newton’s first
law of motion?
(a) Rocket repulsion (b) Releasing an arrow from bow
(c) Wearing a seat belt in a car (d) Rowing of boat
Q13. At which of the following locations is the value of g largest?
(a) On top of the Mount Everest (b) On top of Qutub Minar
(c) At a place on the equator (d) A camp site at Antarctica
Q14. The S.I unit of buoyant force is
(a) pascal (b) joule
(c) newton (d) kg m/s
Q15. A flying aeroplane possesses
(a) only potential energy (b) only kinetic energy
(c) both potential and kinetic energy (d) neither potential nor kinetic energy
Q16. When we go from solid to gaseous state, the speed of sound
(a) increases (b) decreases
(c) remains same (d) none of these

Assertion based Questions


The following questions consist of two statements – Assertion (A) and
Reason(R). Answer these questions seleccting the appropriate option given
below:
(a) Both A and R are true and R is the correct explanation of A
(b) Both A and R are true but R is the correct explanation of A.
(c) A is true but R is false
(d) A is false but R is true.
Q17. Assertion (A): The valency of aluminium is 3 and oxygen is 2.
Reason (R): The chemical formula of aluminium oxide is Al3O2
Q18. Assertion (A): Permanent tissues are derived from the meristematic
tissue once they lose the ability to divide.
Reason (R): Meristematic tissue is the dividing tissue present in the
growing regions of the plant.
Q19. Assertion (A): A boy gets completely exhausted in trying to push a
stationary wall.
Reason (R): Work done by the boy on the wall is zero.
Q20. Assertion (A): Fodder crop like barseem, oats, etc. are also grown along
with the cereal and pulses.
Reason (R): Fodder crop is food for the livestock.
Section B (2marks)
Q21. Which gas is called dry ice? Why?
OR
Suppose you want to convert a gas into a liquid, which two methods can you
apply?
Q22. Write the chemical formula of the following:
(a) Magnesium chloride (b) Calcium oxide
(c) Aluminium chloride (d) Calcium carbonate
Q23. Give one example of polyatomic cation and a polyatomic anion.
Q24. Which organelle is known as the powerhouse of the cell? Why?
OR
Q. What is plasmolysis? Give one example for the same in our daily life.
Q25. A force of 1 N acts on a body of mass 10 kg. As a result, the body covers
100 cm in 4 second, moving away a straight lin. Find the initial velocity.
Q26. When a carpet is beaten with a stick, dust comes out of it. Explain.

Section C (3 marks)
Q27. Explain why:
(i) Wet clothes dry quickly in the sun than in shade.
(ii) Trees acquire more leaves during summer.
(iii) We feel comfortable under a fan when we are perspiring.
Q28. How is bacteria cell different from onion peel?
Q29. Describe the structure and function of stomata.
OR
Q. Why are xylem and phloem called complex tissues? How are they different
from one other?
Q30. A trolley, while going down an inclined plane, has acceleration of 2 cms-2.
What will be its velocity 3s after the start?
Q31. Give any three applications of Archimedes’ principle.
Q32. What is energy? What is the unit of energy? Give any two uses of kinetic
energy.
OR
Q. Give one example each of potential energy (i) due to position (ii) due to
change.
Q33. Write the differences between transverse waves and longitudinal waves.
Section D (5 marks)
Q34. The given figure depicts the atomic structure of an atom of an element
‘X’. Write the following information about the element ‘X’
(i) Atomic number of ‘X’ (ii) Atomic mass of ‘X’
(iii) Valence electrons (iv) Valency of ‘X’
(v) ‘X’ should be metal or non-metal.

Q35. What is a neuron? Write the structure and functions of neuron.


OR
Q. Briefly describe strained and smooth muscles with their functions.
Q36. Describe composite fish culture system. What is the major problem is fish
farming? How is it overcome?
Section E (4 marks)
Read the following passages and answer the following questions that follow:
Q37. In a class of chemistry test which was based on observation some
elements were provided named P, Q, R, S, T with the following data. Teacher
informed the students that out of all these atoms it may be a noble gas, cation
or anion. The teacher provided some information regarding that which is given
in the table.
Elements P Q R S T
Atomic Mass 4 27 40 20 14
Number of neutrons 2 14 22 10 7

He has asked students to find the following:


(A) Find which elements are noble gas, anion, and cation respectively.
(B) Symbolise the element
(C) Why do elements react?
OR
Q. Salima and sheenu have a few doubts over the number of atoms in Helium
Salima says Helium atom has 2 electrons and its valence shell but its valency is
not 2. According to Sheenu Helium has 2 electrons and also its valency is 2. You
are required to work as an arbitrator to solve the doubts of Salima and Sheenu.
Similarly, they both have doubts of Salima and Sheenu. Similarly, they both
have doubts regarding the valency and valence electrons of elements and are
not able to recognize the difference between valence electrons and the valency.
They have the following doubts.
(A) What is the valency of He?
(B) How many valence electrons does each noble gas have?
(C) Why is the valency of inert gases zero?
(D) Why are noble gases unreactive in nature?
Q38. Dr. Parimal, professor of botany went to Botanical Garden of Noida where
many students had come on tour to visit that place. Students were very excited
as that place had many plants having colourful flowers and different varieties
of fruits. Some flowers were white and unique fruits were green in colour in the
park, pleasant fragrance can be smelled due to flowers
Lots of birds and insects were seen who were nurturing themselves from fruits
and flowers, also helping in pollination and fruit dispersion. Dr Parimal and out
of curiosity asked the following questions:
(A) Which process is involved for the pleasant fragrance spread everywhere?
(a) Osmosis
(b) Reverse Osmosis
(c) Diffusion
(d) Evaporation
(B) Which plastid gives colours to flowers and fruits?
(a) Chloroplast (b) Chromoplast
(c) Leucoplast (d) Plasmolysis
(C) Why were unripe fruits green in colour?
(D) Can we find mutualism in between plants and flowers?
(E) The bright colours of ripe fruits is due to
(a) Chromoplasts (b) Chloroplasts
(c) Amyloplasts (d) Leucoplast
OR
Q. A membrane surround each cell, separately its contents from the outside
world. To sustain their sophisticated structure and function large and complex
cells, such as those seen multicellular animals, require a lot of chemical activity.
These cells uses membrane-bound tiny structures within themselves to keep
different kind of activities apart from one another. The cytoplasm is a jelly-like
fluid that fills the space between the plasma membrane’s outer layer, and
includes a variety of specialized cell organelles. Endoplasmic reticulum, Golgi
apparatus, Lysosomes, Ribososmes, Nucleus, Chloroplast, Mitochondria and
Plastids are some of the organelles found in the cells. For the cell, each of these
organelles have a distinct purpose. A electron microscope is required to see
some of these organelles. They’re significant since these play a key role in cell
function.
(A) What are cell organelles, and their function?
(B) Name any five cell organelles.
(C) Name the jelly-like fluid substance present in cells.
(D) The largest cell organelles in the cell are:
(a) Plastids (b) Golgi bodies
(c) Mitochondria (d) Chromosomes
(E) Which of the following cell organells is called digestive bags?
(a) Nucleus (b) Lysososomes
(c) Chloroplast (d) Mitochondria
Q39. Thrust is the force applied on an object perpendicular to the surface.
When you stand on loose sand, the force of your body weight acts on an area
equal to the area of your feet. When you lie down, the same force acts on an
area that is equivalent to your entire body’s contact area, which is larger than
the area of your feet. As a result, forces of the same magnitude have varied
impacts on different locations.
The thrust is the same in all of the examples above. However the outcomes are
not the same. As a result, the thrust’s effect is determined by the area on which
it acts. When standing, the effect of thrust on sand is greater than when lying.
(A) What is the SI unit of thrust?
(a) Newton (b) kg m/s
(c) Pascal (d) kg/ms2
(B) Forces of the same magnitude have different impacts on various
locations. State True or False.
(C) Define atmospheric pressure?
(D) It is easy for camel to walk in sand but difficult for humans. Give reason.
(E) The SI unit of Gravitational constant is:
(a) Nm2 kg-2 (b) Nm-2 kg-2
(c) Nm-2 kg2 (d) None of these
OR
Q. There was a game at the school fair where students had to find the heaviest
ball without holding it in their hand. Three balls were given out, with only a few
disposable glasses remaining. Rakesh observed his friend attempting to win the
game but failing to locate the heaviest ball. Rakesh assisted him by dipping the
three balls one by one in the disposable glasses filled to the brim with water
and they eventually won the game.
(A) Why did Rakesh Tell his friend to dip the balls one by one in a glass of
water that was entirely filled?
(B) Identify the principle applied in this game.
(C) Define density.
(D) Which quantity gives the opposite effect on pressure?
(a) Area of surface
(b) Volume of object
(c) Length of object
(d) Height of object
(E) Which of the following statement is incorrect about buoyant force?
(a) Buoyant force is exerted by liquid in the opposite direction of pressure.
(b) It works in the opposite direction of gravity.
(c) It depends on the density.
(d) It depends on the velocity.
-----------------------------------------------x----------------------------------------------x-------------------------
KENDRIYA VIDYALAYA SANGATHAN
CLASS IX
MARKING SCHEME
SUB—SCIENCE FM 80

:
Section A
1 B 1
2 C 1
3 B 1
4 A 1
5 D 1
6 C 1
7 D 1
8 B 1
9 B 1
10 A 1
11 C 1
12 C 1
13 D 1
14 C 1
15 C 1
16 B 1
17 C 1
18 A 1
19 A 1
20 A 1

Section B
21 Solid CO2 is known as dry ice. This is because it directly gets 2
converted into gaseous state without passing through liquid
state on decreasing the pressure to 1 atmosphere.
OR
By increasing pressure and lowering temperature we can
convert a gas into a liquid.
22 (a) MgCl2 (b) CaO 1/2x4
(c) AlCl3 (d) CaCO3

23 Polyatomic cation: NH3+ 1+1


Polyatomic anion: SO42-

24 Mitochondria are known as powerhouse of the cell because 2


these are the sites of cellular respiration. They release energy in
the form of ATP (Adenosine Triphosphate). This energy is then
utilised by the organelles to carry out their basic functions.
OR
When a living plant cell loses water through osmosis, there is
shrinkage or contraction of the contents of cell away from the
cell wall. This is known as plasmolysis. For example of
vegetables in hypertonic conditions or the loss of water by
sprinkling salt on cucumber.
25 Here, acceleration, a = F/m =1/10 = 0.1ms-2 = 1o cm/s-2 1/2x2
We have,
s = ut + 1/2at2
or 100 = u x 4 + ½ x 10 x (4)2
or 100 = 4u +80
or 20 = 4 u
u = 20/4 = 5 cm/s
26 When the carpet is beaten, it is suddenly set into motion. The 2
dust particles tend to remain at rest due to the inertia of rest,
therefore, dust comes out of it

Section C
27 (i) The temperature in the sunny area is higher than in 1x3=3
the shade and evaporation takes place at a faster rate at
high temperature. Hence wet clothes dry quickly in the
sun.

(ii) During summer temperature is generally very high. In


order to keep cool, a tree must transpire(transpiration is
a phenomenon of evaporation of water from the leaves)
more to keep itself cool. More transpiration requires
more leaves. Hence, a tree acquires more laves during
summer.

(iii) The sweat is readily evaporated from the body in the


air from the fan. As a result, we feel comfortable under
a fan.

28 Bacteria cell Onion peel 1x3


1. Size is small 1. Size is large
2. Nucleus is absent 2. Nucleus is present
3. It is a prokaryotic cell. 3. It is eukaryotic cell.
4. Cell division takes place 4. Cell division takes place
by fission or building by mitosis.

29 Stomata are small pores present in the epidermis of 1


leaves and are enclosed by two kidney-shaped cells 1+1
called guard cell.
Function of stomata:
(i) Necessary for exchanging gases with the atmosphere
during photosynthesis and respiration.

(ii) It helps in transpiration, i.e., loss of water from the


plant takes place through them.
OR
Xylem and phloem are called as complex tissues as they
are made up of more than one type of cells. Following
are the differences between xylem and phloem.
Xylem Phloem
1. Xylem mainly consists 1. Phloem consists of
of dead cells(except xylem living cells(except phloem
parenchyma) fibre).
2. It conducts water and 2. It translocates
minerals from roots to prepared food from
aerial parts of the plant. leaves to storage organs
and growing parts of the
body.

30 Here, 3
Initial velocity, u = 0
Acceleration, = 2 cms-2
Time, t = 3s
Final velocity, v =?
From the equation of motion, v = u + at
= 0 + 2 cms-2 x 3s =
6cms-1

31 It is used in 3
(i) designing of ships and submarines.
(ii) Hydrometers, which are used for determining density
of liquids.
(iii) Lactometers, used to determine the purity of a
sample of milk.

32 The capacity of a body to do work is called energy 1


possessed by the body. It is a scalar quantity and is +
measured in joule(J). 1/2x4=2
Generally, for practical purposes, a bigger unit called
kilojoule(kJ) is used ( 1kJ = 1000 J).
Two uses of kinetic energy are:
(i) The kinetic energy of air is used to run windmills.
(ii) The kinetic energy of the running water is used to
generate electricity.
OR
(i) Potential energy due to position: Water stored in dam
has potential energy.

(ii) Potential energy due to shape: In a toy car, the


wound spring possesses potential energy, and as the
spring is released, its potential energy changes into
kinetic energy due to which the car moves.
33 Transverse waves Longitudinal waves
1. The particles of the medium 1. The particles of the medium
oscillates up and down an=bout move in the parallel to the direction
their mean position. of propagation of the disturbance.
2. They propagates as crests and 2. They propagate as compressions
troughs. and refractions.
3. The propagation of waves is 3. The propagation of these waves is
possible in solid or surface of possible in solids, liquids and gases.
liquid but not in gases.
Example: Sound wave
Example: Light wave

Section D
34 (i) Atomic number = Number of protons = 8 1x5
(ii) Atomic mass = Number of protons + Number of
neutrons = 8 +10 = 18u
(iii) Valence electrons = 6
(iv) Valency of ‘X’ = 8 -6 =2
(v) ‘X’ should be non-metal because there are six valence
electrons hence it will take two more electrons to complete
its outermost shell.
35 Nervous tissue contains highly specialised unit cells called 1x3
nerve cells or neurons. Each neuron has the following three +
parts: 2
(i) The cyton or cell body: It contains a central nucleus and
cytoplasm with characteristic deeply stained particles, called
Nissl granules.
(ii) The dendrites: These are short processes arising from the
Cyton.
(iii) The axon: It is a single, long, cylindrical process of uniform
diameter. It carries impulses away from the cell body.
Functions:
Neurons have the ability to receive stimuli from within or
outside the body and conduct impulses to different parts of
the body. The impulses travel from one neuron to another
neuron and finally to the brain or spinal cord.
OR
The striated muscle fibres are long are elongated, non-
tapering , cylindrical and unbranched. These cells have a
number of nuclei and so called multinucleate. These muscle
fibres show alternate dark and light stripes or striations and
so they are called as striated muscles. These muscles occur in
muscles of limbs, body wall, face, neck, etc.
Functions of striated muscles:
(i) Striated muscles are powerful and undergo rapid
contraction and explanation.
(ii) Striated muscles provide the force for locomotion and all
other voluntary movements of the body.
The smooth muscles are also known un-striated or
involuntary muscles. Smooth muscles occur as bundles or
sheets of elongated fusiform or spindle-shaped cells or fibres.
They are held bands, stripes or striation across them.
These muscles are found in the walls of the alimentary canal
and internal organs, ducts of glands and blood vessels.
Smooth muscles are also found in the stomach, intestine,
uterus, bronchi, iris, of the eye, etc.
Functions of smooth muscles:
(i) Smooth muscles do not work according to our will, so they
are also called involuntary muscles. Movement of food in the
alimentary canal or the contraction and relaxation of blood
vessels are involuntary movements.
(ii) Smooth muscles contract slowly but can remain contracted
for a long period of time. Due to this characteristic, the food
passes to the next step of digestion in the alimentary canal.
36 By adopting composite fish culture system, intensive fish 1
farming can be done. Both local and imported fish species are +
used in such systems. In such a system, a combination of the 1
five or six fish species in a single find pond. These species are +
selected in a such a way that they have different types of food 1
habits and don’t compete for food among themselves. As +
result the, the food available in all the parts of the pond is 1
used. +1
For example: Catlas are surface feeders, Rohus feed in the
middle-zone of the pond, Mrigals and Common Carps are
bottom feeders, and Grass carps feed on the weeds. Together
these species can use all the food in the pond without
competing with each other. This naturally increases the fish
yield from the pond.
One problem with the such a composite fish culture is that
many of these fishes breed only during monsoon. Even if fish
seed is collected from the wild, the can be mixed with that of
other species as well. So a major problem in fish farming is the
lack of availability of good quality seed. To overcome this
problem, now some ways have been worked out to breed
these fish in ponds using hormonal situation. This has ensured
the supply of pure fish seed in desired quantities.

Section E
37 1x4
Elements P Q R S T
Atomic 4 27 40 20 14
mass
Number of 2 14 22 10 7
neutrons
Number of 2 13 18 10 7
protons =
atomic
mass –
Number of
Neutrons
Number of 2 13 18 10 7
Electrons
Electronic 2 2, 8, 3 2, 8, 8 2, 8 2, 5
configurati
on
Type Noble Cation Noble Noble Anion
gas gas gas
Element Helium(H Aluminium( Argon( Neon(N Nitrog
name with e) Al) Ar) e) en(N)
symbol
(A) Noble gases: P, R, S, and T
Anion ---> T
Cation ---> Q
(B) Symbol of elements :
P- He, Q- Al, R- Ar, S- Ne, T- N
(C) All elements apart from the noble gases, are chemically reactive
because they share electrons with atoms. When this happens the
outer shell of the electrons in each atom from a magic number.
OR
(A) Helium has only one shell (K-shell) and the maximum number of
electrons within the K shell can be 2. Hence the valency is zero.
(B) The number of valence electrons present in noble gases is 8.
Noble gas is present in the group 18, the electronic configuration of
every noble gas element is such that the outermost shell of the
atom is full which implies that it does not have any electron to
donate or accept.
(C) The atoms of noble gases already have complete outer shells, so
they do not lose, gain, or share electrons. This is why the noble
gases are inert and do not take part in chemical reactions. Inert
gases have very high ionization enthalpy because they have a
stable electronic configuration. Therefore they do not tend to lose
electrons. Thus, the valency of inert gases is zero.
(D) When elements react, their atoms complete their outer shells
by losing, gaining, or sharing electrons. The atoms of noble gases
already have complete outer shells, so they have no tendency to
lose gain, or share electrons. This is why the noble gases are inert
and do not take part in chemical reactions.

38 (A) (c) Diffusion 1x4


(B) (b) Chromoplast
(C) Chloroplasts are present in the skin of unripe fruits. As fruits
started ripening chloroplasts changed into chromoplasts which
provide attractive colours like red, yellow, orange, brown colours.
(D) Yes, we can find mutualism between plants and birds. While
eating the flesh of fruits, the animals pick up the seeds and
transport them to various locations. Seeds distribution is aided by
this. As a result both animals and plants ae benefitted. It is a form
of mutualism.
(E) Chromoplasts
OR
(A) Cell organelles are the cellular components. These cell
organelles include membrane bound and non-membrane bound
organelles that are present within the cells and have different
structures and activities. For the cell’s regular functioning, they
coordinate and function effectively. A handful of them serve to
form and maintain the cell, while others are engaged in cell motility
and reproduction. Organelles are found throughout the cell and are
divided into three groups based on whether or not they have a
membrane.
(B) Endoplasmic reticulum, Golgi apparatus, Lysosomes,
Ribosomes, Nucleus, Chloroplasts, Mitochondria and Plastids.
(C) Cytoplasm is a viscous liquid that fills each cell and is
surrounded by the cell membrane. Water, salts, and proteins make
up the majority of it. The cytoplasm of eukaryotic cells contains all
of the material inside the cell and outside of the nucleus.
(D) (a) Plastids
(E) (b) Lysosomes
39 (A) (a) Newton 1x4
Explanation: Thrust is the force applied on an item perpendicular
to the surface. The SI unit of thrust is Newton and it is a vector
quantity.
(B) True
Explanation: The effect of force depends upon area on which force
is exerted.
(C) The force per unit area exerted on the Earth’s surface by the
atmosphere is known as atmospheric pressure. The weight of air
molecules in the atmosphere causes atmospheric pressure.
(D) It is because camel’s feet have large surface area, the force of
their weight is distributed over a large area of sand. The pressure
produced on sand is small hence, a camel’s feet do not sink into the
sand. But in case of humans, the weight is exerted on a small area
which increases the pressure and foot sinks.
(E) (a) Nm2 kg-2

OR
(A) Rakesh wanted to see how much water each ball displaces
when dipped in water.
(B) The principle utilized is known as the ‘Archimedes principle’.
(C) Density is defined as mass per unit volume of a substance
Density = Mass/ Volume
SI unit = Kg/m3
(D) (a) Area of object
Explanation: As we know that,
Pressure = Force x 1/Area
So, we can say that when Area will increase, pressure will be
decreased and vice-versa.
(E) (d) It depends on the velocity
Explanation: Buoyant force is the net upward force of any object in
any fluid. If the buoyant force is greater than the object’s weight,
the object will rise to the surface and float. If the buoyant force is
less than the object’s weight, the object will sink, it depends on the
density of liquid. This force always works in the opposite direction
of acceleration.
------------------------------------------------x-------------------------------------------------------
SAMPLE PAPER
KENDRIYA VIDYALAYA SANGATHAN
RANCHI REGION
(2022-23)
Science
Class – IX
Time: 3 Hrs. Full Marks - 80

General Instructions:
(i) All questions are compulsory.
(ii) The question paper has five sections and 39 questions. All questions
are compulsory.
(iii) Sections-A has 20 questions of 01 marks each; Section-B has 6
question of 02 marks each; and Section-C has 7 of 03 marks each; and
Section-D has 3 case based question of 04 marks each; Section-E has
3 question of 05 marks each.
(iv) Internal choices have been provided in some questions. A student has
to attempt only one of the alternatives in such questions.

(MCQ) (Section – A) 1 X 20 = 20 Marks


1. The property to flow is unique to fluids. Which one of the following statement
is correct?
a. Only gases behave like fluids
b. Gases and solids behave like fluids
c. Gases and liquid behave like fluids
d. Only liquid are fluids
2. When iron and sulphur are heated at high temperature
a. Black coloured FeS is formed
b. Mixture of iron and sulphur is obtained
c. Yellow coloured iron sulphide is formed
d. They don’t heat
3. The proteins and lipids, essential for building the cell membrane are
manufactured by
a. Rough endoplasmic reticulum
b. Golgi apparatus
c. Plasma membrane
d. Mitochondria
4. Which of the following can be made into crystal?
a. A bacterium
b. An Amoeba
c. A virus
d. A Sperm
5. The percentage of hydrogen in water is
a. 1.11%
b. 11.11%
c. 8.89%
d. 88.9%
6. A load of 50 kg is pulled upward by 5m, the work done by the external force is
a. 250 J
b. 2500 J
c. -250 J
d. -2500 J
7. Retardation of a body is expressed in
a. m
b. ms-1
c. ms-1
d. ms-2
8. Intestine absorbs the digested food materials. What type of epithelial cells is
responsible for that?
a. Stratified squamous epithelium
b. Columnar epithelium
c. Spindle fibers
d. Cuboidal epithelium
9. Survival of plants in terrestrial environment has been made possible by the
presence of
a. Intercalary meristem
b. Conducting tissue
c. Apical meristem
d. Parenchymatous tissue
10. When we change feeble sound into loud sound we increase its
a. Frequency
b. Amplitude
c. Velocity
d. Wavelength
11. An atom with 3 protons and 4 neutrons will have a valency of
a. 3
b. 4
c. 1
d. 7
12. Anode ray are produced when
a. Cathode is performed
b. Anode is performed’
c. When low voltage is passed
d. When pressure is high
13. The relative atomic masses of many elements are not whole number
because
a. Of the presence of impurities
b. They cannot be determined accurately
c. Of the existence of isotopes
d. None of these
14. An object of mass 2 kg is sliding with a constant velocity of 4 m/s on a
frictionless horizontal table the force required to keep the object moving with
same velocity is
a. 2 N
b. 8 N
c. 32 N
d. 0 N
15. If the radius of earth were to shrink by 1% its mass remaining the same the
acceleration due to gravity on earth surface would
a. Decrease
b. Increase
c. Remain unchanged
d. Can’t be calculated
16. At which of the following locations is the value of g largest
a. On top of the Mount Everest
b. On top of Qutub Minar
c. At a place on the Equator
d. A camp site in Antarctica

Assertion-Reason Question
The following questions consist of two statements – Assertion (A) and Reason
(R). Answer these questions selecting the appropriate option given below :
(a) Both A and R are true and R is the correct explanation of A.
(b) Both A and R are true but R is not the correct explanation of A
(c) A is true but R is false.
(d) A is false but R is true.

17. Assertion (A) : Molecular weight of oxygen is 16


Reason (R) : Atomic weight of oxygen is 16
18. Assertion (A) : All the plant tissues divide into throughout their life.
Reason (R) : Metristematic tissues are localized in certain regions
19. Assertion (A) : A moving hammer drives a nail into wood
Reason (R) : A moving hammer has potential energy in it.
20. Assertion (A) : There is a need for sustainable practice in agriculture
Reason (R) : Population of the country is still growing

(Section – B) 2 X 6 =12 Marks


21. Why do the gases exert more pressure on the walls of the container than the
solids ?
OR
Which of the following diffuses faster ?
Water vapour, wax or, ethyl alcohol.

22. What do you mean by Plasmodesmata?


23. Write the atomicity of following molecules
a. Sulphur
b. Phosphorus
24. Write the Cations and Anions present in the following compound
a. CH3COONa
b. NaCl
c. H2
d. NH4NO3

OR
Which cell organelle controls most of the activities of the cell ?

25. One Electron is present in the outermost shell of the atom of an element X,
what would be the nature of value of charge on the ion formed if this electron
is removed from the outermost shell.
26. In the atom of an Element X, 6 Electrons are present in the outermost shell, if
it acquires noble gas configuration by accepting requisite no of electron, then
what would be the charge on the ion so formed.

(Section – C) 3 X 7 = 21 Marks

27. What do u understand by the term “Latent heat of fusion” ? How much is the
latent heat of ice?
28. What are Asters and Spindle fibres?
29. A) What do you understand by instantaneous velocity?
B) What is negative acceleration?
OR
How are messages conveyed from one place to another within the body ?
30. Write the short not on “Phellogen”?
31. Why does a body reach the ground quicker at poles than at the equator when
dropped
from the same height?
32. A) what kind of energy transformation takes place when a body is dropped
from a certain height?
B) What is a commercial unit of energy?
OR
What is energy ? What is unit of energy ? Give any two uses of kinetic energy.
33. What are longitudinal and transverse waves? Give two examples of each.

(Section – D) 4 X 3 = 12 Marks
34. Differentiate between mitosis and meiosis.
OR
Draw a labelled diagram of mitochondria. Write the functions of mitochondria.

35. The ratio of the radii of hydrogen atom and its nucleus is ~105. Assuming the
atom and the nucleus to be spherical, (i) what will be the ratio of their sizes? {ii) If
atom is represented by planet Earth ‘Re’ = 6.4 x 106m. Estimate the size of the
nucleus.
OR
What do you mean by plasmodesmata ?
36. Give three differences between acceleration due to gravity (g) and universal
gravitational Constant (G).

OR
The earth attracts an apple. Does the apple also attract the earth ? If it does,
why does the earth not move towards the apple ?

(Section – E) 5 X 3 = 15 Marks
37. Iron filings and Sulphur were mixed to gather and divided into two parts ‘A’ and
‘B’. Part ‘A’ was heated strongly while Part ‘B’ was not heated. Dilute hydrochloric
acid was added to both the parts and evolution of gas was seen the cases. How will
you identify the gases evolved?
OR
Distinguish between compounds and mixtures.
38. List the characteristics of cork. How are they formed?
OR
Draw and identify different elements of phloem.
39. Enlist the criteria for the selection of crops for mixed cropping.

------------------x----------------x-------------------
CLASS - IX
MARKING SCHEME
SCIENCE
(2022-23)

SECTION - A
1. 1
Ans. (c)
2. Ans. (a) 1

3. 1
Ans. (a)
4. Ans. (c) 1

5. Ans. (b) 1

6. Ans. (a) 1

7. Ans. (d) 1

8. Ans. (b) 1

9.
Ans. (b) 1

10. Ans. (C) 1

11. Ans. (c) 1

12. Ans. (a) 1

13. Ans. (c) 1

14. Ans. (d) 1

15. Ans. (b) 1

16 Ans. (d) 1
17. Ans. (d) 1

18. Ans. (d) 1

Ans. (c)
19. 1

Ans. (a)
20. 1

SECTION – B

In gases, the particles move randomly at high speed and they collide with each
other and with the walls of the container.
21. 2
OR
Water vapour

22. Due to the presence of cell wall, the exchange of materials between the plant 2
cells is not possible.

23. (i) 8 2
(ii) 4

Anions Cations
(a) CH3COO- Na+
(b) Cl- Na+
(c) H2 -It is a covalent compound
24. (d) NO3- NH4+ 2
OR
Nucleus, also known as the brain of the cell, controls most of the activities of
the cell because it contains DNA.

25. The charge would be +1. 2

26. -2. 2

SECTION – C

The amount of heat that is required to change 1 kg of solid into liquid at


atmospheric pressure without any change in temperature at its melting point, is
27. 3
known as latent heat of fusion. The latent heat of fusion of ice in SI unit is
3.35x105J/kg.
Asters are two star-like fibrous structures radiating from the centrosome. They
help in locating spindle apparatus and triggers cleavage of cytoplasm.
28. Spindle fibres are formed from microtubules during cell division. They pull the 3
chromosomes apart and bring them at the poles. These are broader in the
middle and narrower at the poles.

(a) Instantaneous velocity is the velocity of a body at any particular instant


during its motion.
(b) If the velocity of a body decreases withy time, then its final velocity is
less than the intial velocity and thus its acceleration is negative. Negative
acceleration is called retardation or deceleration.
29. 3
OR

Nervous tissue is made up of neurons that receive and conduct impulses.


Neurons are highly specialized for being stimulated and then transmitting the
stimulus very rapidly from one place to another within the body.

As plants grow older, the outer protective tissue undergoes certain changes. A
30. layer of secondary meristem develops which is called as phellogen. It is also 3
known as cork cambium. It replaces epidermis of stem and roots.

The acceleration due to gravity is more at the poles than at the equator. The
time taken for a body is less if the acceleration due to gravity is more when the
31. 3
initial velocities and the distance travelled are the same. So, when dropped
from the same height a body reaches the ground quicker at the poles than at
the equator.

(a) When a body falls, its potential energy gradually gets converted into
kinetic energy. On reaching the ground, the whole of the potential energy
of the body gets converted into kinetic energy.
(b) The commercial unit of energy is kilowatt hour [kWh].
1 kWh is the energy used in one hour at the rate of 1000 Js-1.

32. OR 3

The capacity of a body to do work is called energy possessed by the body. It is


a scalar quantity and is measured in joule (J).
Two uses of kinetic energy are :
(i) The kinetic energy of air is used to run windmills.
(ii) The kinetic energy of the running water is used to generate electricity.
Longitudinal waves : The waves in which the particles of medium oscillate to
and from their mean position in the direction of propagation of waves are called
longitudinal waves. Give examples.
33. 3
Transverse waves : A wave in which the particles of the medium vibrate up
and down at right angles to the direction in which the wave is moving. Give
examples.

SECTION – D

Mitosis Meiosis

1. It is an equational division. 1. It is a reductional division.

2. Two daughter cells are formed. 2. Four daughter cells are formed.

3. Daughter cells have same number of 3. Daughter cells have half the number
chromosomes as the parent cell. of chromosomes as the parent cell.

4. It takes place in four stages. 4. It takes place in two stages of


divisions.
5. It helps in growth and repair of
injured tissues. 5. It is responsible for production of
gametes.

OR

34. 4

Functions of mitochondria :
The mitochondria are the main sites for cellular respiration, the process in
which the cell converts sugars and oxygen into ATP. ATP is used by various
bodies as a source of energy to perform functions.
(i) Volume of the sphere = 4/3πr3
Let R be the radius of the atom and r be that of the nucleus.

R = 105r

Volume of the atom = 4/3πR3 = 4/3π(105r)3

= 4/3πr3 x 1015

Volume of the nucleus = 4/3πr3

4/3 x 1015 x πr3


Ratio of the size of atom to that of nucleus = ----------------------------- =
1015

4/3πr3

35. (ii) If the atom is represented by the planet Earth ( Re = 6.4 x 106 m ), then 4
the radius of the nucleus would be

rn = Re /105

rn = 6.4 x 106 m = 6.4 x 10 m = 64 m.


-------------------
105

OR

The main function of Golgi bodies is to release proteins or enzymes by vesicles.


No other organelle has this property. Therefore, these are largely present in
secreting cells.

Differences between g and G

36. Universal gravitational constant 4


Acceleration due to gravity (g)
(G)
1. Acceleration due to gravity is the 1. Gravitational constant is numerically
acceleration acquired by a body due equal to the force of attraction
to the earth’s gravitational pull on it. between two masses of 1 kg that are
separated by a distance of 1 m.
2. G is vector quantity. 2. G is a scalar quantity.
3. It is different at different places on 3. The ‘G’ is a universal constant, i.e.,
the surface of the earth. Its value its value is the same everywhere in
also varies from one celestial body to the universe.
another.

OR

According to Newton’s third law of motion, action and reaction are equal and
opposite. It means that the force on the apple due to earth’s attraction is equal
to that on the earth due to apple’s attraction. But we know, acceleration ∞ 1/m.

SECTION – E

Part A
Heat
Fe(s) + S(s) ----------------------à FeS(s)

FeS(s) + 2HCl(aq) ---------------à FeCl2(aq) + H2S(g)

Part B

Fe(s) + S(s) ---------------------à Mixture of iron filings and sulphur

37. When dilute HCl is added to it

Fe(s) + S(s) + 2HCl(aq) --------à FeCl2(aq) + H2(g)

Sulphur remains unreacted.

H2S gas formed has a foul smell and on passing through lead acetate solution,
it turns the solution black. Hydrogen gas burns with a pop sound.

OR 5

Compounds Mixtures
1. Compounds are formed as result of 1. Mixtures are formed by simply mixing
chemical reactions between two or two or more constituents. There are
more elements or compounds. no chemical reactions between the
2. The components of a compound are constituents.
always present in a definite ration by 2. The components of a mixture may be
mass. present in any ration.
3. The properties of a compound are 3. The properties of a mixture are same
entirely different from its as those of its constituents.
constituents. 4. Mixtures are usually heterogeneous
4. Compounds are always (except in solution).
homogeneous in nature. 5. Heat, light or electrical energy may
5. Compound formation is not be evolved or absorbed during
accompanied by absorption or the formation of a mixture.
evolution of light, heat or electrical 6. Melting and boiling points of a
energy. mixture are usually not sharp and
6. Melting and boiling points of a fixed.
compound are usually sharp and 7. The components of a mixture can be
fixed. easily separated by physical
7. The constituents of a compound methods.
cannot be separated by physical or
mechanical means. They can,
however, be separated by chemical
methods.

The characteristics of cork are as follows :

(a) Cells of cork are dead at maturity.


(b) These cells are compactly arranged.
(c) Cells do not possess intercellular spaces.
(d) Cells possess a chemical substance ‘suberin’ in their walls.
(e) There are several thick layers.

38. As plants grow older, a strip of secondary meristem replaces the


epidermis of the stem. Cells on the outside are cut off from this layer.
This forms the several-layer thick cork or the bark of the tree.

OR
5
Sieve tubes, companion cells, phloem fibres and phloem parenchyma are the
different elements of phloem.
Mixed cropping is employed to minimum risk and as an insurance against crop
failure due to abnormal weather conditions. The main criteria’s for selection of
the crops for mixed cropping are as follows :

(a) Duration of Crops : One of the crops should be a long duration and
other should be a short duration crop. 5
(b) Growth Habit : One of the crops should be growing tall and the other
39. should be growing short. The component crops should have different
canopy (i.e., the structure of leaves, stem and flowers found above the
ground).
(c) Nutrient Demand : One of the component crops should require lesser
nutrients than the other crop.
(d) Root Pattern : One of the crop should be deep-rooted while the other
should be shallow rooted.
(e) Water Requirement : One of the component crops should require
lesser water than the other.
Composition of Board Examination 2022-23; Class IX

Blue Print as per CBSE Sample Paper

Unit / Chapter 01 A-R 02 03 05 04 Sub- Total


Name Mark Questio Marks Marks Marks Marks Total Unit
Questio ns Questio Questi Questi Questi Chapte wise
ns ns ons ons ons r wise
[Asserti
MCQ on- Source
Reasoni based
ng] / Case
based

No. of No. of No. of No. of No. of No. of No. of


questio questio questio questio questio questio questi
ns ns ns ns ns ns ons
[Marks [Marks [Marks [Marks [Marks [Marks [Marks
allotted allotted allotted allotte allotte allotte allotte
] ] ] d] d] d] d]

Unit-I Matter - 13(25)


Its Nature and
Behaviour

Matter in Our 1(1) 1(2)OR 1(3) 3(6)


Surroundings

Is Matter 1(1) 1(5)OR 2(6)


Around Us Pure
(Excluding
Separating the
components of
a mixture)
Atoms and 1(1) 1(1) 2(4) 4(6)
Molecules

Structure of 3(3) 1(4)OR 4(7)


Atom

Unit-II 10(22)
Organisation in
the Living
World

Cell: Basic unit 2(2) 1(2)OR 1(3) 1(4)OR 5(11)


of Life

Tissues, 2(2) 1(1) 1(3)OR 1(5)OR 5(11)


Organs, Organ
system,
Organism

Unit-III Motion, 14(27)


Force and Work

Motion 1(1) 1(3) 2(4)

Force and 1(1) 2(4) 3(5)


Newton’s laws

Gravitation 1(1) 1(4)OR 2(5)


Floatation 1(1) 1(3) 2(4)

Work, Energy & 1(1) 1(1) 1(3)OR 3(5)


Power

Sound 1(1) 1(3) 2(4)

Unit IV Food 2(6)


Production

(Improvement 1(1) 1(5) 2(6)


in Food
Resources)

Sub-Total 16(16) 4(4) 6 (12) 7(21) 3(15) 3(12) 39 (80)

“OR” means that there is an alternate question in that particular question.

# Please refer to Sample paper of Class X to get a better view of the question paper of class
IX.

S.N. Particulars Percentage Worked out


marks
1 Competency Based Questions 40 % 32

(MCQs, Source Based Integrated


Questions, Case Based Questions
or other types

2. Objective types Questions 20 % 16

3. SA / LA Questions 40 % 32

Sub-Total 100 % 80

Internal Assessment 20

Grand Total 100


KENDRIYA VIDYALAYA SANGATHAN
RANCHI REGION
SAMPLE PAPER - 1
Session 2022-23
Subject- Science. Class - IX Time:90 MIN Max Marks:80
General Instructions:
(i)The question paper comprises four sections A, B, C, and D. There are 39 questions in the
question paper. All Questions are compulsory.
(ii)Section–A – Question no. 1 to 20 are Multiple choice questions (MCQs) and Assertion
reasoning type questions.
iii)Section–B – Question no. 21 to 26 are short answer type questions (SA), carrying 2 marks
each. Answers to these Questions should be in the range of 20 to 40 words.
(iv)Section–C – Question no. 27 to 33 are short answer type questions (SA), carrying 3 marks
each. Answers to these Questions should be in the range of 50 to 70 words.
(v)Section–D- Questions no. 34 to 36 are long answer type questions, (LA)
carrying 5 marks each. Answers to these Questions should be in the range of 70 to 80
words.
(vi) Section - E - Question no. 37 to 39 are Source- based or Case based questions.
SECTION - A
MCQs
1. Which of the following correctly represent the electronic distribution in the Mg atom?
(A)3,8,1. (B)2,8,2. (C)1,8,3. (D)8,2,2
2. Girth of stem increases due to :
(A) Apical meristem. (B) Lateral meristem.
(C) Vertical meristem (D) Intercalary meristem
3. A boy is whirling a stone tied with a string in a horizontal circular path. If the string
breaks, the stone :
(A) Will continue to move in the circular path.
(B) Will move along a straight line towards the centre of the circular path
(C) Will move along a straight line tangential to the circular path
(D) Will move along a straight line perpendicular to the circular path away from the boy.
4. Which of the following statements is not true about an atom ?
(A) Atoms are not able to exist independently.
(B) Atoms are the basic unit from which molecules and ions are formed.
(C ) Atoms are always neutral in nature
(D) Atoms aggregate in large numbers to form the matter that we can see, feel or touch.
5. Earthquake produces which kind of sound before the main shock wave begins
(A) Ultrasound. (B) Infrasound (C) Audible sound. (D) None of these.
6. Which condition out of the following will increase the evaporation of water?
(A) Increase in temperature of water (B) Decrease in temperature of water
(C) Less exposed surface area of water (D) Adding common salt to water
7. Which of the following are true for an element ?
(A) Atomic number = Number of proton + Number of electrons.
(B) Mass number = Number of protons + Number of neutrons.
(C) Atomic number = Number of protons = Number of neutrons.
(D) Atomic number = Number of protons = Number of electrons.
8. A goalkeeper in a game of football pulls his hands backwards after holding the ball shot at
the goal. This enables the goalkeeper to:
(A) Exert larger force on the ball. (B) Reduce the force exerted by the ball on
hands.
(C) Increase the rate of change of momentum.
(D) Decrease the rate of change of momentum.
9. Which is not a function of epidermis?
(A) Protection from adverse condition (B) Preventing entry of pathogen
(C) Conduction of water. (D) Transpiration.
10. Tincture of iodine has antiseptic properties. This solution is made by dissolving:
(A) Iodine in potassium iodide (B) Iodine in Vaseline
(C) Iodine in water (D) Iodine in alcohol
11. A ship rises when it enters from sea to river because:
(A) The force of buoyancy decreases (B) The force of buoyancy increases
(C) The volume of ship increases (D) The mass of ship decreases
12. The atom with 3 protons and 4 neutrons will have a valency of:
(A) 3. (B) 7. (C) 1. (D) 4
13. Living cells where discovered by:
(A) Robert Hooke. (B) Purkinje (C) Leeuwenhoek. (D) Robert Brown
14. A flying aeroplane possesses:
(A) Only potential energy (B) Only kinetic energy
(C) Both potential and kinetic energy (D) Neither potential nor kinetic energy
15. Amoeba acquire its food through a process, termed:
(A) Exocytosis. (B) Endocytosis. (C) Plasmolysis. (D) Both exocytosis and endocytosis
16. The speedometer of a car measures:
(A) Average speed. (B) Instantaneous speed (C) Acceleration. (D) None of these

Direction: In the following questions a statement of assertion(A) is followed by a


statement of reason(R). Mark the correct choice as:
(A) Both assertion (A) and reason (R ) are true and reason (R ) is the correct
explanation of assertion (A).
(B) Both assertion (A) and reason (R ) are true but reason(R ) is not the correct
explanation of assertion(A).
(C) Assertion (A) is true but reason (R ) is false.
(D) Assertion(A) is false but reason (R ) is true.
17. Assertion: Guard cells are specialized epidermal cells.
Reason: Stomata are found in the epidermis of leaves.
18.Assertion: Work done by the force can be positive or negative.
Reason: Force is taken as positive when it acts opposite to the direction of displacement.
19.Assertion: Carbonates are polyatomic ions.
Reason: The carbonate ion consists of one carbon atom and 3 oxygen atoms and carries an
overall charge of 2–.
20.Assertion: Fodder crops like berseem , oats etc. are also grown along with cereals and
pulses.
Reason: Fodder crop is food for the livestock.
SECTION - B

21. What are polyatomic ions. List two examples.


22. There are three solids made up of Aluminium, Steel and Wood, of the same shape and
same volume. Which of them would have the highest inertia?
23. Why do we see water droplets on the outer surface of a glass containing ice cold water?
OR
Why do the gases exert more pressure on the walls of the container than the solids?
24. Write the name of different plant parts in which chromoplast, chloroplast and leucoplast
are present.
OR
What is cell division? Give the types of cell division.
25. Derive Newton's first law of motion from the mathematical expression of the second law
of motion.
26. Write the chemical formula of:
(i) Zinc phosphate. (ii) Lead carbonate

SECTION - C
27. Define 1 joule of work. Calculate the work done in lifting a box weighing 150 kg through
a vertical height of 7 meters ( Take g=10 ms–2 )
OR
State the law of conservation of energy. The potential energy of a free falling object
decreases progressively. Does this violate the law of conservation of energy? Why?
28. Draw a neat diagram of the plant cell and label any three parts which differentiate it
from the animal cell.
29. How does evaporation differ from boiling?
30. Label the following and give one function of each part labelled (i), (ii) and (iii).

OR
Write three distinguishing features between cells of meristematic and permanent plant
tissues.
31. Explain the following type of motion with one example of each.
(i) acceleration is positive.
(ii) acceleration is negative
(iii) acceleration is zero.
32. Write three differences between transverse waves and longitudinal waves.
33. (i) Why is the tip of the pin sharp but the head is broad?
(ii) How does snow shoes stop you from sinking into snow?
SECTION - D
34. Enlist the criteria for the selection of crops for mixed cropping.
35. Differentiate between a true solution and a colloid.
OR
Distinguish between compounds and mixtures.
36. What are the differences between striated, unstriated and cardiac muscles?
OR
Draw the labelled diagram of neurons. Describe the structure, function and location of
nervous tissue.
SECTION - E
37. Read the following passage and answer the following questions.(Any four)
All living Organisms are made up of cells and these cells perform all the functions essential
for the survival of the Organism, e.g ., respiration, digestion, excretion etc. In Unicellular
organisms, a single cell carries out all these functions and in multicellular organisms
different group of cells carry out different functions.

1.Cell was discovered by:


(A) Robert Brown (B) Robert Hooke (C) A.V. Leeuwenhoek (D) Charles Darwin
2.The structural and functional unit of life is-
(A) Cell (B) Tissue (C) Mitochondria (D) Chloroplast.
3. Select the Prokaryotic cell-
(A) Amoeba (B) Bacteria (C) Paramecium (D) Hydra
4.The Cell which do not have a fixed shape-
(A) Red Blood Cells (B) Paramecium (C) Amoeba (D) All of these
OR
Cell division is the process by which new cells are formed. It is of two main types: mitosis
and meiosis. Meiosis is only confined to specific cells called meiocytes of reproductive
organs or tissues in animals, plants, various Protista and fungi. It takes place at a
particular time. These specific cells divide to form gametes. Mitosis, on the other hand,
occurs in all kinds of cells and may continue throughout life.

1. How many daughter cells are formed in meiosis?


(A)1 (B) 2 (C ) 3 (D)4
2. Which of the following divisions is also termed as somatic division?
(A) Mitosis (B) Meiosis (C ) Both A and B (D) None of these.
3. Which is called reduction cell division?
(A) Mitosis (B) Meiosis (C ) Both A and B (D) None of these.
4. Mitosis has how many distinct phases ?
(A)2 (B) 3 (C ) 4 (D)5
5. In which cell division number of chromosome become halved?
(A)Mitosis (B) Meiosis (C ) Both A and B (D) None of these.

38. Read the following passage and answer the following questions.(Any four)

In order to overcome the objections raised against Rutherford's model of the atom, Neil
Bohr put forward the following postulates about the model of an atom.

1.Atoms are made up of ____ , ____ and ____.


(A)atomic number, mass number, nucleus (B)electrons, nucleus, shells
(C)electrons, protons and neutrons (D)none of these
2. Who amended Rutherford's short comings?
(A) Dalton (B) James Chadwick (C) Neils' Bohr (D) J.J. Thomson
3. The number of electrons that K-shell and L-shell can accommodate:
(A) 8, 2 (B) 2, 8 (C)8, 8 (D) 2, 2
4.Atomic mass is the sum of :
(A)number of electrons and protons (B)number of protons and neutrons
(C)number of electrons and neutrons (D)number of electrons, protons and
neutrons
5.The Law states that can ________ neither be created nor be destroyed in a chemical
reaction.
(A) Atomic number (B) Electron (C) Proton (D)Mass
OR
The maximum number of the electrons which are permitted to be assigned to an energy
shell an atom is called the electron capacity of that shell. The distribution of electrons in
different orbits or shell is governed by a scheme known as the Bohr-Bury scheme.
According to this scheme :
(i) The maximum number of the electrons that can be present in any shell is given by the
formula 2n2 where n is the number of energy levels.
(ii) The maximum number of electrons that can be accommodated in the outermost shell
is 8.Electrons are filled in the shells in a stepwise manner in increasing order of energy of
the energy shell.

1. What is the maximum electrons capacity of N shell?


(A) 24 (B) 8 (C) 18 (D) 32
2. Identify the element with the configuration
K-2, L-8, M-3.
(A) Aluminium (B) Magnesium (C) Sodium (D) Beryllium
3. Which of the following configuration represent sodium?
(A) 2, 8, 4 (B) 2, 8, 5 (C) 2, 3 (D) 2, 8, 1
4. The atomic number of Neon is 10. It's electronic configuration is:
(A) 2, 8 (B) 8, 2 (C )2,7, 1 (D) 2, 6, 2
5. Which of the following do not represent Bohr’s model of an atom correctly?

(A) (i) and (ii) (B) (ii) and (iii) (C) (ii) and (iv) (D) (i) and (iv)
39. Read the following paragraph and choose the correct options to answer the questions
given below: (Any four)

Seema was standing on a terrace of a building. She dropped a coin and a piece of paper
simultaneously from the same height.

1.Which of the two will touch the ground first ?


(A)In air the coin will touch the ground first.
(B)The piece of paper will touch the ground first.
(C)Both will touch the ground simultaneously.
(D)None of them will touch the ground.
2.What will happen if they are dropped in vacuum?
(A)The piece of paper will touch the ground first.
(B)In vacuum both will touch the ground together.
(C)The coin will touch the ground first.
(D)None of them will touch the ground.
3. Give reasons for your answer.
(A)In case of air the resistance provided by the air is less for paper than coin.
(B)No resistance is provided by air.
(C)In case of air the resistance provided by the air is more for paper than coin.
(D)Air provides equal resistance for both the objects.
4. What is meant by free fall?
(A)This is the fall of the body when many forces are acting on it from all the sides.
(B)This is the fall of the body from one medium to another.
(C)In this the body is prohibited from falling down.
(D)The falling of a body from a height towards Earth under the gravitational force of Earth
(with no other force acting on it) is called free fall.
5. Which of the following statements is correct?
(A)In case of a free fall, there is no change in the direction of motion of the objects.
(B)In case of a free fall, there is always a change in the direction of motion of the objects.
(C)In case of a free fall, the direction of motion reverts.
(D)None of the above statements is correct.
OR
There is no atmosphere on the moon. This is because gas molecules need a certain
amount of force of attraction to be retained on a heavenly body. The force of attraction of
the moon is less than the required force, hence no atmosphere can exist.

1. The value of g on moon is ........ times that of earth.


(A) 1/3 (B) 1/4 (C) 1/5 (D) 1/6
2. Mass of the moon is .......... that of earth.
(A) more than (B) less than (C) equal to (D) can’t say
3. If the weight of an object is 60 kg f on earth then, its weight on the moon is .......
(A) 10 kg f (B) 20 kg f (C) 30 kg f (D) 40 kg f

4. An astronaut in the orbit in a spacecraft feels weightlessness:


(A) due to the absence of gravity inside.
(B) due to the fact that spacecraft have no energy.
(C) because acceleration in the orbit is equal to acceleration of gravity outside.
(D) there is no gravity outside.
5. The mass of a body is measured to be 12 kg on the earth. If it is taken to the moon, its
mass will be
(A) 12 kg (B) 6 kg (C) 2 kg (D) 72 kg.

X-X-X-X-X-X-X-X-X-X-X-X-X-X-X-X-X-X-X
Marking Scheme
1. Option (D) is correct. 1

2. Option (B) is correct. 1

3 Option (C) is correct. 1

4 Option (D) is correct. 1

5 Option (B) is correct. 1

6. Option (A) is correct. 1

7. Option (D) is correct. 1

8. Option (B) is correct. 1

9. Option (C) is correct. 1

10. Option (D) is correct. 1

11. Option (A) is correct. 1

12. Option (C) is correct. 1

13. Option (C) is correct. 1

14. Option (C) is correct. 1

15. Option (B) is correct. 1

16. Option (B) is correct. 1

17. Option (B) is correct. 1

18. Option (C) is correct. 1

19. Option (A) is correct. 1

20. Option (A) is correct. 1

21. A group of atoms carrying a charge is known as polyatomic ion. e.g. PO43– , 1+1
SO42– ,NH4+

22. Steel has the highest inertia. As the mass is a measure of inertia, the ball of 2
the same shape and same size, having more mass than other balls, will have
the highest inertia. Since Steel has greatest density and greatest mass
therefore it has highest inertia.

23. The water vapour present in the air comes in contact with the cold surface of 2
the glass, loses its energy and gets converted into droplets of water.
OR
In gases, the particles move randomly at high speed and they collide with 2
each other and with the walls of the container.

24. Chromoplast are present in flowers, fruits or coloured part of the plant. 1+1
Chloroplasts are present in leaves and stems of the plant.
Leucoplast are present anywhere in plants as they are colourless and store
food.
OR
Cell division is a process of formation of two or more daughter cells of its own 1+1
type from the mother cell. These are similar in structure and function.
Cell division occurs in 3 ways:
(i) Amitosis. (ii) Mitosis. (iii) Meiosis

25. Newton's first law states that a body stays at rest if it is at rest and moves with 1
a constant velocity until a net non- zero force is applied on it. Newton's
second law states that the net force applied on the body is equal to the rate
of change of its momentum.
F=ma , 1
or, F=m (v - u) / t
or, Ft=mv - mu
That is when F=0,v = u for whatever time t is taken. This means that the object
will continue moving with uniform velocity, u throughout the time, t . If u is
zero then v will also be zero, i.e., the object will remain at rest.

26. (i) Zn3(PO4)2 (ii) PbCO3 1+1

27. One joule is the amount of work done on an object when a force of 1 N 1
displaces it by 1 m along the line of action of force.
P.E. = mgh 1
m=150kg, g= 10ms-2,h= 7m
P.E.= 150 x 10 x 7 = 10500 J 1

OR
Energy can neither be created nor be destroyed, but can be converted from 1
one form to another.
No, potential energy is converted to kinetic energy. 2
28. 3

29. 1+1+1
Evaporation Boiling

1. Evaporation takes place at all 1. Boiling takes place only at the


temperatures. boiling point of the liquid.

2. Temperature changes during 2. The temperature does not change


evaporation. during boiling.

3. It is a very slow process. 3. It is a fast process.

4. Evaporation takes place only at 4. Boiling takes place in the entire


the surface of the liquid. body of the liquid.

30. (i) Lateral meristem: helps in growth and development of plant's shoot girth. 1+1+1
(ii) Apical meristem : helps in growth of the stem and the root.
(iii) Intercalary meristem: helps in increasing internodes of plants like
sugarcane.
OR

Meristematic tissue Permanent tissue

Cells possess dividing ability. Cells generally do not divide.


1+1+1
Cells are living. Cells can be living or dead.

Main function is to bring about Perform various types of functions.


growth.

31. (i) The motion is accelerated motion, e.g., a car moving on a road with 1
increasing velocity.
(ii) The motion is retarded motion, e.g. Brakes applied to a moving car. 1
(iii) The motion is uniform motion, e.g., a car moving with a constant speed
along a road. 1

32.
Transverse waves Longitudinal waves

The particles of the medium The particles of the medium move in


oscillate up and down about their parallel to the direction of 1
mean position. propagation of the disturbance.

They propagate as crests or They propagate as compressions and 1


troughs. rarefactions.

The propagation of waves is The propagation of these waves is 1


possible in solid or surface of liquid possible in solids, liquids and gases.
but not in gases. Ex. - Sound wave
Ex.- Light wave

33. (i) Tip has less area, so there is more pressure, hence it pierces easily. 1½
Head is broad so that the area of contact is more, so the pressure can be
easily exerted.
(ii) Snowshoes have a broad base, so there is more area of contact. Therefore, 1½
there is less pressure on the snow and the person does not sink.

34. Mixed cropping is employed to minimize risk and as an insurance against crop
failure due to abnormal weather conditions. The main criterias for selection of
the crops for mixed cropping are as follows:
(a)Duration of Crops: One of the crops should be a long duration and the 1
other should be a short duration crop.
(b)Growth Habit: One of the crops should be growing tall and the other 1
should be growing short. The component crops should have different canopy
(i.e ., the structure of leaves, stem and flowers found above the ground).
(c )Nutrient Demand: One of the component crops should require less 1
nutrients than the other crop.
(d)Root Pattern: One of the crops should be deep-rooted while the other 1
should be shallow rooted.
(e)Water Requirement: One of the component crops should require less 1
water than the other.

35.
True solution Colloid

A true solution is a homogeneous A colloidal solution is a 1


mixture of two or more substances. heterogeneous mixture of two
substances.
1
The size of the particles is less than The range of particle size is between
one nanometre. one nanometre to 1000 nanometre.
1
It is always transparent. It is translucent.
The particles cannot be seen even The particles of a colloidal solution 1
with a microscope. can be seen with a microscope.
1
It does not show the Tyndall effect. It shows the Tyndall effect.

OR

Compounds Mixture
1
They have a fixed composition. Does not have a fixed composition.

Elements are chemically combined in Mixing of elements or compounds


1
a fixed ratio. form mixtures.

It does not show properties of its It shows properties of its


components. components. 1

Their components can be separated Components can be easily separated


by certain chemical methods. by physical methods. 1

They have a fixed melting point. They do not have a fixed melting
point. 1
Heat/light/electricity is absorbed or Little or no heat is involved in their
evolved in their formation. formation.

e.g. NaCl, KCI, CuSO4, ZnSO4 are e.g. Mixture of iron filings and
compounds. sulphur, sand + iodine, sea water,
brass.

36.
Striated muscles Unstriated (Smooth) Cardiac muscles
muscles

They are found in limbs, They are present in the They are present in the
tongue, pharynx etc. wall of visceral organs. heart.

Long, cylindrical with Short, spindle shaped Short, branched and


blunt ends. with pointed ends. cylindrical with flat
ends. 1+1+1
+1+1
Multinucleate, nuclei Uninucleate, nucleus in One or two nuclei in
peripheral. the centre. the centre.

They are voluntary in They are involuntary in They are involuntary in


action. action. action.

Dark and light bands No bands present. Bands present.


are present.

OR

Structure: The nervous tissue is made up of neurons which consists of a cell


body with a nucleus and cytoplasm, from which long thin hair-like parts arise.
Function: On stimulation, the nerve cells transmit the stimulus very rapidly 1
from one place to another within the body.
Location: Nervous tissues are located in the brain, spinal cord and nerves. 1

37. 1-B, 2-A ,3-B ,4-C 1+1+1


OR +1
1-(D), 2- (A) ,3- (B) ,4- (D) 5- (B)

38. 1-C, 2-C ,3- B ,4- B ,5- D 1+1+1


OR +1
1-D, 2-A ,3- D, 4-A , 5-C

39. 1-A, 2-B, 3-C , 4-D ,5- A 1+1+1


OR +1
1- D ,2- B,3- A, 4-D ,5-A
KENDRIYA VIDYALAYA SANGATHAN: RANCHI REGION
CLASS IX
SUBJECT: SCIENCE
SAMPLE PAPER BLUE PRINT
2022-2023
Max. Marks: 80 Time allotted: 3
hours

Unit / 01 Mark A-R 02 Marks 03 Marks 05 Marks 04 Marks Sub- Total


Chapter Question Questions Question Question Question Question Total Unit
Name s MCQ [Assertion- s s s s Source Chapter wise
Reasoning] based / wise
Case
based
No. of No. of No. of No. of No. of No. of No. of
questions questions questions questions questions questions question
[Marks [Marks [Marks [Marks [Marks [Marks s [Marks
allotted] allotted] allotted] allotted] allotted] allotted] allotted]
Unit-I Matter 13(25)
- Its Nature
and Behavior
Matter in Our 1(1) 1(2)OR 1(3) 3(6)
Surroundings

Is Matter 1(1) 1(5) 2(6)


Around Us
Pure
(Excluding
Separating
the
components
of a mixture)

Atoms and 1(1) 1(1) 2(4) 4(6)


Molecules

Structure of 3(3) 1(4) 4(7)


Atom
Unit-II 10(22)
Organization
in the Living
World
Cell: Basic 2(2) 1(2)OR 1(3) 1(4) 5(11)
unit of Life
Tissues, 2(2) 1(1) 1(3)OR 1(5)OR 5(11)
Organs,
Organ
system,
Organism

Unit-III 14(27)
Motion,
Force and
Work
Motion 1(1) 1(4)OR 2(5)

Force and 1(1) 2(4) 3(5)


Newton’s
laws
Gravitation 1(1) 1(3) 2(4)

Work, Energy 1(1) 1(1) 1(3)OR 3(5)


& Power
Sound 1(1) 1(3) 2(4)

Unit IV Food 2(6)


Production
(Improvemen 1(1) 1(5) 2(6)
t in Food
Resources)

Sub-Total 16(16) 4(4) 6 (12) 7(21) 3(15) 3(12) 39 (80)


KENDRIYA VIDYALAYA SANGATHAN: RANCHI REGION
CLASS IX
SUBJECT: SCIENCE
SAMPLE PAPER
2022-2023
Max. Marks: 80 Time allotted: 3
hours

General Instructions:
(i) The question paper comprises four sections A, B, C and D. There are 18 questions
in the question paper. All questions are compulsory.
(ii) Section A - question no. 1 to 20 are MCQ / short answer type questions, carrying
1 mark each. Answers to these questions should be in the range of 30 to 50
words.
(iii) Section B - question no. 21 to 26 are short answer type questions, carrying 2
marks each. Answers to these questions should be in the range of 50 to 80 words.
(iv) Section C - question no. - 27 to 33 are case study based questions carrying 3 marks
each. Answers to these questions should be in the range of 80 to 120 words.
(v) Section D - question no. - 34 to 36 are long answer based questions carrying 5
marks each. Answers to these questions should be in the range of 120 to 150
words.
(vi) Section E - question no. - 37 to 39 are case study based questions carrying 4 marks
each. Answers to these questions should be in the range of 120 to 150 words or
one word if they have sub questions as MCQ.
(vii) There are options provided for some questions. Answer any one only.

Section A

1. Which of the following is matter? 1


(a) Chair
(b) Hate
(c) Smell
(d) Love

2. Which of the following will show “Tyndall effect”? 1


(a) Salt solution
(b) Milk
(c) Copper sulphate solution
(d) None of the above

3. Formulae for Sodium Oxide is 1


(a) Na2O
(b) Na2S
(c) Na2O2
(d) None of the above

4. Rutherford’s ‘alpha (α) particles scattering experiment’ resulted in the discovery


of? 1
(a) Electron
(b) Proton
(c) Nucleus in the atom
(d) Atomic mass

5. An atom with 3 protons and 4 neutrons will have a valency of 1


(a) 3
(b) 7
(c) 1
(d) 4

6. The first model of an atom was given by 1


(a) N. Bohr
(b) E. Goldstein
(c) Rutherford
(d) J.J. Thomson

7. Plasmolysis in a plant cell is defined as 1


(a) breakdown (lysis) of plasma membrane in hypotonic medium
(b) shrinkage of cytoplasm in hypertonic medium
(c) shrinkage of nucleoplasm
(d) none of them

8. Chromosomes are made up of 1


(a) DNA
(b) Protein
(c) DNA and protein
(d) RNA

9. A person met with an accident in which two long bones of the hand were
dislocated. Which among the following may be the possible reason? 1
(a) Tendon break
(b) Break of skeletal muscle
(c) Ligament break
(d) Areolar tissue break

10. Which is not a function of epidermis? 1


(a) Protection from adverse condition
(b) Gaseous exchange
(c) Conduction of water
(d) Transpiration

11. A particle is moving in a circular path of radius r. The displacement after half a
circle would be: 1
(a) Zero
(b) πr
(c) 2r
(d) 2πr

12. The gravitational force between two objects is F. If masses of both objects are
halved without changing the distance between them, then the gravitational force
would become 1
(a) F/4
(b) F/2
(c) F
(d) 2F

13. According to the third law of motion, action and reaction 1


(a) always act on the same body
(b) always act on different bodies in opposite directions
(c) have same magnitude and directions
(d) act on either body at normal to each other

14. When a body falls freely towards the earth, then its total energy 1
(a) increases
(b) decreases
(c) remains constant
(d) first increases and then decreases

15. In SONAR, we use 1


(a) ultrasonic waves
(b) infrasonic waves
(c) radio waves
(d) audible sound waves

16. Which one is not a source of carbohydrate? 1


(a) Rice
(b) Millets
(c) Sorghum
(d) Gram

17. How many atoms are present in a H2S molecule ? 1

18. Name the tissue responsible for movement in our body. 1


19. Define 1 J of work. 1

20. What are macro-nutrients and why are they called macro-nutrients? 1

Section B

21. What is the physical state of water at: 2


(a) 250°C
(b) 100°C

OR
Convert the following temperatures to the Celsius scale.
(a) 293 K
(b) 470 K

22. What are polyatomic ions? Give examples. 2

23. Give the names of the elements present in the following compounds: 2
(a) Quick lime
(b) Hydrogen bromide
(c) Baking powder
(d) Potassium sulphate.

24. Why is the cell called the structural and Functional unit of life? 2

OR

Which organelle is known as the powerhouse of the cell? Why?

25. Explain why it is difficult for a fireman to hold a hose, which ejects a large amount
of water at a high velocity. 2

26. Two objects of masses 100 g and 200 g are moving along the same line and direction
with velocities of 2 m/s and 1 m/s respectively. They collide and after the collision
the first object moves at a velocity of 1.67 m./s. Determine the velocity of the second
object. 2

Section C

27. Give two reasons to justify 3


(a) water at room temperature is a liquid.
(b) an iron almirah is a solid at room temperature.

28. Make a comparison and write down ways in which plant cells are also different
from animal cells. 3
29. Identify the type of tissue in the following: Skin, bark of tree, bone. 3
OR
Name the following:
(i) Tissue that forms the inner lining of our mouth.
(ii) Tissue that connect muscle to bone in humans.
(iii) Tissue that transports food in plants.

30. Derive the second equation of motion, s= ut+ 12at2 numerically. 3

31. The volume of 50 g of a substance is 20 cm3. If the density of water is 1 gem 3,


will the substance float or sink? 3

32. A freely falling object eventually stops on reaching the ground. What happens to
its kinetic energy? 3

OR
Find the energy in kWh consumed in 10 hours by four devices of power 500 W
each.

33. Does sound follow the same laws of reflection as light does? Explain. 3

Section D

34. Classify the following into elements, compounds and mixtures. 5


(a) Sodium (b) Soil (c) Sugar solution
(d) Silver (e) Calcium carbonate (f) Tin
(g) Silicon (h) Coal (i) Air
(j) Soap (k) Methane (l) Carbon dioxide
(m) Blood

OR

How would you confirm that a colourless liquid given to you is pure water? Which
of the following materials fall in the category of a “pure substance”?
(a) Ice
(b) Milk
(c) Iron
(d) Hydrochloric acid
(e) Calcium oxide
(f) Mercury
(g) Back
(h) Wood
(i) Air
35. Draw a well labelled diagram of Neuron. 5

OR

Diagrammatically show the difference between the three types of muscle fibres.

36. Explain any one method of crop production which ensures high yield. 5

Section E

37. The maximum number of the electrons which are permitted to be assigned to an
energy shell of an atom is called the electron capacity of that shell. The
distribution of electrons in different orbits or shell is governed by a scheme known
as Bohr-Bury scheme. According to this scheme: 4

(1) The maximum number of the electrons that can be present in any shell is given
by the formula 2n2 where, n is the number of energy level.
(2) The maximum number of electrons that can be accommodated in the
outermost shell is 8. Electrons are filled in the shells in a stepwise manner in
increasing order of energy of the energy shell

(i) What is the maximum electrons capacity of N shell?


(a) 24 (b) 8 (c) 18 (d) 32

(ii) Identify the element with the configuration K-2, L-8, M-3.
(a) Aluminium (b) Magnesium
(c) Sodium (d) Beryllium

(iii) Which of the following configurations represent sodium?


(a) 2, 8, 4
(b) 2, 8, 5
(c) 2, 3
(d) 2, 8, 1

38. All living Organisms are made up of cells and these cells perform all the functions
essential for the survival of the Organism e.g. Respiration, digestion, excretion etc.
In Unicellular organisms, a single cell carries out all these functions and in
multicellular organisms different group of cells carry out different functions. Cells
were first discovered by Robert Hooke in 1665. He observed the cells in a cork
slice with the help of a primitive microscope. Leeuwenhoek (1674), with the
improved microscope, discovered the free living cells in pond water for the first
time. It was Robert Brown in 1831 who discovered the nucleus in the cell. Purkinje
in 1839 coined the term ‘protoplasm’ for the fluid substance of the cell. 4

(i) Who discovered the cell?


(a) Robert Hooke
(b) Leeuwenhoek
(c) Robert Brown
(d) T. Schwann

(ii) Who discovered the nucleus in the cell?


(a) Robert Hooke
(b) Leeuwenhoek
(c) Robert Brown
(d) T. Schwann

(iii) Who coined the term ‘Protoplasm’?

(a) Robert Hooke


(b) Leeuwenhoek
(c) Robert Brown
(d) Purkinje

(iv) What is protoplasm?

(a) Unit of life


(b) Cell organelle
(c) Fluid substance of the cell.
(d) Cytoplasm

39. Distance and displacement are two quantities that seem to mean the same but
are different with different meanings and definitions. Distance is the measure of
“how much distance an object has covered during its motion” while displacement
refers to the measure of “how far the object actually from initial place.” using this
data answer following questions. 4

(i) Which of the following relation is always true when object moves in straight
line:

(a) distance is always equal to displacement


(b) distance is always greater than or equal to displacement
(c) distance is always lesser than or equal to displacement
(d) none of the above

(ii) Kapil travels 20 km to North but then come back to South for 40 km to pick up
a friend. What is the total distance travelled by Kapil?

(a) 60km
(b) 80km
(c) 20km
(d) none of the above
(iii) Rahul travels 20 km to East but then come back to West for 10 km. Find
displacement.

(a) 30km
(b) 20km
(c) 10km
(d) none of the above

(iv) Define distance and displacement of particle.

OR

The speed of an object is the distance covered per unit time, and velocity is the
displacement per unit time. To specify the speed of an object, we require only its
magnitude while Velocity is the speed of an object moving in a definite direction.

(i) S.I unit of speed is


(a) m/s
(b) s/m
(c) m/s2
(d) none of these

(ii) Which of the following is true


(a) Speed is scalar
(b) Velocity is vector
(c) Both a and b
(d) None of these

(iii) To specify speed we require


(a) magnitude
(b) direction
(c) both magnitude and direction
(d) none of these

(iv) Define speed and velocity of particles.


KENDRIYA VIDYALAYA SANGATHAN: RANCHI REGION
CLASS IX
SUBJECT: SCIENCE
SAMPLE PAPER MARKING SCHEME
2022-2023
Max. Marks: 80 Time allotted: 3
hours

No. Answer Marks

1 (a) Chair 1
2 (b) Milk 1

3 (a) Na2O 1

4 (c) Nucleus in the atom 1

5 (c) 1 1

6 (d) J.J Thomson 1


7 (b) shrinkage of cytoplasm in hypertonic medium 1
8 (c) DNA and protein 1
9 (c) Ligament break 1
10 (c) Conduction of water 1
11 (c) 2r 1
12 (a) F/4 1
13 (b) always act on different bodies in opposite directions 1
14 (c) remains constant 1
15 (c) radio waves 1
16 (d) Gram 1

17 H2S —> 3 atoms are present 1


18 (Any one) 1. Muscular tissue, 2. Nervous tissue, 1
combination of both the tissues are responsible for
movement in our body.
19 When a force of IN causes a displacement of 1m, in its own 1
direction the work done is said to be one joule.
20 Macro-nutrients are the essential elements which are ½ +½
utilised by plants in large quantities. Many macro-nutrients
are required by the plants for the following functions:

As the constituent of protoplasm


N, P, S are present in proteins
Ca is present in cell wall
Mg is important constituent of chlorophyll
21 (a) 250°C = gas (b) 100°C liquid as well as gas 1+1

OR

(a) 293 K into °C


293 – 273 = 20°C
(b) 470 K into °C 470 – 273 = 197°C
22 The ions which contain more than one atoms (same kind or 1.5+0.5
may be of different kind) and behave as a single unit are
called polyatomic ions e.g., OH–, SO42-, CO32-.
23 (a) Quick lime —> Calcium oxide ½+½+½+½
Elements —> Calcium and oxygen
(b) Hydrogen bromide
Elements —> Hydrogen and bromine
(c) Baking powder —> Sodium hydrogen carbonate
Elements —> Sodium, hydrogen, carbon and oxygen
(d) Potassium sulphate
Elements —> Potassium, sulphur and oxygen
24 A cell is capable of independently carrying out all 2
necessary activities of life. So, they are called basic or
functional unit of life.

OR
Mitochondria is known as the powerhouse of the cell
because it releases the energy required for different
activities of life.

25 The water that is ejected out from the hose in the 1.5 + ½
forward direction comes out with a large momentum
and equal amount of momentum is developed in the
hose in the opposite direction and hence the hose is
pushed backward. It becomes difficult for a fireman
to hold a hose which experiences this large
momentum.
26 2

27 (a) Water at room temperature is a liquid because its 1.5+1.5


freezing point is 0°C and boiling point is 100°C.
(b) An iron almirah is a solid at room temperature
because melting point of iron is higher than room
temperature.
28 3

29 (a) Skin—Striated squamous epithelium 1+1+1


(b) Bark of tree—Cork, protective tissue
(c) Bone—Connective tissue

OR
(i) Squamous epithelium
(ii) Tendons
(iii) Phloem
30 Let at time t=0 3
, body has initial velocity = Vo

At time‘t’, the body has final velocity=V


S=distance travelled in time‘t’

We know, total distance travelled = Average velocity ×


time

!"!#!$% '(%)*!#+,-!"$% '(%)*!#+


Average velocity = .

⇒Average velocity = (Vo+V)/2

Total distance=s=[(Vo+V)/2]×t

⇒2S=(Vo+V)t→(i)

Now from first equation of motion, V=Vo+at→(ii)

Use the value of (V) from (ii) in (i)

2s=(Vo+Vo+at)t

2s=2Vot+12at2

Let,Vo=u

⇒s=ut+1/2at2
31 2+1

32 As the object hits the hard ground, its kinetic energy gets 1+1+1
converted into
(i) heat energy (the object and the ground become
slightly warm)
(ii) sound energy (sound is heard when the object hits
the ground)
(iii) potential energy of configuration of the body and the
ground (the object and the ground get deformed a
little bit at the point of collision).

OR

Power rating of each device, P = 500 W = 0.50 kW


Time for which each device runs, t = 10 h
Work done = Energy consumed by each device (E)
We know, power = Energy consumed / Time
Energy consumed by each device= Power × Time
E=Pxt
= 0.50×10 = 5 kWh
Hence, the energy consumed by four devices of power
500 W each in 10 h will be
4 × 5 kWh = 20 kWh = 20 units

33 Yes. Sound follows the same laws of reflection as that of 1+1+1


light because,
(i) Angle of incidence of sound is always equal to that of
angle of reflection of sound waves.
(ii) The direction in which sound is incident, the direction
in which it is reflected and normal all lie in the same
plane.
34 Elements – Compounds – Mixtures 2.5 +
Sodium – Calcium carbonate – Sugar solution ½+½+½+½+½
Silver – Methane – Soil
Tin – Carbon dioxide – Coal
Silicon – Soap – Air ,Blood

OR

By finding the boiling point of a given colorless liquid. If


the liquid boils at 100°C at atmospheric pressure, then
it is pure water. This is because pure substances have
fixed melting and boiling point.

Pure substances are: Ice, iron, hydrochloric acid,


calcium oxide and mercury
35 3+2

OR

(Any 3 labels)

OR
3+2

36 One method used for crop production which ensures 5


high yield is plant breeding. It is the science involved
in improving the varieties of crops by breeding
plants. The plants from different areas/places is
picked up with desired traits and then hybridisation
or cross-breeding of these varieties is done to obtain
a plant/crop of desired characteristic.
The high yielding crop variety shows the following
characteristics:
High yield, early maturation, less water for irrigation,
better quality seeds are produced, less fertilizers
required, adapts itself to the environmental
conditions.
37 (i) d 1+1+2
(ii) a
(iii) d
38 (i) a 1+1+1+1
(ii) c
(iii) d
(iv) c
39 (i) b 1+1+1+1

(ii) a

(iii) c

(iv) Distance and displacement are two quantities that


seem to mean the same but are different with
different meanings and definitions. Distance is the
measure of “how much distance an object has
covered during its motion” while displacement
refers to the measure of “how far the abject actually
from initial place.”

OR

(i) a

(ii) c

(iii) a

(iv)The speed of an object is the distance covered by


object per unit time.Velocity is defined as the
displacement by particle per unit time.
KENDRIYA VIDYALAYA SANGATHAN

RANCHI REGION

CLASS IX (Science)

Term II exam

(2022-2023)

Max. Marks :- 80 Time allotted :- 3 hours

General Instruction :-

· All questions are compulsory.

· The question paper has five sections- A, B, C, D & E and there are 39 questions.

· Section - A has two patterns of questions MCQ and ASSERTION-REASON.

Q.No. 1-16 :- MCQ (multiple choice questions)

Q.No. 17-20 :- ASSERTION-REASON

. Section - B has 6 questions carrying 2 marks each.

. Section - C has 7 questions carrying 3 marks each; and

. Section – D has 3 questions carrying 5 marks each.

. Section - E has 3 questions based on Case-based study carrying 4 marks each.

SECTION - A

M.C.Q (multiple choice questions) :- ( From 1-16)

1. What would be the displacement of the particle moving in a circular path of radius r after
a displacement full of circle?

(a) 2r (b) r ( c ) 3r (d) 0

2.If the mass of the body is doubled and its velocity becomes half , then the linear
momentum of the body will
(a) Becomes half (b) becomes doubled ( c) 0 (d) remain same

3. The value of acceleration due to gravity

(a) is same on equator and poles (b) is least on poles

(c) is least on equator (d) increases from pole to equator

4. The relationship between thrust and pressure is :-

(a)Pressure=Thrust/Area (b) Thrust= Pressure/ Area

(c)Pressure =Thrust ✕ Area (d) Thrust= Pressure ✕ Area

5.Which one of the following is not the unit of energy?

(a) joule (b) newton meter (c) kilowatt (d) kilowatt hour

6. In SONAR, we use

(a) ultrasonic waves (b) infrasonic waves

(c) radio waves (d) audible sound waves

7. Is there any similarity in materials ?

8. What is the mass percent of a solution ?

9. What is the atomicity of Argon ?

10. Which subatomic particle is absent in an ordinary hydrogen atom ?

11. Is it possible for the atom of an element to have one electron, one proton and no
neutron? If so, name the element .

12. Write the electronic configuration of an element whose atomic number is 12.

13. Animal cells doesn’t possess :-

a) Plasma membrane (b) Vacuoles (c)Cell wall (d) Mitochondria

14. Which of the cell organelle acts as- Protein factory?

a) Lysosomes (b) Ribosomes (c ) Golgi apparatus (d) Chloroplast

15. Xylem differs from phloem due to :-

a) Presence of vascular bundle


b) Parenchymal tissue
c) Fibrous tissues
d) Having tracheids

16. Husk of Coconut is made up of -

a) Sclerenchyma (b)Parenchyma (c )aerenchyma (d) collenchyma

ASSERTION-REASON :- (17-20)

Following questions consist of two statements – Assertion (A) and Reason (R). Answer these
questions selecting the appropriate option given below:

(a) Both A and R are true and R is the correct explanation of A.

(b) Both A and R are true but R is not the correct explanation of A.

(c) A is true but R is false.

(d) A is false but R is true.

17. Assertion :- Permanent tissue is made up of mature cells.

Reason :- Meristematic tissue is composed of a group of actively dividing cells.

18. Assertion: Atoms always combine to form molecules and ions.

Reason: Atoms of most elements are not able to exist independently.

19. Assertion :- Fertilizers and manure enhances crop production.

Reason :- Fertilizers and manure produce chemicals which enhance soil fertility.

20. Assertion:-The object which is in motion posses energy which we generally called kinetic
energy.

Reason :- A moving object always does work.

SECTION - B
21. How does evaporation differ from boiling ? Give two differences.

OR

Which gas is called dry ice? Why ?

22. What are polyatomic ions ? Give two examples.

23. Give two drawbacks of Dalton’s atomic theory.

24. Give a brief account on how the discovery of Cell has happened?
25. Plot a graph between force applied on a body and the acceleration produced in
the given mass, assuming that the magnitude of force is constantly changing.

26. Does Newton’s third law apply to a system where bodies do not actually touch
each other?

SECTION- C
27. Explain interconversion of three states of matter with the help of flow charts. Name the
process of each interconversion.

28. Give a comparative account on Diffusion and Osmosis. (compare at least between 3
points)

29.Draw and label different elements of Phloem.

OR

Draw and label different parts of Neuron.

30. Mention three differences between speed and velocity.

31. Illustrate Archimedes’ principle into your own words.

32. Name the transformation of energy involved in the following cases :

(a) When a body is thrown upwards.

(b) When a body falls from the top of a hill.

(c) When coal burns.

33. Write the difference between transverse waves and longitudinal waves.

SECTION- D
34. Non-metals are usually poor conductors of heat and electricity. They are non-lustrous,
non-sonorous, non-malleable and are coloured.

(a) Name a lustrous non-metal.

(b) Name a non-metal which exists as a liquid at room temperature.

(c) The allotropic form of a non-metal is a good conductor of electricity. Name the
allotrope.

(d) Name a non-metal which is known to form the largest number of compounds.
(e) Name a non-metal other than carbon which shows allotropy.

OR

Classify the following into metals, non-metals and metalloids:

(i) Germanium (ii) Boron

(iii) Diamond (iv) Iodine

(v) Copper

35. a) what will happen to the cell when;

i) A cell having higher water concentration than the surrounding medium.

ii) A cell having lower water concentration than the surrounding medium.

iii) A cell having equal water concentration to its surrounding medium.

b) Name the materials of which the cell membrane and cell wall are composed of ?

36. The earth attracts an apple. Does the apple also attract the earth? If it does, why
does the earth not move towards the apple?

OR

Mention any four phenomena that the universal law of gravitation was able to
explain.

SECTION- E
37. Cattle husbandry is done for two purposes— milk and draught labour for agricultural work
such as tilling, irrigation and carting. Indian cattle belong to two different species, Bos Indicus,
cows, and Bosbubalis, buffaloes. Milk-producing females are called milch animals (dairy
animals), while the ones used for farm labour are called draught animals.

Milk production depends on the duration of the lactation period, meaning the period of milk
production after the birth of a calf. So, milk production can be increased by increasing the
lactation period. Exotic or foreign breeds (for example, Jersey, Brown Swiss) are selected for
long lactation periods, while local breeds (for example, Red Sindhi, Sahiwal) show excellent
resistance to diseases. The two can be cross-bred to get animals with both the desired qualities.
Proper cleaning and shelter facilities for cows and buffaloes are required for humane farming,
for the health of the animals and for production of clean milk as well. The food requirements of
dairy animals are of two types: (a) maintenance requirement, which is the food required to
support the animal to live a healthy life, and (b) milk producing requirement, which is the type
of food required during the lactation period.
(1) Identify the exotic breed of cow
(a) Red Sindhi

(b) Sahiwal
(c) Brown Swiss

(d) All of the above


(4) Enlist any two Indian cattle species.

(5) What are the food requirements of dairy animals?

38. There are three states of matter – solid, liquid and gas.

Solids have a definite shape, distinct boundaries and fixed volumes, that is, have negligible
compressibility. Solids have a tendency to maintain their shape when subjected to outside force.
Solids may break under force but it is difficult to change their shape, so they are rigid.
Liquids have no fixed shape but have a fixed volume. They take up the shape of the container in
which they are kept. Liquids flow and change shape, so they are not rigid but can be called fluid.
Gas has an indefinite shape, no fixed volume. Gas gets the shape and volume of the container.

Gas has very low density hence is light. Gas can flow easily and hence is called fluid.
i.) Which of the following states of matter takes the shape of the container in which it is filled?
a.) Solid

b.) Liquid
c.) Gas

d.) Both b and c

ii.) Distance between particles of matter least in


a.) Solid
b.) Liquid
c.) Gas

d.) None of these

iii.) Compressibility is least in case of


a.) Solid
b.) Liquid
c.) Gas

d.) None of these


iv) Compare between Solids and gases.
39. The growth of plants occurs only in certain specific regions. This is because the dividing
tissue, also known as meristematic tissue, is located only at these points. Depending on the
region where they are present, meristematic tissues are classified as apical, lateral and
intercalary. New cells produced by meristem are initially like those of meristem itself, but as
they grow and mature, their characteristics slowly change and they become differentiated as
components of other tissues.

Apical meristem is present at the growing tips of stems and roots and increases the length of
the stem and the root. The girth of the stem or root increases due to lateral meristem
(cambium). Intercalary meristem seen in some plants is located near the node
Cells of meristematic tissue are very active, they have dense cytoplasm, thin cellulose walls and
prominent nuclei. They lack vacuoles.

(1) Which meristem helps in increasing the girth of the plant?

(a) Primary meristem


(b) Apical meristem
(c) Intercalary meristem

(d) Lateral meristem


(2) Lateral meristem is responsible for_____________
(a) Growth of apical portion

(b) Increase in the length

(c) Increasing the girth of stem and root

(d) Growth in parenchyma


(3) The meristem present at the base of the internode is_____________
(a) Lateral meristem
(b) Intercalary Meristem
(c) Apical Meristem

(d) All of the above

4) Which meristem is responsible for the vertical growth of plant_____________

a) Apical meristem
b) Lateral meristem
c) Intercalary meristem
d) None of the above
KENDRIYA VIDYALAYA SANGATHAN, RANCHI REGION
SCIENCE
Sample Paper
MARKING SCHEME
CLASS IX (SESSION-2022-23)
1. 1
(a) 2r

2. 1
(d) remain same

3. 1
(c) is least on the equator .

4. 1
(a)Pressure=Thrust/Area

5. 1
(c) kilowatt

6. 1
(c) radio waves

7. Yes, all materials possess mass and occupy space. 1

8. It is defined as the mass in grams of the solute present in one hundred grams 1
of a solution.

9. Mono atomic 1

10. Neutron 1

1
11. Yes, it is true for hydrogen atom which is represented as 1H 1

12. K, L, M `1
2, 8, 2

13. c) cell wall 1

14. 1
(b) Ribosomes
15. 1
(d) Having tracheids

16. 1
a) Sclerenchyma

17. (b) Both Assertion and reason are true, but reason is not the correct 1
explanation of Assertion.

18. (b) Both A and R are true but R is not the correct explanation of A. 1

19. (c ) Assertion is true but Reason is false. 1

20. (b) Both Assertion and reason are true, but reason is not the correct 2
explanation of Assertion.

21.
Evaporation Boiling

1. Evaporation takes place at all 1. Boiling takes place only at the 1


temperatures. boiling point of the liquid.

2. It is a very slow process. 2. It is a fast process. 1

OR

Solid CO2 is known as dry ice. 1


This is because it directly gets converted into gaseous state without passing
through liquid state on decreasing the pressure to 1 atmosphere. 1

22. A group of atoms having a charge is known as polyatomic ion. 2


Examples : NH4+ , SO42-

23. Drawbacks of Dalton’s Atomic Theory :

(i) According to modern theory, an atom is not the ultimate indivisible particle 1
of matter. Today, we know that atoms are divisible, i.e., they are themselves
made-up of particles (protons, electrons, neutrons, etc.).

(ii) In the case of isotopes of an element, the assumption that the atoms of 1
the same element have the same mass does not hold good.

24. Robert Hooke in 1665, examined a thin slice of cork under a self-designed 2
crude microscope and observed that the cork resembled the structure of a
honeycomb. This structure consisted of many tiny compartments. Hooke
called them cellulae (Singular cellula), which is now termed as Cells. Cellula is
a Latin name which means a 'little room'.

25. Here, in this graph x-axis is showing the acceleration, y-axis is showing 2
the force.

26. Yes, whenever the bodies are in actual contact or even if there is an 2
interaction between the bodies (e.g., attraction or repulsion between
two magnets charges, etc.), Newton’s third law is applicable.

27. 3

28. 3

29. Sieve tubes, companion cells, phloem fibers and phloem parenchyma 3
are the different elements of phloem.
OR

30. 3

Speed Velocity

1.Distance travelled by an object 1.The distance travelled by an


per unit time is known as speed. object particular distance that is
displacement per unit time is
known as velocity

2. Average speed of an object 2. average velocity of an object


cannot be zero. can be zero.

3. Speed tells about how fast an 3. velocity tells about how fast an
object is moving object is moving and in which
direction.

31. When a body is immersed partially or completely in a fluid (liquid or 3


gas), it experiences an upthrust or buoyant force which is equal to the
weight of the fluid displaced by the body. The weight of the body
decreases due to the buoyant force acting on the body, when immersed
in a fluid. In other words, a body loses its weight, when immersed
completely or partially in a fluid. The loss of weight of a body in a fluid
is equal to the upthrust or buoyant force.
The upthrust or buoyant force = weight of fluid displaced by a body =
weight of body in air – weight of body in fluid.

32. (a) Kinetic energy into potential energy. 3


(b) Potential energy into kinetic energy.
(c) Chemical energy into heat energy.

33. 3

Transverse Waves Longitudinal Waves

1. The particles of the medium 1. The particles of the medium

oscillate up and down about their move in parallel to the direction

mean position. of propagation of the

disturbance.

2. They propagate as crests and 2. They propagate as

troughs. compressions and rarefactions.

3. The propagation of waves is 3. The propagation of these

possible in solid or surface of waves is possible in solids,

liquid but not in gases. liquids and gases.

Example: Light wave Example: Sound wave


34. (a) Iodine 1
(b) Bromine 1
(c) Graphite 1
(d) Carbon 1
(e) Sulphur 1
OR
Metal – Copper 1
Non-metals – Diamond, iodine 1+1
Metalloids – Germanium, boron. 1+1

35. i) As the cell has higher water concentration, water will move from higher 1
concentration to lower concentration, thus the cell will shrink.
ii) Here, the cell will swell. 1
iii) Here, due to equal water concentration ,no change will occur in the cell. 1
b) Plasma membrane composed of lipids and protein, and cell wall composed
of cellulose. 1

36. According to Newton’s third law of motion, action and reaction are equal and 4
opposite. It means that the force on the apple due to earth’s attraction is
equal to that on the earth due to apple’s attraction. But we know,
acceleration ∝ 1/m.
As the mass of the earth is very large as compared to that of the apple,
the acceleration experienced by the earth will be so small that it will not
be noticeable.
OR
The universal law of gravitation was able to explain successfully

● the force that binds us to the earth.


● the motion of the moon around the earth.
● the motion of planets around the sun.
● the tides due to the moon and the sun.

37. (1) C 1
(4) Indian cattle belong to two different species 2

● Bos Indicus – cows.


•Bosbubalis – buffaloes.

(5) The food requirements of dairy animals are of two types 2


● Maintenance requirement – which is the food required to support the
animal to live a healthy life.
•Milk producing requirement -which is the type of food required
during the lactation period.

38. 1
i.) d 1
1
ii.) a

iii.) a

Solids Gases 2
Solid has fixed volume.
Gas has no fixed volume.

Solid has fixed shape Gas gets the shape and volume of the
container.

Solids have high density. Gases have very less density.

39. 1. d) Lateral meristem 1

c) Increasing the girth of stem and root 2


(b) Intercalary Meristem
1
a) Apical meristem
1
KENDRIYA VIDYALAYA SANGATHAN RANCHI REGION

Sample Paper (MLL)


Class IX 2022-23
Science (086)
Time: 3 Hours Max. Marks: 80
General Instructions:
1. This question paper consists of 39 questions in 5 sections.
2. All questions are compulsory. However, an internal choice is provided in some
questions. A student is expected to attempt only one of these questions.
3. Section A consists of 20 Objective Type questions carrying 1 mark each.
4. Section B consists of 6 Very Short questions carrying 02 marks each. Answers to these
questions should in the range of 30 to 50 words.
5. Section C consists of 7 Short Answer type questions carrying 03 marks each. Answers
to these questions should in the range of 50 to 80 words.
6. Section D consists of 3 Long Answer type questions carrying 05 marks each. Answer to
these questions should be in the range of 80 to 120 words.
7. Section E consists of 3 source-based/case-based units of assessment of 04 marks each
with sub-parts.

SECTION-A
Select and write one most appropriate option out of the four options given for each of the
questions 1 – 20.

1. Figure P below shows a beaker of water being heated directly. However, some liquids
like alcohol are heated using a water bath (figure Q). Which of these is NOT likely to be
a reason for water baths to be used?
(a) It is safer to use them with inflammable liquids.

(b) It allows more uniform heating of the liquid.

(c) It is a faster way of heating a liquid.

(d) It is convenient for liquids with low boiling points.


2. When a beam of light is passed through a colloidal solution, it gets
(a) reflected (b) absorbed

(c) scattered (d) refracted

3. The molecular formula of water is


(a) H2 O (b) CO2 (c) CO (d) HO

4. Which of the following has a charge of +1 and a mass of 1 amu?


(a) A neutron (b) A proton

(c) An electron (d) A helium nucleus

5. Amoeba acquires its food by the process of


(a) exocytosis (b) endocytosis

(c) osmosis (d) diffusion

6. Arenchyma cells containing air cavities are called

(a) aerenchyma (b) sclerenchyma

(c) chlorenchyma (d) prosenchyma

7. A passenger travels along the straight road for half the distance with velocity v1 and the
remaining half distance with velocity v2. Then average velocity is given by

(a) v v1 2 (b) vv1222


(c) ^v1+2 v2h (d) ^v21v v+1 2v2h

8. A number of forces acting on a body changes velocity of the body. The forces cannot be
(a) paralleled (b) unbalanced

(c) balanced (d) inclined


9. When a solid cube made of wax (density 0.9 g/cc) is placed in a beaker of alcohol
(density 0.8 g/cc), it sinks (see figure 1). The same cube when placed in water (density
1g/cc), it floats (see figure 2). What will happen if the cube of wax is placed in vegetable
oil (density 0.9 g/cc, almost the same as the wax itself)?

(a) It will sink to the bottom (same as figure 1).

(b) It will float (same as figure 2).

(c) It is not possible that a solid and liquid have the same density.

(d) The solid will stay in the liquid at any point it is placed without sinking or floating.

10. What energy conversion takes place when a TV is switched on for a long period of time?
(a) Electrical energy to light and sound energy only.

(b) Electrical energy to light, heat and sound energy.

(c) Electrical energy to mechanical energy only.

(d) Electrical energy to light energy only.

11. In case of transverse waves, the particles of a medium vibrate


(a) in the direction of wave propagation

(b) opposite to the direction of wave propagation

(c) at the right angles to the direction of wave propagation (d) none of these

12. Inner surface of fallopian tubes, bronchi and bronchioles are lined by epithelial cells.
Find the type of epithelia which lines them.
(a) Squamous epithelium (b) Ciliated epithelium
(c) Columnar (d) Cubical epithelium

13. A mixture contains four solid compounds A, B, C, D. On heating C changes to vapour


state. C can be separated from rest of the solids by
(a) crystallisation (b) sublimation

(c) distillation (d) filtration


14. In which of the following the valency of each of the constituent elements is equal to the
total number of atoms is one molecule of the compound?
(a) HCl (b) H S2

(c) CaO (d) MgCl2

15. A plant cell placed in a hypotonic solution will not burst because of presence of
(a) plasma membrane (b) cell wall

(c) chloroplast (d) cytoplasm

16. Which of the following tissues provides flexibility and mechanical support to the plant
organs?
(a) Collenchyma (b) Sclerenchyma

(c) Parenchyma (d) Chlorenchyma

Question no. 17 to 20 are Assertion-Reasoning based questions.

17. Assertion : The displacement of an object can be either positive, negative or zero.
Reason : Displacement has both the magnitude and direction.
(a) Both assertion and reason are true and reason is the correct explanation of
assertion.

(b) Both assertion and reason are true but reason is not the correct explanation of
assertion.

(c) Assertion is true but reason is false.

(d) Both assertion and reason are false.

18. Assertion : Mass is a measure of inertia of the body in linear motion.


Reason : Greater the mass, greater is the force required to change its
state of rest or motion. (a) Both assertion and reason are true and
reason is the correct explanation of assertion.
(b) Both assertion and reason are true but reason is not the correct explanation of
assertion.

(c) Assertion is true but reason is false.

(d) Both assertion and reason are false.

19. Assertion : A block of ice with a lead shot embedded in it floats on water contained in a
vessels. The temperature of the system is kept at 0cC as the ice melts. When ice has
melted completely, water level in the vessel rises. Reason : The melted ice will raise the
water level.
(a) Both assertion and reason are true and reason is the correct explanation of
assertion.

(b) Both assertion and reason are true but reason is not the correct explanation of
assertion.

(c) Assertion is true but reason is false.

(d) Both assertion and reason are false.


20. Assertion : A spring has potential energy, both when it is compressed or stretched.
Reason : In compressing or stretching, work is done on the spring
against the restoring force. (a) Both assertion and reason are true and
reason is the correct explanation of assertion.

(b) Both assertion and reason are true but reason is not the correct explanation of
assertion.

(c) Assertion is true but reason is false.

(d) Assertion is false but reason is true.

SECTION-B
Question no. 21 to 26 are very short answer questions.

21. List any three applications of centrifugation.


o
Give difference between mixture and compound.

22. What is the difference between hydrogen chloride and nitrogen molecule formation ?

23. Write the functions of cell wall in plant cell.

24. Why should we be sent flying in space if the force of gravity somehow vanishes today?
25. How does a stretched string on being set into vibration, produce the audible sound?
o
Will the sound be audible if the string is set into vibration on the surface of the Moon?
Give reason for your answer.

26. Organism which enriches the soil with nutrients is called biofertilizers.
(a) Write its advantage.
(b) Give example.

SECTION-C
Question no. 27 to 33 are short answer questions.

27. What do you observe when force is applied and then removed on the plunger of the
syringe containing air? Give a reason for your answer.

28. Write any three differences between a physical change and a chemical change.

29. Why do plant cells have more in number and big-sized vacuoles as compared to the
animal cells?
o
(i) What will happen when eukaryotic cells are placed in hypotonic solution?
(ii) What will happen if eukaryotic cells are placed in hypertonic solution?
(iii) What will happen if eukaryotic cells are placed in isotonic solution?
30. Explain how the bark of a tree is formed. How does it act as a protective tissue?

31. Give one example each of type of motion when :


(a) acceleration is positive.
(b) acceleration is negative.
(c) acceleration is zero.

32. Name the transformation of energy involved in the following cases :


(a) When a body is thrown upwards.
(b) When a body falls from the top of a hill.
(c) When coal burns.
(d) When a gas burns.
o
Is it possible that a body is in accelerated motion tinder a force acting on the body, yet
no work is being done by the force? Explain your answer giving a suitable example.

33. How do you account for the fact that two strings can be used to give notes of the same
pitch and loudness but of different quality?
SECTION-D
Question no. 34 to 36 are Long answer questions.

34. Is there any relationship between atomic number, mass number, isotopes, isobars and
valency of an atom ? Explain.
o
Number of electrons, protons and neutrons in chemical species A, B, C and D is given
below :
Elements Electrons Protons Neutrons
A 2 3 4
B 10 9 8
C 8 8 8
D 8 8 10
Now, answer the following questions :
(a) What is the mass number of A and B ?
(b) What is the atomic number of B ?
(c) Which two elements represent a pair of isotopes and why ?
(d) What is the valency of element C ?
Also, justify your answers.

35. What are three main categories of connective tissue?


o
Explain the structure of a fluid connective tissue.
36.The following is the distance-time table of an object in motion :
Time (in seconds) Distance (in metres)
0 0
1 1
2 8
3 27
4 64
5 125
6 216
7 343
(a) What conclusion can you draw about the acceleration? Is it constant, increasing,
decreasing, or zero?
(b) What do you infer about the forces acting on the object?

SECTION-E
Question no. 37 to 39 are case-based/data-based questions with 2 to 3 short sub-parts.
Internal choice is provided in one of these sub-parts.
37. Rutherford (1871-1937) was known as the ‘Father’ of nuclear physics. He is famous for
his work on radioactivity and the discovery of the nucleus of an atom with the gold foil
experiment. Ernest Rutherford was interested in knowing how the electrons are
arranged within an atom. Rutherford designed an experiment for this. In this
experiment, fast moving alpha α -particles were made to fall on a thin gold foil. On the
basis of his experiment, Rutherford put forward the nuclear model of an atom, which
had the following features:
• There is a positively charged centre in an atom called the nucleus. Nearly all the mass
of an atom resides in the nucleus.
• The electrons revolve around the nucleus in circular paths.
• The size of the nucleus is very small as compared to the size of the atom.
Drawbacks of Rutherford’s model of the atom: The revolution of the electron in a
circular orbit is not expected to be stable. Any particle in a circular orbit would
undergo acceleration. During acceleration, charged particles would radiate energy.
Thus, the revolving electron would lose energy and finally fall into the nucleus. If this
were so, the atom should be highly unstable and hence matter would not exist in the
form that we know. We know that atoms are quite stable.
(a) Who is the father of nuclear physics?
(b) What is the name of positively charge center in an atom?
(c) Write the features of Rutherford’s nuclear model of an atom?
o
(c) What do you mean by nucleus?
38. Blood is a type of connective tissue. The cells of connective tissue are loosely spaced and
embedded in an intercellular matrix. The matrix may be jelly like, fluid, dense or rigid.
The nature of matrix differs in concordance with the function of the particular
connective tissue. Blood has a fluid (liquid) matrix called plasma, in which red blood
corpuscles (RBCs), white blood corpuscles (WBCs) and platelets are suspended. The
plasma contains proteins, salts and hormones. Blood flows and transports gases,
digested food, hormones and waste materials to different parts of the body. Bone is
another example of a connective tissue. It forms the framework that supports the
body. It also anchors the muscles and supports the main organs of the body. It is a
strong and non-flexible tissue. Bone cells are embedded in a hard matrix that is
composed of calcium and phosphorus compounds. Two bones can be connected to
each other by another type of connective tissue called the ligament. This tissue is very
elastic. Another type of connective tissue, cartilage, has widely spaced cells. The solid
matrix is composed of proteins and sugars. Cartilage smoothness bone surfaces at
joints and is also present in the nose, ear, trachea and larynx.
(a) Where are connective tissues located?
(b) Is areolar tissue is found between skin and muscles?
(c) Which of the tissues connect two bones?
o
(c) What are the function of connective tissue?
39. Newton’s first law of motion states that a body at rest will remain at rest position only
and a body which is in motion continues to be in motion unless otherwise they are
acted upon by an external force. In other words, all objects resist a change in their
state of motion. In a qualitative way, the tendency of undisturbed objects to stay at
rest or to keep moving with the same velocity is called inertia. This is why, the first law
of motion is also known as the law of inertia.
(a) What is the another name of first law of motion?
(b) What will happen if there is an absence of external force?
(c) State Newton’s first law of motion.
or
(d) Why Newton’s first law of motion is called law of inertia?
Sample Paper MLL CLASS -IX
Science (086)
Marking Scheme
Time: 3 Hours Max. Marks: 80

SECTION-A
Select and write one most appropriate option out of the four options given for each of the
questions 1 – 20.
1. Figure P below shows a beaker of water being heated directly. However, some
liquids like alcohol are heated using a water bath (figure Q). Which of these is NOT likely to
be a reason for water baths to be used?
(a) It is safer to use them with inflammable liquids.
(b) It allows more uniform heating of the liquid.
(c) It is a faster way of heating a liquid.
(d) It is convenient for liquids with low boiling points.
Ans : (c) It is a faster way of heating a liquid.
Water bath is used to boil the liquids which easily catches fire. Water is heated directly over
flame as it does not catch fire easily. But liquids like alcohol are heated in water bath to
avoid catching fire by alcohol. Heating in water bath allows more uniform heating of the
liquids. Water bath is convenient for liquids with low boiling points. As low boiling point
liquids are volatile in nature. On heating over flame, temperature rises faster, and the liquid
evaporate easily. Therefore option(c) is correct.
2. When a beam of light is passed through a colloidal solution, it gets
(a) reflected (b) absorbed
(c) scattered (d) refracted
Ans : (c) scattered
The scattering of beam of light on passing through colloidal solution is known as Tyndall
effect.
3. The molecular formula of water is
(a) H2 O (b) CO2 (c) CO (d) HO

Ans : (a) H2O


4. Which of the following has a charge of +1 and a mass of 1 amu?
(a) A neutron (b) A proton
(c) An electron (d) A helium nucleus
Ans : (b) A proton
The mass of a proton is taken as one unit and its charge as plus one.
5. Amoeba acquires its food by the process of (a) exocytosis (b) endocytosis (c)
osmosis (d) diffusion
Ans : (b) endocytosis
Amoeba acquires its food through endocytosis Endocytosis refers to invagination of a small
region of the plasma membrane and ultimately forming an intra-cellular membrane bound
vesicle. This process is generally involved in the ingestion of food material. Intake of liquid
food using endocytosis is called pinocytosis or cell drinking. Similarly, intake of solid
particles by a cell through its cell membrane is called phagocytosis or cell eating. In this
process, cell membrane puts up protoplasmic processes around the food particle. The
processes join, fuse to form phagosome.
6. Aerenchyma cells containing air cavities are called

(a) aerenchyma (b) sclerenchyma


(c) chlorenchyma (d) prosenchyma
ns : (a) aerenchyma
In aquatic plants (hydrophytes) large air cavities are present in the parenchymatous tissue.
These cavities store gases and provide buoyancy to aquatic plants to help them float. Such
parenchyma are called aerenchyma.
7. A passenger travels along the straight road for half the distance with velocity v1 and
the remaining half distance with velocity v2. Then average velocity is given by
(a) v v1 2 (b) vv1222
(c) ^v1+2 v2h (d) ^v21v v+1 2v2h
ns : (d) ^v21v v+1 2v2h
2xvx21 ++ 2x2xv2 = dv22v v+11 2v1n = v21v v+1 2v2
8. A number of forces acting on a body changes velocity of the body. The forces cannot
be
(a) paralleled (b) unbalanced
(c) balanced (d) inclined
ns : (c) balanced
When a number of forces acting on a body change velocity of body, they produce a non zero
acceleration, the forces are unbalanced, parallel and inclined.
9. When a solid cube made of wax (density 0.9 g/cc) is placed in a beaker of alcohol
(density 0.8 g/cc), it sinks (see figure 1). The same cube when placed in water (density
1g/cc), it floats (see figure 2). What will happen if the cube of wax is placed in vegetable oil
(density 0.9 g/cc, almost the same as the wax itself)?

(a) It will sink to the bottom (same as figure 1).


(b) It will float (same as figure 2).
(c) It is not possible that a solid and liquid have the same density.
(d) The solid will stay in the liquid at any point it is placed without sinking or floating.
ns : (d) The solid will stay in the liquid at any point it is placed without sinking or floating.
An object will float if it is less dense than the liquid it is placed in and an object will sink if it
is more dense than the liquid it is placed in.
10. What energy conversion takes place when a TV is switched on for a long period of
time?
(a) Electrical energy to light and sound energy only.
(b) Electrical energy to light, heat and sound energy.
(c) Electrical energy to mechanical energy only. (d) Electrical energy to light energy
only.
ns : (b) Electrical energy to light, heat and sound energy
When a TV is switched on then electrical energy is converted into light energy and sound
energy and for a long period this energy is converted into heat energy and dissipated at the
component to the TV.
11. In case of transverse waves, the particles of a medium vibrate
(a) in the direction of wave propagation
(b) opposite to the direction of wave propagation
(c) at the right angles to the direction of wave propagation
(d) none of these
ns : (c) at the right angles to the direction of wave propagation
If the particles of a medium move perpendicular to the direction of motion of the wave, the
wave is called a transverse wave.
12. Inner surface of fallopian tubes, bronchi and bronchioles are lined by epithelial cells.
Find the type of epithelia which lines them.
(a) Squamous epithelium
(b) Ciliated epithelium
(c) Columnar
(d) Cubical epithelium
ns : (b) Ciliated epithelium
Columnar cells which bear cilia on their free surfaces, lines the nasal passages, oviducts
(fallopian tubes), terminal bronchioles, ventricles of brain, central canal of spinal cord of the
embryo.
13. A mixture contains four solid compounds A, B, C, D. On heating C changes to vapour
state. C can be separated from rest of the solids by (a) crystallisation (b) sublimation
(c) distillation (d) filtration
ns : (b) sublimation
Sublimation technique of purification is used for those solids which are directly converted to
gas phase without undergoing liquid phase.
14. In which of the following the valency of each of the constituent elements is equal to
the total number of atoms is one molecule of the compound?
(a) HCl (b) H S2
(c) CaO (d) MgCl2
ns : (c) CaO
In CaO, valency of each of the constituent elements is 2 and is equal to the total number of
atoms (2) in one molecule of the compound.
15. A plant cell placed in a hypotonic solution will not burst because of presence of
(a) plasma membrane (b) cell wall
(c) chloroplast (d) cytoplasm
ns : (b) cell wall
Cell walls permit the cells of plants, fungi and bacteria to withstand very dilute (hypotonic)
external media without bursting. In such media the cells tend to take up water by osmosis.
The cell swells, building up pressure against the cell wall. The wall exerts an equal pressure
against the swollen cell. Because of their walls, such cells can withstand much greater
changes in the surrounding medium than animal cells.
16. Which of the following tissues provides flexibility and mechanical support to the
plant organs?
(a) Collenchyma (b) Sclerenchyma
(c) Parenchyma (d) Chlorenchyma
ns : (a) Collenchyma
Collenchyma is a living tissue of primary body. The cells are thin-walled but possess
thickenings of cellulose and pectic substances at the corners where number of cells join
together. The tissue provides flexibility to soft aerial parts (e.g., leaves, young stems) of
plant so that they can bend without breaking. The cells are compact and the intercellular
spaces are absent.
Question no. 17 to 20 are Assertion-Reasoning based questions.
17. Assertion : The displacement of an object can be either positive, negative or zero.
Reason : Displacement has both the magnitude and direction.
(a) Both assertion and reason are true and reason is the correct explanation of
assertion.
(b) Both assertion and reason are true but reason is not the correct explanation of
assertion.
(c) Assertion is true but reason is false. (d) Both assertion and reason are false.
ns : (b) Both assertion and reason are true but reason is not the correct explanation of
assertion.
Displacement may be positive, negative or zero. Displacement is a vector quantity.
18. Assertion : Mass is a measure of inertia of the body in linear motion.
Reason : Greater the mass, greater is the force required to change its state of rest or
motion.
(a) Both assertion and reason are true and reason is the correct explanation of
assertion.
(b) Both assertion and reason are true but reason is not the correct explanation of
assertion.
(c) Assertion is true but reason is false. (d) Both assertion and reason are false.
Ans : (a) Both assertion and reason are true and reason is the correct explanation of
assertion.
According to Newton’s second law of motion, a=mF i.e., magnitude of the acceleration
produced by a given force is inversely proportional to the mass of the body. Higher is the
mass of the body, lesser will be the acceleration produced i.e., mass of the body is a
measure of the opposition offered by the body to change a state, when the force is applied
i.e., mass of a body is the measure of its inertia.
19. Assertion : A block of ice with a lead shot embedded in it floats on water contained
in a vessels. The temperature of the system is kept at 0cC as the ice melts. When ice has
melted completely, water level in the vessel rises.
Reason : The melted ice will raise the water level.
(a) Both assertion and reason are true and reason is the correct explanation of
assertion.
(b) Both assertion and reason are true but reason is not the correct explanation of
assertion.
(c) Assertion is true but reason is false. (d) Both assertion and reason are false.
Ans : (d) Both assertion and reason are false.
The water level in the vessel remains unchanged. The volume or melted ice and lead shot, is
equal to the volume of the water displaced when the ice block with lead shot is initially
placed in water.
20. Assertion : A spring has potential energy, both when it is compressed or stretched.
Reason : In compressing or stretching, work is done on the spring against the restoring
force.
(a) Both assertion and reason are true and reason is the correct explanation of
assertion.
(b) Both assertion and reason are true but reason is not the correct explanation of
assertion.
(c) Assertion is true but reason is false.
(d) Assertion is false but reason is true.
ns : (a) if both assertion and reason are true and reason is the correct explanation of
assertion.
When spring is compressed or stretched, the work is done on the spring. Due to this work,
the energy gets stored in it as elastic potential energy.
SECTION-B
Question no. 21 to 26 are very short answer questions.
21. List any three applications of centrifugation.
Ans :
Applications of centrifugation are as follows :
(i) Used in dairies and homes to separate cream from milk or butter from cream.
(ii) Used in washing machines to squeeze out water from clothes.
(iii) Used in laboratories to separate colloidal particles from their solutions.
(iv) Used in diagnostic labs for blood and urine test.
o
Give difference between mixture and compound.
Ans :
The difference between mixture and compound are as follows :
Compounds Mixtures
1. Compounds are pure substances. Mixtures are impure substances.
2. Compounds are made up of two or more elements combined chemically. Mixtures
are made up of two or more substances mixed physically.
3. The components of a compound are present in a fixed ratio. The components
of a mixture are present in different ratio.
4. Compounds have same properties throughout the compound part. Mixtures
do not have same properties throughout the mixture part.
5. A new substance is formed. No new substance is formed.
6. The components of a compound can be separated only by chemical methods. The
components of a mixture can be separated by physical methods.
22. What is the difference between hydrogen chloride and nitrogen molecule formation
?
Ans :
Hydrogen chloride is molecular compound and formed by the union of different kinds of
atoms while nitrogen is diatomic molecule and formed by union of two atoms of same
kinds.
23. Write the functions of cell wall in plant cell.
Ans :
The functions of cell wall in plant cell are as follows:
(i) It provides rigidity and strength to the cell.
(ii) It gives a definite shape to the cell.
(iii) It withstands the osmotic pressure which is developed by cell contents.
(iv) It protects the inner cell organelles bounding the cell from outside.
24. Why should we be sent flying in space if the force of gravity somehow vanishes
today?
Ans :
The centripetal force required to keep us rotating
along with the Earth would not be available in the absence of force of gravity. We would
then fly off along the tangent to the Earth into the space.
25. How does a stretched string on being set into vibration, produce the audible sound?
Ans :
On being set into vibrations, the stretched string, forces the surrounding air to vibrate. This
vibrating air, in turn, affects our eardrum and produces an audible sound.
or
Will the sound be audible if the string is set into vibration on the surface of the Moon? Give
reason for your answer.
Ans :
No, we will not hear any audible sound on the surface of the Moon. This is because sound
requires a medium to propagate, since there is no atmosphere on the surface of Moon,
therefore, the sound will not be heard.
26. Organism which enriches the soil with nutrients is called biofertilizers.
(a) Write its advantage.
(b) Give example.
Ans :
(a) Biofertilizers are non-pollutant sources of plant nutrients. They are renewable.
(b) Example of biofertilizers are Rhizobium, Blue green algae.
SECTION-C
Question no. 27 to 33 are short answer questions.
27. What do you observe when force is applied and then removed on the plunger of the
syringe containing air? Give a reason for your answer.
Ans :
The plunger moves downward on the application of force to a considerable length. The
plunger moves backward and takes its original position when the force is removed.
The gases have large intermolecular spaces. So, they easily get compressed on the
application of force. The compressed gases are under high pressure. When the force is
removed, this high pressure forces the plunger back to its original position.
28. Write any three differences between a physical change and a chemical change.
Ans :
Given below is a comparison of two types of changes:
Physical changes Chemical changes
(i) Changes take place only in properties such as colour, physical state, density, etc.
Change results in the formation of new chemical substance(s).
(ii) Change is temporary. Change is permanent.
(iii) Original substance(s) can be obtained back easily. Original substance(s) cannot be
obtained back easily.
(iv) Chemical properties of a substance remain unchanged even after the change.
New substance(s) with different properties are formed.
29. Why do plant cells have more in number and bigsized vacuoles as compared to the
animal cells?
Ans :
Plants cells attain turgidity and rigidity due to the more number of vacuoles as well as large-
sized vacuoles help the plant cells to withstand the wear and tear, external environmental
conditions. They also help in the storage of essential material required by plants for their
growth like amino acids, sugar and various organic substances.
or
(i) What will happen when eukaryotic cells are placed in hypotonic solution?
(ii) What will happen if eukaryotic cells are placed in hypertonic solution?
(iii) What will happen if eukaryotic cells are placed in isotonic solution?
Ans :
(i) When eukaryotic cells are placed in hypotonic solution, the water molecules will
enter into the cell and the cell will swell up.
(ii) If eukaryotic cells are placed in hypertonic solution, the water molecules will come
out of the cell and the cell will shrink.
(iii) If the eukaryotic cell is placed in isotonic solution, the amount of water molecule will
remain the same; it will neither move out nor will go inside. The cell will remain same sized.
30. Explain how the bark of a tree is formed. How does it act as a protective tissue?
Ans :
As trees grow old, a strip of secondary meristem replaces the epidermis of the stem. Cell on
the outside are cut off this layer. This forms the several layer thick cork or the bark of the
tree. Bark is a mass of dead tissue lying in the peripheral region of the plant body as a hard
dry covering. Its function is protection. It protects the inner tissues against the attack of
fungi and insects, against loss of water by evaporation, etc.
31. Give one example each of type of motion when : (a) acceleration is positive.
(b) acceleration is negative.
(c) acceleration is zero.
Ans :
(a) When the driver of a car starts motion from rest and increases its velocity,
acceleration is positive.
(b) When driver of a running car/train applies brakes so as to stop the car/train,
acceleration is negative.
(c) When train is running at constant, velocity along a straight railway track, its
acceleration is zero.
32. Name the transformation of energy involved in the following cases :
(a) When a body is thrown upwards.
(b) When a body falls from the top of a hill.
(c) When coal burns.
(d) When a gas burns.
Ans :
(a) Kinetic energy into potential energy.
(b) Potential energy into kinetic energy.
(c) Chemical energy into heat energy.
(d) Chemical energy into heat energy.

Is it possible that a body is in accelerated motion tinder a force acting on the body, yet no
work is being done by the force? Explain your answer giving a suitable example.
Ans :
Yes, it is possible, when the force is perpendicular to the direction of motion. The Moon
revolving round the Earth under the centripetal force of attraction of the Earth but Earth
does not do any work on the motion of the Moon.
33. How do you account for the fact that two strings can be used to give notes of the same
pitch and loudness but of different quality?

Ans :
The ‘quality’ of a given note is determined by the overall effect of the harmonics present in
it. The harmonics are multiples of the fundamental or basic frequency of the ‘note’.
Depending on the conditions under which vibrations are taking place, sometimes we get
one set of harmonics and sometimes another set. The quality of the two notes will,
therefore, different even though their fundamental frequencies may be the same.
SECTION-D
Question no. 34 to 36 are Long answer questions.
34. Is there any relationship between atomic number, mass number, isotopes, isobars and
valency of an atom ? Explain.
Ans :
Atomic number : It tells the number of protons ( )Z . Atomic Mass : Total number of proton
and neutron ( )A is called atomic mass.
Isotopes : When atoms of same element have same number of protons but different
number of neutrons is called isotopes.
Isobars : When atoms of different element have same atomic mass but different atomic
number, such atoms are called isobars.
Valency : It is the combining capacity of an atom or it is defined as number of electrons lost
or gained by an atom to acquire noble gas configuration.

Number of electrons, protons and neutrons in chemical species A, B, C and D is given below
:
Elements Electrons Protons Neutrons
A 2 3 4
B 10 9 8
C 8 8 8
D 8 8 10
Now, answer the following questions :
(a) What is the mass number of A and B ?
(b) What is the atomic number of B ?
(c) Which two elements represent a pair of isotopes and why ?
(d) What is the valency of element C ?
Also, justify your answers.
Ans :
(a) Mass number of A = 3 + 4 = 7
Mass number of B = 9 + 8 = 17
(b) Atomic number of B = Number of protons = 9
(c) Elements C and D represent a pair of isotopes because their atomic numbers are the
same.
(d) Electronic configuration of C (8) = 2, 6. So, its valency is 2.
35. What are three main categories of connective tissue? ns :
Categories of connective tissue are following : Connective tissue proper : There is a matrix in
which generally two types of (white and yellow) fibres are present. In between these fibres
some connective tissue cells are present. Example of this kind of connective tissues are
areolar tissue and adipose tissue.
Skeletal tissue : This type of tissues form the skeleton of an organism. It is of two types :
Cartilage and bone.
(i) Cartilage has solid matrix called chondrin, in which fibres and cells known as
chondrocytes are present. Usually cells are present in clusters of 2-3 cells in small spaces
called lacunae. Cartilage is found in the regions of pinna, nose, trachea and larynx.
(ii) In bones, matrix is formed of a protein called ossein impregnated with phosphate
and carbonates of calcium and magnesium.
Fluid tissue : Blood and lymph are examples of fluid connective tissues. These are
specialized connective tissues. It consists of liquid matrix with no fibres. In liquid matrix
called plasma corpuscles remain suspended. Blood transports food material, gases and
other substances to the various parts of the body.

Explain the structure of a fluid connective tissue.


Ans :
Blood is a fluid connective tissue.
Blood consists of : 1. Blood plasma, 2. Blood cells.
1. Blood plasma : It is the fluid matrix which contains 85 to 95% water, 7% different
types of proteins, 0.9% of salts, about 0.1% glucose and a very small amount of hormones,
wastes, etc. In the plasma, blood corpuscles (cells) are suspended.
2. Blood cells : Three kinds of blood cells are found suspended in the blood plasma.
These are : (i) Red blood corpuscles (Erythrocytes) or RBCs (ii) White blood corpuscles
(leucocytes) or WBCs and (iii) Blood platelets.
(i) Red blood corpuscles (Erythrocytes) or RBCs : The red blood corpuscles are
biconcave, disc-like cells which are devoid of nucleus. They contain a substance called
haemoglobin because of this they appear red in colour. The most important function of the
RBCs is the transport of oxygen and carbon dioxide.
(ii) White blood corpuscles (Leucocytes) or WBCs : These cells are comparatively large in
size, colourless and irregular in appearance. They are devoid of haemoglobin. They protect
our body from diseases by destroying germs.
(iii) Blood platelets : These are small, 2-4 µ in diameter. They are without nucleus. Their
main function is to liberate some substances which helps in blood clotting.
36. The following is the distance-time table of an object in motion :
Time (in seconds) Distance (in metres)
0 0
1 1
2 8
3 27
4 64
5 125
6 216
7 343
(a) What conclusion can you draw about the acceleration? Is it constant, increasing,
decreasing, or zero?
(b) What do you infer about the forces acting on the object?
Ans :
(a) There is an unequal change of distance in an equal interval of time. So, the given
objects have a non-uniform motion. Thus, the acceleration is increasing as the velocity of
the object increases with time.
(b) According to Newton’s second law of motion, the force acting on an object is directly
proportional to the acceleration produced in the object. In the given case, the increasing
acceleration of the given object indicates that the force acting on the object is also
increasing.
SECTION-E
Question no. 37 to 39 are case-based/data-based questions with 2 to 3 short sub-parts.
Internal choice is provided in one of these sub-parts.
37. Rutherford (1871-1937) was known as the ‘Father’ of nuclear physics. He is famous for
his work on radioactivity and the discovery of the nucleus of an atom with the gold foil
experiment. Ernest Rutherford was interested in knowing how the electrons are arranged
within an atom. Rutherford designed an experiment for this. In this experiment, fast moving
alpha α -particles were made to fall on a thin gold foil. On the basis of his experiment,
Rutherford put forward the nuclear model of an atom, which had the following features:
• There is a positively charged centre in an atom called the nucleus. Nearly all the
mass of an atom resides in the nucleus.
• The electrons revolve around the nucleus in circular paths.
• The size of the nucleus is very small as compared to the size of the atom.
Drawbacks of Rutherford’s model of the atom: The revolution of the electron in a circular
orbit is not expected to be stable. Any particle in a circular orbit would undergo
acceleration. During acceleration, charged particles would radiate energy. Thus, the
revolving electron would lose energy and finally fall into the nucleus. If this were so, the
atom should be highly unstable and hence matter would not exist in the form that we know.
We know that atoms are quite stable.
(a) Who is the father of nuclear physics?
(b) What is the name of positively charge center in an atom?
(c) Write the features of Rutherford’s nuclear model of an atom?
or
(c) What do you mean by nucleus?
ns : (a) E. Rutherford
(b) Nucleus
(c) The feature of Rutherford nuclear model are as follows:
1. There is a positively charged centre in an atom called the nucleus. Nearly all the
mass of an atom resides in the nucleus.
2. The electrons revolve around the nucleus in circular paths.
3. The size of the nucleus is very small as compared to the size of the atom. 39.
or
(c) There is a positively charged centre in an atom called the nucleus. Nearly all the mass of
an atom resides in the nucleus.
38. Blood is a type of connective tissue. The cells of connective tissue are loosely spaced and
embedded in an inter-cellular matrix. The matrix may be jelly like, fluid, dense or rigid. The
nature of matrix differs in concordance with the function of the particular connective tissue.
Blood has a fluid (liquid) matrix called plasma, in which red blood corpuscles (RBCs), white
blood corpuscles (WBCs) and platelets are suspended. The plasma contains proteins, salts
and hormones. Blood flows and transports gases, digested food, hormones and waste
materials to different parts of the body. Bone is another example of a connective tissue. It
forms the framework that supports the body. It also anchors the muscles and supports the
main organs of the body. It is a strong and non-flexible tissue. Bone cells are embedded in a
hard matrix that is composed of calcium and phosphorus compounds. Two bones can be
connected to each other by another type of connective tissue called the ligament. This
tissue is very elastic. Another type of connective tissue, cartilage, has widely spaced cells.
The solid matrix is composed of proteins and sugars. Cartilage smoothness bone surfaces at
joints and is also present in the nose, ear, trachea and larynx.
(a) Where are connective tissues located?
(b) Is areolar tissue is found between skin and muscles?
(c) Which of the tissues connect two bones?
or
(c) What are the function of connective tissue?
Ans :
(a) Connective tissue is found in between other tissues everywhere in the body,
including the nervous system.
(b) Yes, Areolar connective tissue is found between skin and muscles.
(c) A ligament is the fibrous connective tissue that connects two bones to form a joint.
These are all found in the skeletal system of the human body.
or
(c) Functions of connective tissue are as follows:
1. Binding together other tissues
2. Supporting various parts of the body
3. Forming a packing around organs
Newton’s first law of motion states that a body at rest will remain at rest position only and a
body which is in motion continues to be in motion unless otherwise they are acted upon by
an external force. In other words, all objects resist a change in their state of motion. In a
qualitative way, the tendency of undisturbed objects to stay at rest or to keep moving with
the same velocity is called inertia. This is why, the first law of motion is also known as the
law of inertia.
(a) What is the another name of first law of motion?
(b) What will happen if there is an absence of external force?
(c) State Newton’s first law of motion.
or
(c) Why Newton’s first law of motion is called law of inertia?
Ans :
(a) Law of inertia
(a) In the absence of any external force, an object at rest will stay at rest , and a moving
object will keep on moving with constant (unchanged) speed in a straight line.
(b) Newton’s first law of motion states that a body at rest will remain at rest position
only and a body which is in motion continues to be in motion unless otherwise they are
acted upon by an external force.

(c) All objects resist a change in their state of motion. In a qualitative way, the tendency
of undisturbed objects to stay at rest or to keep moving with the same velocity is called
inertia. This is why, the first law of motion is also known as the law of inertia.
KENDRIYA VIDYALAYA SANGATHAN:::RANCHI REGION
SAMPLE PAPER (HOTS)

Science (086)
Time: 3 Hours Max. Marks: 80
General Instructions:
1. This question paper consists of 39 questions in 5 sections.
2. All questions are compulsory. However, an internal choice is provided in some
questions. A student is expected to attempt only one of these questions.
3. Section A consists of 20 Objective Type questions carrying 1 mark each.
4. Section B consists of 6 Very Short questions carrying 02 marks each. Answers to these
questions should in the range of 30 to 50 words.
5. Section C consists of 7 Short Answer type questions carrying 03 marks each. Answers
to these questions should in the range of 50 to 80 words.
6. Section D consists of 3 Long Answer type questions carrying 05 marks each. Answer to
these questions should be in the range of 80 to 120 words.
7. Section E consists of 3 source-based/case-based units of assessment of 04 marks each
with sub-parts.

SECTION-A

Select and write one most appropriate option out of the four options given for each of the
questions 1 – 20.

1. The diagrams show the arrangement of particles of a substance at temperatures 20cC


and 40 0C.
What are the likely melting and boiling points of the substance?

Melting point/ 0C Boiling point/ 0C


(a) -12 35
(b) -25 45
(c) -98 100
(d) 44 80
2. Select the incorrect statements(s).
1. Although ice, water and water vapour all look different and display different
physical properties, they are chemically the same.
2. During burning of a candle, both physical and chemical changes take place.
3. Both water and cooking oil are liquid but their chemical characteristics are
different. They differ in odour and inflammability.
4. It is the physical property of oil that makes it different from water.
(a) 1 and 2. (b) 2 and 3
(c) 1, 2 and 3 (d) Only 4

3. Which of these is/are conserved during a chemical reaction?


(a) mass only (b) charge only

(c) both mass and charge (d) neither mass nor charge

4. We know that like charges repel each other. Then how do the protons, which are all
positively charged, stay together in an atom’s nucleus?
(a) The neutral charge of the neutron keeps them together.

(b) Nuclei keep decaying in short intervals because of this.

(c) The nucleic force is stronger than their mutual repulsion.

(d) That like charges repel is not true at the level of the nucleus.

5. Plasma membrane is composed of


(a) cellulose and lipids (b) lipids and proteins (c) peptidoglycan and lipids

(d) cellulose and proteins

6. Which of the following protects the animal cell from the outside environment?
(a) Cell wall (b) Plasma membrane

(c) Nuclear membrane (d) Cytoplasm


7. Which of the following options is correct for the object having a straight line motion
represented by the following graph?

(a) The object moves with constantly increasing velocity from O to A and then it moves
with constant velocity.

(b) Velocity of the object increases uniformly.

(c) Average velocity is zero.

(d) The graph shown is impossible.

8. A hockey player pushes the ball on the ground. It comes to rest after travelling certain
distance because
(a) the player stops pushing the ball. (b) balanced force acts on the ball.

(c) the opposing force acts on the ball. (d) none of these

9. Which of the following statements is/are correct?


1. Mass of an object is the measure of its inertia.
2. Heavier the object smaller is the inertia.
3. The mass of an object is variable.
(a) Only 1 (b) 1 and 3

(c) 2 and 3 (d) 1 and 2


10. A cricket ball is projected vertically upward such that it returns back to the thrower. The
variation in kinetic energy with time is best represented by
11. The sound waves having a frequency more than 20,000 Hz are called
(a) infrasonic waves (b) supersonic waves (c) ultrasonic waves (d)

hypersonic waves

12. A waxy, water resistant layer is observed in the xerophytic plants. What is the layer
called as?
(a) Endodermis (b) Cortex

(c) Phloem (d) Epidermis

13. If the component of the substance can be separated by a chemical change only then it
is
(a) element (b) compound

(c) mixture (d) both (a) and (b)

14. The symbol of gold is


(a) Ca (b) Au

(c) Ag (d) Cu

15. A cell loses water by osmosis when kept in a solution having a lower concentration of
water than the cell. The given solution is
(a) hyper-tonic (b) hypo-tonic

(c) isotonic (d) dilute


16. Which component of sclerenchyma tissues harden and thicken their secondary walls?
(a) Suberin (b) Calcium

(c) Lignin (d) Magnesium

Question no. 17 to 20 are Assertion-Reasoning based questions.

17. Assertion : Motion with uniform velocity is always along a straight line path.
Reason : In uniform velocity a motion, speed is the magnitude of the velocity and is
equal to the instantaneous velocity.
(a) Both assertion and reason are true and reason is the correct explanation of
assertion.

(b) Both assertion and reason are true but reason is not the correct explanation of
assertion.

(c) Assertion is true but reason is false.

(d) Both assertion and reason are false.

18. Assertion : Linear momentum is conserved in both elastic and inelastic collisions.
Reason : Total energy is conserved in all collisions.
(a) Both assertion and reason are true and reason is the correct explanation of
assertion.

(b) Both assertion and reason are true but reason is not the correct explanation of
assertion.

(c) Assertion is true but reason is false.

(d) Both assertion and reason are false.

19. Assertion : A man is sitting in a boat which floats on a pond. If the man drinks some
water from the pond, the level of water in the pond will decrease.
Reason : The weight of the liquid displaced by the body is greater than
the weight of the body. (a) Both assertion and reason are true and
reason is the correct explanation of assertion.

(b) Both assertion and reason are true but reason is not the correct explanation of
assertion.

(c) Assertion is true but reason is false.

(d) Both assertion and reason are false.

20. Assertion : A spring has potential energy, both when it is compressed or stretched.
Reason : In compressing or stretching, work is done on the spring
against the restoring force. (a) Both assertion and reason are true and
reason is the correct explanation of assertion.

(b) Both assertion and reason are true but reason is not the correct explanation of
assertion.

(c) Assertion is true but reason is false.

(d) Assertion is false but reason is true.


SECTION-B
Question no. 21 to 26 are very short answer questions.

21. Write down four properties of a Solution.


or
State the principle of the process of centrifugation.

22. Why the number of atoms in one mole of hydrogen gas is double the number of atoms
in one mole of helium gas? Explain.

23. Who discovered cells in living organisms? Give an example of unicellular organism.

24. A ball moving on a table reaches the edge and falls. Sketch the path it will follow while
falling.

25. What is the range of frequencies associated with :


(a) Infrasound
(b) Ultrasound
or
What are longitudinal waves? Give two examples.

26. What are the three advantages of shorter duration of the crop in between sowing and
harvesting?

SECTION-C
Question no. 27 to 33 are short answer questions.

27. A karate expert can easily move his hand through a solid block of wood but we cannot.
Why ?

28. Write characteristics of compounds.


29. What would happen if when we put an animal cell into a solution of sugar or salt in
water?
o
What is the function of plastids?

30. What are the four main functions of epithelial tissue?

31. In a long distance race, the athletes were expected to take four rounds of the track such
that the line of finish was same as the track was 200 m.
(i) What is the total distance to be covered by the athletes?
(ii) What is the displacement of the athletes when they touch the finish line?
(iii) Is the motion of the athletes uniform or non-uniform?
(iv) Is the distance moved by and displacement of athletes at the end of the race
equal?

32. (a) What is meant by potential energy? Is potential energy vector or scalar
quantity?
(b) Give one example of a body having potential energy.
o
Define : (a) power (b) work done (c) kinetic energy. Give SI unit of each.

33. Distinguish between transverse and longitudinal waves (three points).

SECTION-D
Question no. 34 to 36 are Long answer questions.

34. Explain Rutherford’s atomic model.


o
Define isotopes. Why do isotopes have same atomic number but different mass
number ? Explain with the help of an example.

35. Explain the structure of three types of muscle fibres. Also write the locations where they
are found in the body.
o
How many types of meristems are present in plants, on the basis of position?

36. Give brief sketch on advantages and disadvantages on manure and fertilizers.

SECTION-E
Question no. 37 to 39 are case-based/data-based questions with 2 to 3 short sub-parts.
Internal choice is provided in one of these sub-parts.
37. The nucleus of atom contains positively charged particles called protons and neutral
particles called neutrons. The number of protons in an atom is called the atomic number
and is denoted by the symbol ‘Z’. All atoms of an element have the same atomic number.
The electrons occupy the space outside the nucleus. In order to account for the
electrically neutral nature of the atom, the number of protons in the nucleus is exactly
equal to the number of electrons.
Thus, Atomic number = Number of protons = Number of electrons
You would remember that according to Dalton’s theory, the atoms of different
elements are different from each other. We can now say that this difference is due to
difference in the numbers of protons present in the nucleus of the element. In other
words, different elements differ in terms of their atomic number.
For example, the atoms of hydrogen and helium are different because hydrogen has
one proton in its nucleus whereas the nucleus of helium atom contains two protons.
Their atomic numbers are 1 and 2, respectively. You have learnt in the Rutherford’s
model that the mass of the atom is concentrated in its nucleus. This is due to the
presence of two heavy particles namely protons and neutrons in the nucleus. These
particles are called nucleons. The number of nucleons in the nucleus of an atom is
called its mass number. It is denoted by ‘A’ and is equal to the total number of protons
and neutrons present in the nucleus of an element.
Thus, Mass number ( )A = Number of protons ( )Z + Number of neutrons ( )n Atomic
number and mass number are represented on the symbol of an element.
(i) The diagram shows the nucleus of an atom of X. What is the X in the given diagram?

(ii) The diagram given below shows the sub-atomic particles present in the nucleus of
atom X.

What is the symbol for atom X ?


(iii) The formula of a molecule is X2. One molecule of X2, contains 18 protons. If the
nucleon number of X is
19, how many neutrons are there in one atom of X ?
or
(iii) Find the number of neutrons in 31X15 ?
38. Fish is one of the easily available, highly affordable and nutritious food for people,
especially in coastal regions. Fish is a rich source of proteins. Fish liver oil is obtained
for vitamin A and vitamin D. The different type of fish obtained for food are :
(i) Agnatha (jawless fish)
(ii) Chondrichthyes (cartilagenous fish)
(iii) Osteichthyes (bony fish)
Other fish include Jelly fish (Aurelia), Star fish, shell fish (prawns and molluscs), cattle
fish etc.
The establishments which are associated with production, preservation, capture and
exploitation of fishes, crabs, lobsters, prawns etc. are called fisheries. Both true fish
(gill containing finned vertebrates which contain scales on their skin) and shelled fish
are produced in fisheries. Fisheries are of following types, based on mode of obtaining
fish.
(i) What do you mean by mariculture.?
(ii) What is the common name of Penaeus monodon?
(iii) Which of the marine fish of high economic value?
(iv) What is the correct function of the figure given below?

or
(iv) What is fresh water prawn called?
39. Newton’s first law of motion states that a body continues to remain in state of rest or of
uniform motion along a straight line unless an external unbalanced force acts on it.
Thus, a body cannot change its state of rest or motion unless compelled by an external
force. In other words, every body possesses a property by virtue of which it resists any
change in its state of rest or of uniform motion. This property is known as inertia of
that body.
Inertia is an inherent property or the tendency of a body to oppose any change in its
state of rest or of uniform motion in a particular direction. Thus, Newton’s first law of
motion is also called law of inertia as it defines inertia in its statement.
To change the state of rest or motion of the body, force is required. If greater force is
required to do so, it means the body has more inertia. Since more force is required to
change the state of heavier objects than light objects, we conclude that greater the
mass of a body, more is the inertia possessed by it. We can also state that mass is a
measure of inertia of the body.
(i) A hockey player pushes the ball on the ground. Why it comes to rest after travelling
certain distance?
(ii) What is velocity-time graph of a moving particle on which net external force is
zero?
(iii) On what factor does the inertia of a body depend?
(iv) Which has more inertia, a cricket ball or rubber of the same size?
or
(iv) Does inertia change with velocity?
Sample Paper(HOTS)
Solutions Class IX 2022-23
Science (086)
Time: 3 Hours Max. Marks: 80
General Instructions:
1. This question paper consists of 39 questions in 5 sections.
2. All questions are compulsory. However, an internal choice is provided in some
questions. A student is expected to attempt only one of these questions.
3. Section A consists of 20 Objective Type questions carrying 1 mark each.
4. Section B consists of 6 Very Short questions carrying 02 marks each. Answers to these
questions should in the range of 30 to 50 words.
5. Section C consists of 7 Short Answer type questions carrying 03 marks each. Answers
to these questions should in the range of 50 to 80 words.
6. Section D consists of 3 Long Answer type questions carrying 05 marks each. Answer to these
questions should be in the range of 80 to 120 words.
7. Section E consists of 3 source-based/case-based units of assessment of 04 marks each
with sub-parts
four options given for each of the questions 1 – 20.

1. The diagrams show the arrangement of particles of a substance at temperatures 200C


and 400C.

What are the likely melting and boiling points of the substance?

Melting point/ 0C Boiling point/0C


(a) -12 35
(b) -25 45
(c) -98 100
(d) 44 80
Ans : (a)
At 200C, substance is a liquid. At 400C, substance is a gas. Thus, its melting point is
below 200C and its boiling point is below 400C.

2. Select the incorrect statements(s).


1. Although ice, water and water vapour all look different and display different
physical properties, they are chemically the same.
During burning of a candle, both physical and chemical changes take place.
2. Both water and cooking oil are liquid but their chemical characteristics are different.
They differ in odour and inflammability.
3. It is the physical property of oil that makes it different from water.
(a) 1 and 2 (b) 2 and 268
(c) 1, 2 and 3 (d) Only 268
Ans : (d) OnlyIt is the chemical property of oil that makes it different from water.

3. Which of these is/are conserved during a chemical reaction?


(a) mass only
(b) charge only
(c) both mass and charge
(d) neither mass nor charge
Ans : (c) both mass and charge
According to the law of conservation of mass, mass can neither be created nor be
destroyed in a chemical reaction. It is always conserved.
The law of conservation of charge says that the net charge of an isolated system
will always remain constant.

4. We know that like charges repel each other. Then how do the protons, which are all
positively charged, stay together in an atom’s nucleus?
(e) The neutral charge of the neutron keeps them together.
(f) Nuclei keep decaying in short intervals because of this.
(g) The nucleic force is stronger than their mutual repulsion.
(h) That like charges repel is not true at the level of the nucleus.
Ans : (c) The nucleic force is stronger than their mutual repulsion. 7.
The strong nuclear force is responsible for holding together the nucleus. At the
distance scale of a proton (about 1 femtometre, or 10-15 metres), the strong
nuclear force is about 137 times stronger than the electromagnetic force.
Therefore, the repulsive force due to both the protons being electrically positive
is not strong enough to break the nucleus apart.

5. Plasma membrane is composed of

(i) cellulose and lipids


(j) lipids and proteins
(k) peptidoglycan and lipids
(l) cellulose and proteins
Ans : (b) lipids and proteins
Plasma membrane is a living, thin, delicate, elastic, selectively permeable. Chemically, it is
made up of 75% phospholipid. In addition to phospholipid, the membrane contains proteins,
cholesterol and polysaccharides.

6. Which of the following protects the animal cell from the outside environment? (a) Cell
wall
(b) Plasma membrane
(c) Nuclear membrane (d) Cytoplasm
Ans : (b) Plasma membrane
Plasma membrane found in all cells that separates, the interior of the cell from the outside
environment.
7. Which of the following options is correct for the object having a straight line motion
represented by the following graph?

(a) The object moves with constantly increasing velocity from O to A and then it moves with
constant velocity.
(b) Velocity of the object increases uniformly.
(c) Average velocity is zero.
(d) The graph shown is impossible.
Ans : (c) Average velocity is zero. From given, it is clear that the net displacement is zero. So,
average velocity will also be zero.
8. A hockey player pushes the ball on the ground. It comes to rest after travelling certain
distance because
(a) the player stops pushing the ball.
(b) balanced force acts on the ball.
(c) the opposing force acts on the ball. (d) none of these
Ans : (c) the opposing force acts on the ball.
The opposing force of friction acts on the ball due to which ball comes to rest after
travelling certain distance.

9. Which of the following statements is/are correct?


1. Mass of an object is the measure of its inertia.
2. Heavier the object smaller is the inertia.
3. The mass of an object is variable.
(d) Only 1 (b) 1 and 3
(c) 2 and 3 (d) 1 and 2
Ans : (a) Only 1
Heavier the object, greater is the inertia. The mass of an object is constant.

10. A cricket ball is projected vertically upward such that it returns back to the thrower. The
variation in kinetic energy with time is best represented by

Ans : (a)
When the ball rises up its kinetic energy decreases, till it becomes zero. On falling down
its kinetic energy increases. Therefore option (a) is correct.

11. The sound waves having a frequency more than


20,000 Hz are called
(e) infrasonic waves (b) supersonic waves
(c) ultrasonic waves (d) hypersonic waves
Ans : (c) ultrasonic waves
Waves of frequency above 20000 Hz cannot be heard by human beings and are called
ultrasonic.

12. A waxy, water resistant layer is observed in the xerophytic plants. What is the layer called
as?
(f) Endodermis (b) Cortex
(c) Phloem (d) Epidermis
Ans : (d) Epidermis
A xerophyte is an organism, which is able to survive in an ecosystem with little or no
water (or moisture). To reduce transpiration the epidermal cells of xerophytes (plants)
secrete a waxy (fatty) water resistant layer of cutin.

13. If the component of the substance can be separated by a chemical change only then it is
(g) element (b) compound
(c) mixture (d) both (a) and (b)
Ans : (b) compound
A compound can be separated into its components by chemical means while an
element cannot be further separated. A mixture can be separated by physical means.

22. 14. The symbol of gold is


(a) Ca (b) Au

(c) Ag (d) Cu

Ans : (b) Au [ Derived from the name Aurum]

15. A cell loses water by osmosis when kept in a solution having a lower concentration of water
than the cell.
The given solution is
(h) hyper-tonic (b) hypo-tonic (c) isotonic (d) dilute
Ans : (a) hyper-tonic
If a cell is a placed in a hyper-tonic solution which has higher concentration of solute
and lower concentration of water as compared to the concentration of cell sap, the
water molecules move from cell sap to the external solution so that the cell shrinks.

16. Which component of sclerenchyma tissues harden and thicken their secondary walls?
(i) Suberin (b) Calcium
(c) Lignin (d) Magnesium
Ans : (c) Lignin
Sclerenchyma consists of thick, hard secondary walls due to deposition of lignin. It
provides mechanical strength to the cells.

Question no. 17 to 20 are Assertion-Reasoning based questions.

17. Assertion : Motion with uniform velocity is always along a straight line path.
Reason : In uniform velocity a motion, speed is the magnitude of the velocity and is
equal to the instantaneous velocity.
(j) Both assertion and reason are true and reason is the correct explanation of
assertion.
(k) Both assertion and reason are true but reason is not the correct explanation of
assertion.
(l) Assertion is true but reason is false. (d) Both assertion and reason are false.
Ans : (b) Both assertion and reason are true but reason is not the correct explanation
of assertion.
The assertion is true and the reason is not the correct explanation of the assertion.
Uniform velocity means that speed and direction remain unchanged.

18. Assertion : Linear momentum is conserved in both elastic and inelastic collisions.
Reason : Total energy is conserved in all collisions.
(m) Both assertion and reason are true and reason is the correct explanation of
assertion.
(n) Both assertion and reason are true but reason is not the correct explanation of
assertion.
(o) Assertion is true but reason is false. (d) Both assertion and reason are false.
Ans : (b) Both assertion and reason are true but reason is not the correct explanation
of assertion.
Both linear momentum and kinetic energy are conserved in elastic collisions. Linear
momentum is conserved in inelastic collisions but kinetic energy is not conserved. The
total energy is conserved in all collisions.
19. Assertion : A man is sitting in a boat which floats on a pond. If the man drinks some
water from the pond, the level of water in the pond will decrease.
Reason : The weight of the liquid displaced by the body is greater than the weight of the
body.
(p) Both assertion and reason are true and reason is the correct explanation of
assertion.
(q) Both assertion and reason are true but reason is not the correct explanation of
assertion.
(r) Assertion is true but reason is false. (d) Both assertion and reason are false.
Ans : (d) Both assertion and reason are false.
Water level in the pond remains unchanged. When the man drinks water from the
pond, a larger volume of water is displaced by the boat and hence or original water
level is maintained.
20. Assertion : A spring has potential energy, both when it is compressed or stretched.
Reason : In compressing or stretching, work is done on the spring against the restoring
force.
(s) Both assertion and reason are true and reason is the correct explanation of
assertion.
(t) Both assertion and reason are true but reason is not the correct explanation of
assertion.
(u) Assertion is true but reason is false.
(v) Assertion is false but reason is true.
Ans : (a) Both assertion and reason are true and reason is the correct explanation of
assertion.
The work done on the spring against the restoring force is stored as potential energy in
both cases, when it is compressed or stretched.

SECTION-B
Question no. 21 to 26 are very short answer questions.

21. Write down four properties of a Solution.


Ans :
The four properties of solution are as follows:
(i) It is a homogeneous mixture of solute and solvent.
(ii) Solute particles cannot be separated by filtration.
(iii) True solution is clear and transparent.
(iv) True solution does not scatter light.
or
State the principle of the process of centrifugation.
Ans :
Centrifugation is the process of separating suspended particles from a liquid like
colloids by churning the liquid at a high speed. The principle is that denser particles are
forced to the bottom and lighter stay at the top when spun rapidly.

22. Why the number of atoms in one mole of hydrogen gas is double the number of
atoms in one mole of helium gas ? Explain.
Ans :
The number of atoms in one mole of hydrogen gas is double the number of atoms in
one mole of helium gas because hydrogen molecule is diatomic, i.e., a molecule of
hydrogen consists of two atoms of hydrogen, whereas helium is monatomic.
23. Who discovered cells in living organisms? Give an example of unicellular organism.
Ans :
Leeuwenhoek (1674) was the first to observe the free living cells in pond water.
Example of unicellular organisms : Amoeba, Chlamydomonas, Paramoecium, Bacteria,
etc.

24. A ball moving on a table reaches the edge and falls. Sketch the path it will follow
while falling.
Ans :
As the ball falls, it has a horizontal velocity and a vertical downward acceleration due to
gravity. Under the combined effect of these two motion the ball moves along a
parabolic trajectory as shown in figure

25. What is the range of frequencies associated with :


(a) Infrasound (b) Ultrasound ns :
(a) Infrasound : Sound waves between the frequencies 1 to 20 Hz.
(b) Ultrasound : Sound waves of the frequencies above 20,000 Hz.
o
What are longitudinal waves? Give two examples.
Ans :
A wave in which the particles of the medium vibrate back and forth along the same
direction, in which the wave is moving, is called a longitudinal wave.
Examples :
(a) The sound waves in air.
(b) The waves produced in air when a sitar wire is plucked.
26. What are the three advantages of shorter duration of the crop in between sowing and
harvesting?
Ans :
Short durations allow farmers to grow multiple rounds of crops in a year. Short
duration also reduces the cost of crop production. Uniform maturity makes the
harvesting process easy and reduces losses during harvesting.
SECTION-C
Question no. 27 to 33 are short answer questions.

27. A karate expert can easily move his hand through a solid block of wood but we
cannot. Why ?
Ans :
In a solid block of wood, the inter-particle forces are very strong and hence, it is not
easy to separate the particles. Therefore, it is not easy to move our hand through a
solid block of wood, only a karate expert can do it as he has expertise in this.
28. Write characteristics of compounds.
Ans :
Characteristics of compounds are as follows :
a. Compounds are the substances formed by chemical combination of two or more
elements.
b. The constituent elements are present in a fixed ratio.
c. A chemical reaction takes place during the formation of a compound.
d. Properties of a compound are different to those of its elements.
Constituent elements cannot be separated by physical processes.
29. What would happen if when we put an animal cell into a solution of sugar or salt in water?
Ans :The following three things could happen :
a. If the solution surrounding the cell is very dilute than cytoplasm, the water will move
into the cell, i.e., the cell will gain water.
b. If the solution has exactly similar water concentration as that of cytoplasm of
cell, there will be no net movement of water across the cell membrane, i.e., no gain
or loss of water from the cell.
c. If the medium (solution) has a lower concentration of water than the cell, i.e.,
the solution is concentrated, the cell will lose water by osmosis.
o
What is the function of plastids?
Ans :
Plastids are present only in plant cells. There are two types of plastids chromoplasts
(coloured plastids) and leucoplasts (white or colourless). Chromoplast : Consists of
coloured pigments and given different colours to flowers, fruits and leaves. The green
colour pigment present in leaf is called chlorophyll which helps in the photosynthesis
and a plastid with chlorophyll is called chloroplast.
Leucoplast : It stores starch, oil and protein granules in it.
30. What are the four main functions of epithelial tissue?
Ans :
The main functions of epithelial tissue are :
(i) It forms the outer layer of skin and hence it protects the underlying cells from drying,
injury, bacterial and chemical effects.
(ii) It forms lining of mouth, alimentary canal and other internal organs and thus
protects these organs.
(iii) It helps in absorption of water and other nutrients in alimentary canal. Some of them
are greatly specialized and perform secretary function.

31. In a long distance race, the athletes were expected to take four rounds of the track
such that the line of finish was same as the track was 200 m.
(iv) What is the total distance to be covered by the athletes?
(v) What is the displacement of the athletes when they touch the finish line?
(vi) Is the motion of the athletes uniform or nonuniform?
(vii) Is the distance moved by and displacement of athletes at the end of the race equal?
Ans :
(i) Total distance covered by the athletes = 4 × 200 = 800 m.
(ii) The line of start and the line of finish are the same so, Displacement = 0
(iii) The motion of the athletes is non-uniform.
(iv) The distance and displacement of an athlete at the end of the race are not equal.

32. (a) What is meant by potential energy? Is potential energy vector or scalar quantity?
(b) Give one example of a body having potential energy.
Ans :
(a) The energy possessed by a body by virtue of its position or configuration. It is a scalar
quantity.
(b) Stretched string of a bow.

or
Define : (a) power (b) work done (c) kinetic energy. Give SI unit of each.
Ans :
(a) The rate of doing work is called power. Its SI unit is watt.
(b) Work is the product of force and displacement. Its SI unit is joule.
It is the energy possessed by a body by virtue of its motion. Its SI unit is joule.
33. Distinguish between transverse and 35. Explain the structure of three types of
longitudinal waves (three points). muscle fibres. Also write the locations where
Ans : they are found in the body.
Ans : The followings are the three types of muscle cells
:
Transverse waves Longitudinal waves
1. Unstriated muscles (also known as smooth,
1. Particles involuntary muscles) : This type of muscular tissue
Particles the vibrate consists of spindle-shaped, long uninucleated cells. This
medium vibrate at parallel to the type of muscles are present in alimentary canal, blood
vessels, iris of eye, in ureters and bronchi of lungs, etc.
right angles. direction of waves.
2. Striated muscles (also known as voluntary
2. Alternate crests Alternate
and troughs compressions,
formed. rarefaction formed.
3. e.g., water waves. e.g., sound waves.
muscles because of their function being in
our control or will) : This type of muscular
cells are long multi-nucleated and enclosed in
a membrane called sarcolemma. Each fibre
SECTION-D has several longitudinal filaments embedded
Question no. 34 to 36 are Long answer questions. in cytoplasm. These filaments give these
muscles striated appearance. These muscles
34. Explain Rutherford’s atomic model. are attached to the skeleton; so they are
called skeletal muscles.
Ans :
3. Cardiac muscles : These muscles are found
Rutherford purposed a model of an atom on in heart. They are not under the control
the basis of α -particles scattering of the will. They contract rhythmically
experiment. This is known as Rutherford’s and involuntarily throughout life without
nuclear model of atom. the sign of fatigue. Structurally they show
(i) An atom consist, a heavy positively charged the characters of both unstriated and
core called nucleus. striated muscles. They are made up of
(ii) Nucleus is surrounded by electrons. branched fibres. These fibres are
(iii) Electrons and nucleus are held together by uninucleated and show alternate light
electrostatic force of attraction. and dark bands (striation).
(iv) Size of nucleus is very small as compared to or
the size of atom. How many types of meristems are present in
(v) Almost the entire mass of the atom is plants, on the basis of position?
concentrated in the nucleus.
Ans :
or
Define isotopes. Why do isotopes have same On the basis of location of meristem, it is
atomic number but different mass number ? classified into three types :
Explain with the help of an example. 1. Apical meristem is present at the tip of
stem, roots and their branches.
Ans :
2. Intercalary meristem is found at the leaf
Isotopes : Atoms of the same element, having base, above the nodes (i.e. at the base of
the same atomic number but different mass internodes as in grasses) or below the
numbers are called isotopes. nodes (i.e. at the uppermost region of
Isotopes have same atomic number but internode as in mint).
different mass number because they contain 3. Lateral meristem
different number of neutrons. 4. Vascular cambium and cork cambium are
For example : In nature chlorine occurs in two the examples of lateral meristem.
isotopic forms; 17Cl35 and 17Cl37. 5. Vascular cambium is found in vascular
Here atomic number of both the atoms is bundles while cork cambium is found
same but due to the difference in the number underneath the bark of trees. Both of these
of neutrons, their mass number is different. cause increase in girth of plants.
36. Give brief sketch on advantages and disadvantages on manure and fertilizers.
Ans :
disadvantages of manure :
(i) It increases the number of friendly microbes.
(ii) It improves the texture of soil by adding organic matter (humus).
(iii) It increases soil fertility, water holding capacity and aeration.
(iv) It reduces soil erosion.
(v) It is cheap.
Disadvantages of manure :
(i) They have fewer amounts of nutrients as compared to fertilizers.
(ii) Manures are bulky and not easy to store and transport.
Fertilizers : These are commercially manufactured inorganic salts containing one or
more essential plant nutrients like NPK, which are used to increase soil fertility.
disadvantages of fertilizers :
(i) They are nutrient specific and required in small amounts.
(ii) They are water soluble and absorbed by the plant easily.
(iii) They are easy to store and transport.
Disadvantages of fertilizers :
(i) Fertilizers can change the soil structure by killing the soil microbes.
(ii) Fertilizers can change the chemical composition of soil.
(iii) Accumulation of fertilizers in water bodies causes eutrophication.

SECTION-E
Question no. 37 to 39 are case-based/data-based questions with 2 to 3 short sub-parts.
Internal choice is provided in one of these sub-parts.

37. The nucleus of atom contains positively charged particles called protons and neutral
particles called neutrons. The number of protons in an atom is called the atomic number and
is denoted by the symbol ‘Z’. All atoms of an element have the same atomic number. The
electrons occupy the space outside the nucleus. In order to account for the electrically neutral
nature of the atom, the number of protons in the nucleus is exactly equal to the number of
electrons.
Thus, Atomic number = Number of protons = Number of electrons
You would remember that according to Dalton’s theory, the atoms of different elements are
different from each other. We can now say that this difference is due to difference in the
numbers of protons present in the nucleus of the element. In other words, different
elements differ in terms of their atomic number.
For example, the atoms of hydrogen and helium are different because hydrogen has one
proton in its nucleus whereas the nucleus of helium atom contains two protons. Their
atomic numbers are 1 and 2, respectively. You have learnt in the Rutherford’s model that the
mass of the atom is concentrated in its nucleus. This is due to the presence of two heavy
particles namely protons and neutrons in the nucleus. These particles are called nucleons.
The number of nucleons in the nucleus of an atom is called its mass number. It is denoted by
‘A’ and is equal to the total number of protons and neutrons present in the nucleus of an
element.
Thus, Mass number (A) = Number of protons (Z ) + Number of neutrons ( n)
Atomic number and mass number are represented on the symbol of an element.
(i) The diagram shows the nucleus of an atom of X. What is X in the given diagram?

(ii) The diagram given below shows the sub-atomic particles present in the nucleus of
atom X.

What is the symbol for atom X ?


(iii) The formula of a molecule is X2. One molecule of X2, contains 18 protons. If the nucleon
number of X is 19, how many neutrons are there in one atom of X ?
Or
(iii) Find the number of neutrons in 31X15 ?
Ans :
(i)
X has 3 protons and 4 neutrons.
Hence, atomic number of X is 3 and mass number of X is (3 + 4) = 7.
X is represented as 73X.
(ii) Nucleon number = Number of neutrons + Number of protons = 5 + 4 = 9

(iii) Number of protons in one X2 is 18.


Number of protons in X = 18 / 2 = 9
Nucleon number =p + n
19 = 9+n
n = 19 − 9 = 10
Or
(iii) 31X15 indicate that no. of proton
= 15 and mass number = 31
Mass number = No. of protons
+ No. of neutrons = 31
Number of neutrons
= 31 – number of protons
= 31 – 15 = 16

38. Fish is one of the easily available, highly affordable and nutritious food for people,
especially in coastal regions. Fish is a rich source of proteins. Fish liver oil is
obtained for vitamin A and vitamin D. The different types of fish obtained for
food are :
(i) Agnatha (jawless fish)
(ii) Chondrichthyes (cartilagenous fish)
(iii) Osteichthyes (bony fish)
Other fish include Jelly fish (Aurelia), Star fish, shell fish (prawns and molluscs),
cattle fish etc. The establishments which are associated with production,
preservation, capture and exploitation of fishes, crabs, lobsters, prawns etc. are
called fisheries. Both true fish (gill containing finned vertebrates which contain
scales on their skin) and shelled fish are produced in fisheries. Fisheries are of
following types, based on mode of obtaining fish.
(i) What do you mean by mariculture.?
(ii) What is the common name of Penaeus monodon?
(iii) Which of the marine fish of high economic value?
(iv) What is the correct function of the figure given below?

or
(iv) What is fresh water prawn called?
Ans :
(i) Culture of marine finned fish, shellfish and seaweeds is called mariculture.
(ii) giant tiger prawn
(iii) Mullet, Bhetki, Oyster
(iv) The given figure is of a honey extractor. It is used for extracting honey from the comb.
or
(iv) Macrobrachium rosenbergii.

39. Newton’s first law of motion states that a body continues to remain in state of rest or of
uniform motion along a straight line unless an external unbalanced force acts on it. Thus, a
body cannot change its state of rest or motion unless compelled by an external force. In
other words, everybody possesses a property by virtue of which it resists any change in its
state of rest or of uniform motion. This property is known as inertia of that body.
Inertia is an inherent property or the tendency of a body to oppose any change in its state of
rest or of uniform motion in a particular direction. Thus, Newton’s first law of motion is also
called law of inertia as it defines inertia in its statement. To change the state of rest or
motion of the body, force is required. If greater force is required to do so, it means the body
has more inertia. Since more force is required to change the state of heavier objects than
light objects, we conclude that greater the mass of a body, more is the inertia possessed by
it. We can also state that mass is a measure of inertia of the body.
(i) A hockey player pushes the ball on the ground. Why it comes to rest
after travelling certain distance?
(ii) What is velocity-time graph of a moving particle on which net external
force is zero?
(iii) On what factor does the inertia of a body depend?
(iv) Which has more inertia, a cricket ball or rubber of the same size?
or
(iv) Does inertia change with velocity?
Ans :
(i) the opposing force acts on the ball.
(ii) Net external force is zero, it means, particle is moving with
constant velocity. Velocity-time graph of such a particle is
represented by a straight line parallel to time-axis.

(iii) The inertia of a body depends on its mass.


(iv) A cricket ball has more inertia than a rubber ball of the same
size because it has more mass than the rubber ball.
Or
(iv) Inertia doesn’t depend on velocity. Inertia is a tendency of an object to stay in motion or
rest unless external forces are applied.
KENDRIYA VIDYLAYA SANGATHAN::: RANCHI REGION

BLUE PRINT FOR MLL CLASS- 9

Chapter 1 marks 2 3marks(SA) 4 5 Total


(MCQS) marks(VSA) marks marks(LA)
+ (SA)
Assertion
Reason
Matter in our 1(1) 2(4) 3(5)
surrounding
Matter 2(2) 1(4) 3(6)
around us
pure
Atoms and 2(2) 1(5) 3(7)
Molecules
Structure of 1(1) 1(2) 1(3) 4(7)
Atom +1(1)
The 3(3) 1(2) 1(3) OR 1(5) 6(13)
fundamental
Unit of life
Tissues 3(3) 1(2) 1(3) 6(9)
+1(1)
Motion 1(1) 1(5) 2(6)
Force and 1(1) 2(4) 4(6)
laws of +1(1)
motion
Gravitation 1(2) 1(3) OR 2(5)
Work and 1(1) 1(4) 2(5)
energy
Sound 1(1) 1(3) 3(5)
+1(1)
KENDRIYA VIDYLAYA SANGATHAN::: RANCHI REGION

The 1(2) 1(4) 2(6)


improvements
in food
resources
Total 20(20) 9(18) 5(15) 3(12) 3(15) 40(80)
KENDRIYA VIDYLAYA SANGATHAN::: RANCHI REGION

KENDRIYA VIDYALAYA SANGATHAN RANCHI REGION


SAMPLE PAPER (086)-IX
(MLL)
Max. Marks: 80 SUBJECT SCIENCE Time Allowed: 3 hours
General Instructions:
i. This question paper consists of 40 questions in 5 sections.
ii. .All questions are compulsory. However, an internal choice is provided in some questions.
A student is expected to attempt only one of these questions.
iii. Section A consists of 20 objective type questions carrying 1 mark each.
iv. Section B consists of 9 Very Short questions carrying 02 marks each. Answers to these
questions should in the range of 30 to 50 words.
v. Section C consists of 5 Short Answer type questions carrying 03 marks each. Answers
to these questions should in the range of 50 to 80 words
vi. Section D consists of 3 Long Answer type questions carrying 05 marks each. Answer
to these questions should be in the range of 80 to 120 words.
Section E consists of 3 source-based/case-based units of assessment of 04 marks each with
sub- parts.

SECTION - A
Select and write one most appropriate option out of the four options given for each of the

Q. No Questions Marks
1 The mass per unit volume of a substance is called 1
(a) Pressure
(b) Force
(c) Velocity
(d) Density

2 Brass is a mixture of 1
(a) 30% Copper and 70% Zinc
(b) 70% Copper and 30% Zinc
(c) 40% Zinc and 60% Copper
(d) 60% Zinc and 40% Copper
KENDRIYA VIDYLAYA SANGATHAN::: RANCHI REGION

3 1

In the given figure (a) is copper sulphate solution and (b) is mixture of water and milk.
According to you which solution shows the Tyndall effect.?
(a) Copper sulphate solution
(b) Mixture of water and milk
(c) Both of these
(d) None of these
4 What is the Atomicity of Phosphorus 1
(a) Monoatomic
(b) Diatomic
(c) Triatomic
(d) Tetra atomic
5 Earnest Rutherford designed an experiment for arrangements of electron within an atom. 1
He uses which metal
(a) Iron
(b) Sodium
(c) Gold
(d) Silver
6 The ammonia is formed by the combination of nitrogen and hydrogen in the ratio by 1
mass
(a) 12:1
(b) 14:3
(c) 12:3
(d) 14:1

7 The cell theory was given by 1


(a) Leeuwenhoek
(b) Robert Hooke
(c) Virchow
(d) Schleiden and Schwann
8 The________ helps in the manufacture of fat molecules, or lipids, important for cell 1
function.

(a) Plasma membrane


(b) Rough endoplasmic reticulum
(c) Smooth endoplasmic reticulum
(d) Cytoplasm
KENDRIYA VIDYLAYA SANGATHAN::: RANCHI REGION

9 1
The full form of ATP is
(a) Artificial Thin Pore
(b) Atal Tinkering Process
(c) Adenosine Tri Phosphate
(d) Adenosine Tetra Phosphate

10 What type of Tissue will develop from jar1 to jar 2 as shown in the figure 1

(a) Meristematic Tissue


(b) Permanent Tissue
(c) Epithelial Tissue
(d) Connective Tissue
11 The fat storing ………….is found below the skin and between internal organs. 1

(a) Aerolar Tissue


(b) Adipose Tissue
(c) Compact tissue
(d) Hyaline cartilage
12 1
In aquatic plants large air cavities are present in parenchyma to give buoyancy to the
plants to help them float such a parenchyma, type is called
(a) Collenchyma
(b) Sclerenchyma
(c) Aerenchyma
(d) Parenchyma
KENDRIYA VIDYLAYA SANGATHAN::: RANCHI REGION

13 1

In given figure the fielder pulls his hands gradually with the moving ball while holding a
catch. This is an example of which type of motion
(a) First law of motion
(b) Second law of motion
(c) Third law of motion
(d) None of these
14 The graph of an object moving with uniform speed is a 1
(a) straight lines
(b) radial lines
(c) concentric circles
(d) Parallel lines

15 Work done by force on an object is equal to the magnitude of the force multiplied by 1
the distance mode in the direction of the force. From the above out which statement is
true
(a) Work has no magnitude and no direction
(b) Work has magnitude and direction
(c) Work has no magnitude but only have direction
(d) Work has magnitude only but no direction
16. The audible range of human ear is about 1
(a) 1Hz to 20Hz
(b) 20Hz to 20000Hz
(c) 25Hz to 1000Hz
(d) 50Hz to 50000Hz
Q. no 17 to 20 are Assertion - Reasoning based questions.
These consist of two statements – Assertion (A) and Reason (R). Answer these
questions selecting the appropriate option given below:
(a) Both A and R are true and R is the correct explanation of A
(b) Both A and R are true and R is not the correct explanation of A
(c) A is true but R is false
A is False but R is true
17. Assertion:- If the net external force on the body is zero, then it’s acceleration is zero. 1
Reason:- Acceleration doesn’t depend on force.
KENDRIYA VIDYLAYA SANGATHAN::: RANCHI REGION

18. Assertion:- If the frequency of vibration is higher we say that the sound is shrill and has 1
a higher pitch.
Reason:- The frequency determine the shrillness Or the pitch of sound

19. Assertion:- Permanent tissue is composed of mature cells. 1


Reason:- Meristematic tissue is a group of actively dividing cells.

20. Assertion:- Number of electron always equal to the proton number of atom. 1
Reason:- Atom is made up of electron, proton and neutron.

SECTION – B
Q. no. 21 to 29 are very short answer questions.
21 What do you mean by 2
(a) inertia
(b) momentum
OR
An object of mass 30 kg changes it’s velocity from 10ms-1 to15ms-1.
Time taken by object is 5 second. Find the force required to stop the object.

22 State the second law of motion. 2

23 2
Define:- (a) Valency (b) Atomic number

24 What do you mean by buoyant force. State the Archimedes’ principle. 2

25 What is Intercropping. Write it’s two advantage. 2

26. What do you understand by diffusion. How it is different from osmosis. 2

27. Write the four types of epithelial tissue. 2


OR
Write the two types of plant tissue.
28 What are the characteristics of particles of matter. 2
OR
Solids have higher density than liquid so why ice float on water.
29 What is latent heat of fusion. Change the temperature into Celsius from Kelvin:- 2

(a) 373 K
(b) 273 K

SECTION – C
Q. no. 30 to 34 are short answer questions
KENDRIYA VIDYLAYA SANGATHAN::: RANCHI REGION

30 What is the conclusion of Rutherford from the alpha particle scattering experiment on 3
gold foil.
OR
Explain with examples:-
(a) Isotopes (b) Isobars
31 What is the difference between eukaryote and prokaryote cell. 3
OR
What is the function of:-
(a) Ribosome
(b) Mitochondria
32 Write in brief :- 3
(a) Skeletal muscles
(b) Smooth muscle
(c) Cardiac muscle
33 3
State the law of gravitation. What is the value and S.I unit of ‘G'. Calculate the
relationship between ‘g ' and ‘G'.
OR
What is acceleration due to gravity? The mass of the earth is 6×1024 kg and that of
the moon is 7.4×10 22 kg. If the distance between the earth and the moon is 3.84×10
km, calculate the force exerted by the earth on the moon. ( G = 6.7×10-11 Nm2 kg-2)

34 What do you understand by the time period of the wave? What is the relationship 3
between time period and frequency? What is the SI unit for time period and
frequency?
SECTION - D
Q. no. 35 to 37 are Long answer questions.
35 The molecular mass of a substance is the sum of the atomic masses of all the atoms in a 5
molecule of the substance therefore the relative mass of a molecule expressed in
atomic mass unit(u)
(a) Calculate the molecular mass of ( H2O)
(b) Calculate the molecular mass of (HCl)

OR
(a) What do understand by the Law of conservation of mass? In a reaction, 5.6g of
sodium carbonate reacted with 5.7g of ethanoic acid. The products were carbon
dioxide of 2.4 g, 0.9 g of water and 8 g of sodium ethanoate. State whether the
reaction follow the law of conservation of mass.
(b) What is the law of constant proportion? Explain it with an example.
36 (a) Draw a well labelled diagram of plant cell. 5
(b) Write the function of any two organelle:-
Mitochondria, Lysosomes, Endoplasmic reticulum, Plastids
OR
(a) What are unicellular and multicellular organisms?
(b) Write a short note on cell wall.
KENDRIYA VIDYLAYA SANGATHAN::: RANCHI REGION

37 Due to wrong parking a car was towed and a 800 kg towing car pulls the car of 200 5
kg, along a horizontal track. If the towing car exerts a force of
4000 N and the track offers a friction force of 500 N, then calculate:
(a) The net accelerating force
(b) The acceleration of the car
SECTION - E
Q.no. 38 to 40 are case - based/data -based questions with 2 to 3 short sub - parts. Internal choice is provided
in one of these sub-parts.
38 Work is the process of energy transfer to the motion of an object via the 4
application of a force, often represented as the product of force and displacement. A
force is said to do positive work if the force has a component in the direction of the
displacement of the point of application. And if components are in opposite
directions, then the work done will be negative. Power measures the speed of work
done, that is how fast or slow work is done. Power is defined as the rate of doing
work or the rate of transfer of energy.
(a) What do you understand by power? What is the unit of power?
(b) A girl of weight 200 N climbs up a rope through a height of 10 m. The time taken
by the girl to reach the height is 20 second. What is the power expended by the
girl.
39 Beekeeping is the maintenance of honey bee colonies, commonly in hives, by human. 4

Bees are accommodated in artificial hives where they live comfortably within easy
reach of the bee keeper for examination and extraction of surplus honey, after keeping
of sufficient in the combs for the bees. Honey is a product of bees, which gather sugar
containing nectars from flowers. Honey should be processed as soon as possible after
removal from the hive. The value or quality of honey depends on the pasturage, or the
flowers available to the bees for nectar and pollen collection.

(a) What is the other name for beekeeping?


(b) What is the pasturage and how is it related to honey production.
Or
(a) What is advantage of bee farming?
(b) Name some varieties of bees used for commercial honey production.
KENDRIYA VIDYLAYA SANGATHAN::: RANCHI REGION

40 4

(a) What are elements? Write two properties each of Metal and Non metals.
(b) Write the difference between Homogenous and Heterogeneous mixture.
(c) Write the difference between Mixture and compounds.
Or
( c) what are metalloids. Give two examples.

…………………
KENDRIYA VIDYLAYA SANGATHAN::: RANCHI REGION

KENDRIYA VIDYALAYA SANGATHAN RANCHI REGION


SAMPLE PAPER (086)
CLASS-IX
(MLL)
Max. Marks: 80 SUBJECT SCIENCE Time Allowed: 3Hrs.

SECTION - A
Select and write one most appropriate option out of the four options
given for each of the questions 1 – 20
Q No QUESTIONS MARKS
1 (D) Density 1
2 (C) 30% Zinc And 70%Copper 1
3 (B) Mixture Of Water And Milk 1
4 (D) Tetra Atomic 1
5 (C) Gold 1
6 (B) 14:3 1
7 (D) Schleiden And Schwann 1
8 ( C) Smooth Endoplasmic Reticulum 1
9 (C) Adenosine Triphosphate 1
10 10. (A ) Meristematic Tissue 1
11 (B) Adipose Tissue 1
12 © AERENCHYMA 1
13 (B) Second Law Of Motion 1
14 (A) Straight Line 1
15 (B) Work Has Magnitude And Direction 1
16 (B) 20 To 20000hz 1
17 A 1
18 A 1
19 B 1
20 A 1
21 (A) It Is The Property Of An Object To Remain In The State Of Rest And Motion 2
Is Called Inertia.
(B) It Is The Product Of Mass And Velocity Of The Object. Or
Mass= 30 Kg ,Time Taken(T) =5 Second , Initial Velocity(U) = 10m/S Final
Velocity(V) =15m/S Acceleration(A) =( Final – Initial)/Time Taken
A = (V-U)/T = (15-10)/5= 1 M/S2 Force = Mass (M) X
Accleration (A) = 30 X 1 = 30 N
22 The Second Law Of Motion States That The Rate Of Change Of Momentum Of 2
An Object Is Proportional To The Applied Unbalanced Force In The Direction Of
The Force.
23 (A) The Combining Capacity Of An Element Is Known As Its Valency . (B) The 2
Number Of Proton Present In The Nucleus Atom Of An Element Is Known As
Atomic Number
24 The Upward Force Exerted By The Water On The Object Is Known As Upthrust 2
Or Buoyant Force.
Archimedes Principle States That “ When Body Is Immersed Fully Or
Partially In A Fluid , It Experience An Upward Force That Is Equal To The Weight
Of The Of The Fluid Displaced By It.”
KENDRIYA VIDYLAYA SANGATHAN::: RANCHI REGION

25 Inter Cropping Is Growing Two Or More Crops Simultaneously On The Same 2


Field In The Definite Pattern. For Example:-Soyabean +Maize. I
Advantages Of Intercropping Are:- (A) In Intercropping There Is Maximum
Utilisation Of The Nutrient Supplied. (B) Also Prevent The Spreading Pest And
Diseases To All The Plants.
26 . Diffusion: - The Spontaneous Movement Of A Substance From A Region Of 2
Higher Concentration To A Region Of Lower Concentration. It Is Different From
Osmosis As It does not require any permeable membrane for the movement of
the substance.
27 . Diffusion: - The Spontaneous Movement Of A Substance From A Region Of 2
Higher Concentration To A Region Of Lower Concentration. It Is Different From
Osmosis As It Does Not Require Any Permeable Membrane For The Movement
Of The Substance.
28 Characteristics Of Particles Of Matter:- (A) Particles Of Matter Are Continuously 2
Moving (B) Particles Of Matter Attract Each Other (C) Particles Of Matter
Have Space Between Them.
Or
Because The Frozen Water (Ice) Is Less Dense Than The Liquid Water.
29 . Latent Heat Of Vaporisation Is The Heat Energy Required To Change 1 Kg Of A 2
Liquid To Gas At Atmospheric Pressure At Its Boiling Point.
(A) 373k = 100⁰C (B) 273 = 0⁰C
30 Rutherford Concluded From The Experiment That- (I) Most Of The Spaces Inside 3
The Atom Is Empty. (Ii) Very Few Particles Were Deflected From The Path ,
Indicating That The Positive Charge Of The Atom Occupies Very Little Space .
(Iii) A Very Small Fraction Of Α- Particles Were Deflected By 180⁰, Indicating That
All The Positive Charge And Mass Of The Gold Atom Were Concentrated In A
Very Small Volume.
Or
(A) Isotopes: - The Atoms Of The Same Element , Having The Same Atomic
Number But Different Mass Number. For Ex- Carbon , C612 , C614
(B) Isobars: - Atoms Of Different Element With Different Atomic Number But
Having The Same Mass Number. For Ex- Calcium And Argon.
31 Prokaryotic cell Eukaryotic cell 3
1. Size Generally small 1-10 micrometre 1.Size generally large 5- 100
micrometre
2.Nuclear region is undefined containing only nucleic acid is called a nucleoid
2.Nuclear region is well defined and surrounded by a nuclear membrane
3. Single chromosome 3. More than one chromosome
4.Membrane bound cell organelles absent 4.Membrane bound cell
organelles are present
OR
(a) Ribosomes are present in all active cells are the site of protein
manufacturer They manufactured proteins and then sent to various places in
the cell depending on need using the ER.
(b) Mitochondria are known as the powerhouse of the cell the energy
required for various chemical activities needed for life is released by
mitochondria in the form of ATP molecule ATP is known as the energy currency
of the cell.
32 (a) Muscles present in our limbs move when we want them to these muscles are 3
also called skeletal muscles as they are mostly attached to bones and help in
KENDRIYA VIDYLAYA SANGATHAN::: RANCHI REGION

body movement. The cells of this tissue are long cylindrical and branch and
multinucleated.
(b) Smooth muscles are involuntary muscles control the movement of food in
the elementary canal or the contraction and relaxation of blood vessels they are
also found in the iris of the eye in ureters add in the bronchi of the lungs the
cells are long with pointed ends and uni nucleated.
( c) The muscles of heart show rhythmic contraction and relaxation throughout
life these involuntary muscles are called cardiac muscles heart muscles cells are
cylindrical branched and uni nucleated
33 . According to the universal law of gravitation the force between 2 objects is 3
directly proportional to the product of their masses. The value of ‘G' = 6.673×10-
11
N m2 Kg -2..
The relationship between ‘g ' and ‘ G' is g= G M/ R2
OR Whenever an object falls towards the earth and acceleration is involved dis
acceleration is due to gravitational force therefore it is called the acceleration
due to gravity.
The mass of earth, M = 6×1024Kg, The Mass of the moon, m = 7.4× 1022Kg ,
The distance between the earth and the moon, d = 3.84 × 108 m. So F = G M×
m/ d2 = 2.01 × 1020 N.

34 The time taken by two consecutive compression or rarefaction to cross a fixed 3


point is called the time period of the wave. In other words the total time taken
to complete one oscillation in the density of the medium is called the time
period of the sound wave. It is represented by the symbol capital T . Its SI unit is
second(s). The SI unit of frequency is Hertz (Hz).
Relationship between Frequency and Time period:- v= 1/T
35 (a) Atomic mass of hydrogen =1u Oxygen =16 u the molecular mass of water 5
which contains two atoms of hydrogen and one items of oxygen is= 2×1+1×16=
18u
(b) The molecular mass of HCl= the atomic mass of H + the atomic mass of Cl=
1+35.5= 36.5 u. OR
(a) Law of conservation of mass states that that “mass can neither be created
nor destroyed in a chemical reaction.”
Sodium carbonate (5.6g) + Ethanoic acid ( 5.7g) = carbon dioxide( 2.4g ) +
water( 0.9g) + sodium ethanoate(8g). As the total mass reactant side and
products sides are equal therefore the reaction follows the law.
b) The law of constant proportion is also known as the law of definite
proportion this law states that in a chemical substance the elements are always
present in definite proportions by mass. In a compound such as water the ratio
of the mass of hydrogen to the mass of oxygen is always 1:8 whatever the
source of water thus if 9 gram of water is decomposed one gram of hydrogen
and 8 gram of oxygen are always obtained.
36 CORRECT DIAGRAM 5
Function of:- a. Mitochondria:- Mitochondria are known as the powerhouse of
the cell the energy required for various chemical activities needed for life is
released by mitochondria in the form of ATP molecule. ATP is known as the
energy currency of the cell. The body uses energy stored in ATP for making new
chemical compounds and for mechanical work.
b. Lysosomes:-Lysosomes help to keep the cell clean by digesting any foreign
material as well as worn out cell organelles foreign materials entering the cells
KENDRIYA VIDYLAYA SANGATHAN::: RANCHI REGION

such as bacteria or food as well as old organelles end up in the lysosome which
break them into small pieces.
c. ER:- There are two types of ER rough endoplasmic reticulum (RER) and
smooth endoplasmic reticulum (SER) . The rough endoplasmic reticulum has
ribosome attached to its surface which manufactures protein. The SER helps in
the manufacture of fat molecules or lipids.
d. Plastids:- Plastids are present only in plants as there are 2 types of plastids:-
chloroplasts and leucoplasts . Plastids containing the pigment chlorophyll are
known as chloroplast. They are very important for the photosynthesis.
Leucoplasts are primary organelle in which material such as starch, oil and
protein granules are stored.
OR
(a) Organisms which are made up of a single cell are known as unicellular
organisms. On the other hand Organisms which are made up of many cells
together in a single body assumed different function in it to form various body
parts are known as multicellular organisms.
(b) Plant cells in addition to the plasma membrane have another rigid outer
covering called the cell wall. The cell wall lies outside the plasma membrane the
plant cell wall is mainly composed of cellulose. Cell wall Permit the cells of
plants fungi, bacteria to withstand very dilute external media without bursting.
37 (a) The net acceleration force= force of towing car – friction = 4000 - 500 = 5
3500N.
(b) Mass = 200 kg Force= 3500 N.
Force = m× a , => 3500= 200 kg ×a => a= 3500/200 => a= 17.5 m/ s 2
38 (a) Power is defined as the rate of doing work or the rate of transfer of energy. 4
The unit of power is watt.
(b) Weight of the girl, mg= 200 N, Displacement(h) = 10 m, Time taken (t)= 20 s,
power (P) = Work done/ Time taken = mgh/t = (200× 10) / 20 = 100 W.
39 (a) Apiculture is the other name of bee- keeping. 4
(b) The value or quality of honey depends upon the pasturage, or the flowers
available to the bees for nectar and pollen collection in addition to adequate
quantity of pasturage the kind of flowers available will determine the taste of
the honey.
OR (a) Since beekeeping needs low investments farmer use it as an additional
income generating activities in addition to honey the bee hives are a source of
wax which is used in various medicinal preparations.
(b) The local varieties of bees for commercial honey production are Apis cerana
indica , A. dorsata , A. florae.
40 (3) An element is a basic form of matter that cannot be broken down into 4
simpler substance as by chemical reactions.
Properties of metal:- i. They have lustre. (ii) They conduct Heat and
electricity. (iii). They are malleable and they are ductile.
Properties of non – metal :- (I) They display a variety of colours. (II) They
are poor conductors of heat and electricity. (III) They are not lustrous,
sonorous or malleable.
(b) Given in the figure.
( c)
Mixture Compounds
1. Elements or compounds just mix together to form a mixture and no new
compound is formed. 1. Elements react to form new compounds.
KENDRIYA VIDYLAYA SANGATHAN::: RANCHI REGION

2. A mixture has a variable composition. 2. The composition of each new


substance is always fixed.
3. A mixture shows the properties of the constituent substances. 3. The
new substance has totally different properties.
OR, ( c) Some elements have intermediate properties between those of
metals and non-metals they are called metalloids examples are boron
silicon etc.

………………….
KENDRIYA VIDYLAYA SANGATHAN::: RANCHI REGION

Composition of Board Examination 2022-23; Class IX

Blue Print as per CBSE Sample Paper

Unit / 01 A-R 02 03 05 04 Sub- Total


Chapter Mark Questio Marks Marks Marks Marks Total Unit
Name Questi ns Questi Questi Questi Questi Chapt wise
ons [Asserti ons ons ons ons er
MCQ on- Sourc wise
Reasoni e
ng] based
/ Case
based

No. of No. of No. of No. of No. of No. of No. of


questi questio questi questi questi questi questi
ons ns ons ons ons ons ons
[Mark [Marks [Mark [Mark [Mark [Mark [Mark
s allotted s s s s s
allotte ] allotte allotte allotte allotte allotte
d] d] d] d] d] d]

Unit-I Matter 13(25)


- Its Nature
and
KENDRIYA VIDYLAYA SANGATHAN::: RANCHI REGION

Behaviour

Matter in Our 1(1) 1(2)O 1(3) 3(6)


Surroundings R

Is Matter 1(1) 1(5)O 2(6)


Around Us R
Pure
(Excluding
Separating
the
components
of a mixture)

Atoms and 1(1) 1(1) 2(4) 4(6)


Molecules

Structure of 3(3) 1(4)O 4(7)


Atom R

Unit-II 10(22)
Organisation
in the Living
World

Cell: Basic 2(2) 1(2)O 1(3) 1(4)O 5(11)


unit of Life R R

Tissues, 2(2) 1(1) 1(3)O 1(5)O 5(11)


Organs, R R
Organ
system,
Organism

Unit-III 14(27)
Motion,
Force and
Work

Motion 1(1) 1(3) 2(4)

Force and 1(1) 2(4) 3(5)


Newton’s
laws
KENDRIYA VIDYLAYA SANGATHAN::: RANCHI REGION

Gravitation 1(1) 1(4)O 2(5)


R

Floatation 1(1) 1(3) 2(4)

Work, Energy 1(1) 1(1) 1(3)O 3(5)


& Power R

Sound 1(1) 1(3) 2(4)

Unit IV Food 2(6)


Production

(Improvemen 1(1) 1(5) 2(6)


t in Food
Resources)

Sub-Total 16(16) 4(4) 6 (12) 7(21) 3(15) 3(12) 39


(80)

“OR” means that there is an alternate question in that particular question.


KENDRIYA VIDYLAYA SANGATHAN::: RANCHI REGION

KENDRIYA VIDYALAYA SANGATHAN RANCHI REGION


SAMPLE PAPER (086)-IX
(AVERAGE LEVEL)
Max. Marks: 80 SUBJECT SCIENCE Time Allowed: 3 hours
General Instructions:
i. This question paper consists of 39 questions in 5 sections.
ii. All questions are compulsory. However, an internal choice is provided in some questions. A student
is expected to attempt only one of these questions.
iii. Section A consists of 20 objective type questions carrying 1 mark each.
iv. Section B consists of 6 Very Short questions carrying 02 marks each. Answers to these questions
should in the range of 30 to 50 words.
v. Section C consists of 7 Short Answer type questions carrying 03 marks each. Answers to these
questions should in the range of 50 to 80 words
vi. Section D consists of 3 Long Answer type questions carrying 05 marks each. Answer to these
questions should be in the range of 80 to 120 words.
vii. Section E consists of 3 source-based/case-based units of assessment of 04 marks each
with sub-parts.

SECTION - A
Select and write one most appropriate option out of the four options given for each of the
questions 1 – 20
Q No QUESTIONS MARKS
1 The Temperature at which a solid melts to become a liquid at atmospheric 1
pressure is called
(a) Melting point
(b) Boiling point
(c) Sublimation
(d) Freezing point
2 The boron, germanium, silicon etc. are an example of 1
(a) Metalloids
(b) Metal
(c) Non metals
(d) Compounds
3 What is the radius of atom of hydrogen 1
(a) 10 9 nm
(b) 10 -1 nm
(c) 10 nm
(d) 10-10 nm
4 According to Bohr model the electron revolve in discrete orbit. These orbits or 1
shells are called
(a) Azimuthal quantum number
(b) Energy levels
(c) Magnetic number
(d) Spectra
14
5 7X , this is symbol for which element 1
KENDRIYA VIDYLAYA SANGATHAN::: RANCHI REGION

(a) Oxygen
(b) Nitrogen
(c) Hydrogen
(d) Sodium
6 Thomson model of an atom is similar to that of a Christmas pudding in which the 1
sphere denotes
(a) Negative charge
(b) Positive charge
(c) Neutral charge 1
(d) Both Negative and positive charge
7 The plasma membrane allows or permits the entry and exit of some materials in 1
and out of the cell. This property of cell membrane is called
(a) Selective Permeable membrane
(b) Biogenesis membrane
(c) Nuclear membrane
(d) Mitochondrial membrane
8 In Golgi apparatus consist of a system of membrane bound vesicles arrange 1
approximately parallel to each other in stacks called
(a) Lysosomes
(b) Cisterns
(c) Plastids
(d) Vacuoles
9 Tissues that are found below the skin and between internal organs and store fats 1
are known as

(a) Aerolar tissue


(b) Adipose tissue
(c) Epithelial tissue
(d) Permanent tissue
10 The cells of cork are dead and compactly arranged without intercellular spaces. 1
They have a chemical in the walls that make them impervious to gases and water.
(a) Lignin
(b) Suberin
(c) Cellulose
(d) Chitin
11 1

Observe the figure given. What happens when the stone describing a circular path
with a velocity of constant magnitude , on being released the stone moves along
(a) Circular path
(b) Hexagonal track
(c) Straight line tangential to the circular path
(d) Circular path perpendicular to the person
12 The recoiling of gun is an example of which law of motion 1
(a) First
(b) Second
KENDRIYA VIDYLAYA SANGATHAN::: RANCHI REGION

(c) Third
(d) None of them
13 The value of ‘ G ‘ is 1
(a) 6.67 × 1011 N m2 kg -2
(b) 6.67 × 10-11 N m2 kg 2
(c) 6.67 × 1011 N m -2 kg -2
(d) 6.67 × 10-11 N m 2 kg -2
14 In a beaker the cork floats and the iron nail sink due difference in their 1

(a) Mass
(b) Volume
(c) Density1
(d) Composition
15 1

When we stretch the string of the bow to release the arrow what type of energy is
converted to kinetic energy due to change in the shape of bow.
(a) Chemical energy
(b) Electrical energy
(c) Potential energy
(d) Heat energy
16 Sound of frequency… … … . are called infrasound. 1
(a) Above 20 Hz
(b) Below 20 Hz
(c) Between 20- 20 kHz
(d) None of these
Q. no 17 to 20 are Assertion - Reasoning based questions.
These consist of two statements – Assertion (A) and Reason (R). Answer these
questions selecting the appropriate option given below:
(d) Both A and R are true and R is the correct explanation of A
(e) Both A and R are true and R is not the correct explanation of A
(f) A is true but R is false
(g) A is False but R is true
17 Assertion: Parenchyma cells help in storage of food. 1
Reason: Parenchyma cells are the main seats of photosynthesis.
KENDRIYA VIDYLAYA SANGATHAN::: RANCHI REGION

18 Assertion: Atomicity of ozone is three while that of oxygen is two. 1


Reason: Atomicity is the number of atoms constituting a molecule.
19 Assertion: The work done during a round trip is not zero.. 1
Reason: No force is required to move a body in its round trip.
20 Assertion: Light is essential in poultry farm management. 1
Reason : 14-16 hours of light including day light is required for optimal production
of eggs.
SECTION – B
Q. no. 21 to 26 are very short answer questions.
21 What are the factors which affect the rate of evaporation. How evaporation causes 2
cooling.
OR
Convert the following to Celsius scale and write the state of the matter after
conversion as shown in the table given below:
Matter Temperature in Kelvin
Ice 310K
Water 373 K
Water 270 K
22 What is molecule? How Atomicity is different from polyatomic ion. 2
23 When a sailor jumps out of a rowing boat. As the sailor jumps forward what 2
happens to the boat. Which law of motion is applicable to explain it.
24 Define atomic mass unit ( u). What is the atomic mass of:- 2
(a) Sodium
(b) Chlorine
25 2

(a) When a Striker strike the pile of coins at the bottom so strongly. What
would happen to the piles of the coins?
(b) State why the first law of motion is known as the law of inertia.
OR
(a) What is momentum? Write its SI unit.
(b) A force act on an object of mass 7 kg for duration of 5 s. It increase the
velocity from 3metre per second to 8 metre per second. Find the force
applied.
26 What is Osmosis? What is movement of water when we put a animal cell in 2
hypertonic solution and hypotonic solution.
SECTION - C
Q.no. 27 to 33 are short answer questions.
27 (a) What do you understand by low pitch and high pitch 3
sound?
KENDRIYA VIDYLAYA SANGATHAN::: RANCHI REGION

(b) What do you mean by Compression and Rarefaction.


(c) What is a transverse wave?
28 3

(a) Identify the gasses evolved in the above experimental set up.
(b) Name and define the process that occurs.
(c) Illustrate what do you infer from the above experiment.
29 Draw a labelled diagram of neuron. Write one function of neuron. 3
OR
Write a short note on any two of the following:- (a) Blood (b) Cartilage
( c) ligament (d) Bone
30 (a) Under what conditions work is said to be done ? 3
(b) (b) A porter lifts a luggage of 10 kg from the ground and puts it on his head
1.5m above the ground. Calculate the work done by him on the luggage.
31 (a) An object of mass 10 kg is accelerated uniformly reach a velocity from 5 m 3
s -1 to 9 m s -1 in 8 second. Calculate the initial and final momentum of the
object. Also calculate the force applied.
(b) A fielder pulls his hands gradually with the moving ball while holding a
catch. Give reason.
32 What are the important phenomenon explained by Universal Law of Gravitation. 3
Earth attracts object towards itself. Calculate the acceleration due to gravity, if the
mass of the earth = 6×1024 kg , radius of earth =6.4×10 6 m. ( G= 6.7×10 -11 Nm/s -2)
OR

(a) What do you understand by the term weight.


(b) An object weighs 30N when measured on earth. The weight of the same
person on the surface of moon will be?
33 What do you mean by membrane biogenesis? What are the two types of 3
Endoplasmic reticulum, what they manufacture in the cell?
SECTION - D
Q.no. 34 to 36 are Long answer questions.
34 A group of students took a box and painted it with a black paint from all sides. A 5
torch is fixed at the end of the box by making a hole in it . On the another side a
hole is made to see the effect. On placing a milk sample contained in a
beaker/tumbler in the box as shown in the figure. Milk taken in the tumbler was
illuminated. Same procedure is repeated with a salt solution but found that light
simply passed through it.
KENDRIYA VIDYLAYA SANGATHAN::: RANCHI REGION

(a) What is reason that the milk beaker illuminated .Name the phenomenon
involved.
(b) Explain why light simply pass through the salt solution in the experiment.
c) Milk is what type of mixture. Does the solution of copper sulphate show
scattering of light
d) What is heterogeneous mixture.
35 (a) What are Micronutrients and Macronutrients? Why are they named so. 5
What is its role in the development of plant.
(b) What is the difference between Manure and Fertilizers.

OR
(a) What is the method used scientifically to improve the cattle breed. Why it is
needed to improve a variety of Milk giving cattle and how it can be done.
(b) In India there is a intense fish farming in composite fish culture systems.
What is its advantages and disadvantages. How you will overcome it.
36 The given diagram is a diagram of different plant tissues.. The diagram is shown in 5
the longitudinal section. Identify them
(a) What is role of tissue shown in the diagram (a) in the plant?
(b) All the above diagrams are what type of tissue.
(c) The diagram (b) there is chloroplast in which photosynthesis occurs what
type of tissue it is called under such situation.
(d) Husk of coconut is made up of which type of tissue.

SECTION - E
Q.no. 37 to 39 are case - based/data -based questions with 2 to 3 short sub - parts.
Internal choice is provided in one of these sub-parts.
37 The atom is a complex arrangement of negatively charged electrons arranged in 4
defined shells about a positively charged nucleus. This nucleus contains most of
the atom's mass and is composed of protons and neutrons (except for common
KENDRIYA VIDYLAYA SANGATHAN::: RANCHI REGION

hydrogen which has only one proton). All atoms are roughly the same size. A
convenient unit of length for measuring atomic sizes is the angstrom (Å), which is
defined as 1 x 10-10 meters. The diameter of an atom is approximately 2-3 Å.
In 1897, J. J. Thomson discovered the existence of the electron, marking the
beginning of modern atomic physics. The negatively charged electrons follow a
random pattern within defined energy shells around the nucleus. Most properties
of atoms are based on the number and arrangement of their electrons. The mass
of an electron is 9.1 x 10-31 kilograms.
(a) What is the diameter of an atom
(b) What is the charge on electron?
(c) Which element had no neutron?
(d) 1 angstrom is equal to… … m
38 Gravitation is the force of attraction between two objects in the universe. 4
Gravitation maybe the attraction of objects by the earth disposes proportional to
the product of masses subject and inversely proportions to the square of the
distance between them it is independent of medium .Gravitation maybe the
direction between objects in outer space.
(a) What happens to the force of gravitation when the distance between two
objects is doubled?
(b) Gravitation between two objects increased four times if the masses of the
two objects are the same, so what will be its new distances.
39 Plasma/Cell membrane: This is the outermost covering of the cell that separates the 4
contents of the cell from its external environment. The plasma membrane allows or
permits the entry and exit of some materials in and out of the cell so the cell membrane is
called a selectively permeable membrane. Some substances like CO2 or O2 gases can move
across the cell membrane by a process called diffusion. The movement of water molecules
(liquid) through such a selectively permeable membrane is called osmosis. Osmosis is the
passage of water from a region of high water concentration through a semi-permeable
membrane to a region of low water concentration. If the medium surrounding the cell has
a higher water concentration than the cell, the cell will gain water by osmosis. Such a
solution is known as hypotonic solution.
(a) What is Osmosis?
(b) Why the cell membranes are called selective permeable.
(c) What happens to the cell when we put it in a isotonic solution.
Or
( c) What is hypotonic solution.

……………….
KENDRIYA VIDYLAYA SANGATHAN::: RANCHI REGION

KENDRIYA VIDYALAYA SANGATHAN RANCHI REGION


SAMPLE PAPER (086)
CLASS-IX
(Average level)
Solution
Max. Marks: 80 SUBJECT SCIENCE Time Allowed: 3 Hrs.

SECTION – A
Select and write one most appropriate option out of the four options
given for each of the questions 1 – 20
Q No QUESTIONS MARKS
1 Melting point
2 metalloids
3 10-10 nm
4 energy levels
5 Nitrogen
6 Positive charge
7 Selective permeable membrane
8 b) Cisterns
9 (b) Adipose tissue
10 (b) Suberin
11 © Straight line tangential to the circular path
12 © Third
13 (d) 6.67×10-11 N m2 kg- 2
14 © Density
15 © Potential energy
16 ( b) beloy30 Hz
17 B
18 A
19 19. D
20 A
21 Factors affecting evaporation:-
• An increase of surface area
• an increase of temperature
• And decrease in humidity
• an increase in wind speed
The particles of liquid absorb energy from the surrounding to regain the energy
lost during evaporation this absorption of energy from the surroundings make
the surroundings cold.
22 A molecule can be defined as the smallest particle of an element or a
compound that is capable of independent existence and shows all the
properties of that substance.
The number of atoms constituting a molecule is known as its atomicity. A
group of atoms carrying a charge is known as a polyatomic ion. OR
310 K= 37°C Ice changes to liquid. Ii) 373 K = 100°C Water changes to
vapour. Iii) 270K = -3°C Water changes to solid or ice.
23 3rd law of motion is applicable as the sailor jumps forward so that the boat
moves backward
KENDRIYA VIDYLAYA SANGATHAN::: RANCHI REGION

24 One atomic mass unit is a mass unit equal to exactly 1/12 the mass of one
atom of carbon 12.(a) sodium ( Na) = 23u (b ) Chlorine (Cl) = 35.5 u
25 (a) If the hit is strong enough, the bottom coins moves out quickly. The inertia
other coins make them fall vertically on the table.
(b) All objects resists a change in their state of motion. In qualitative way the
tendency undisturbed objects to stay at rest or motion This is why first law of
motion is also known as law of inertia.
OR Momentum is the product of mass and velocity . The SI unit of momentum
is KG metre per second.
(b) mass(m) = 7kg , u= 3 m/s v= 8 m/s time (t) = 5 s
F= 7(8-3) /5 =7N
26 When a animal cell is put in a hypertonic solution the water moves out of the
cell while in a hypotonic solution the water moves inside the cell cause
swelling of cell.
27 (a) The faster the vibration of the source the higher is the frequency and the
higher is the pitch . Thus low pitch has less number of compression and
rarefaction passing a fixed point per unit time whereas high pitch sound
corresponds to more number of compression and rarefaction passing a fixed
point per unit time.
(b) When a vibrating object moves forward it pushes and compress is the air
in front of it creating a reason of high pressure play stop this reason is called a
compression. When the vibrating object moves backwards it creates a reason
of low pressure called rarefaction.
( c ) A transverse wave is the one in which the individual particles of the
medium move about their mean positions in a direction perpendicular to the
direction of wave propagation.
28 (a) Ammonium chloride (b) This process is known as sublimation it is
defined as a change of a state directly from solid to gas without changing into
liquid state or vice versa is called sublimation.
( c) Some substances undergoes vapour stage from solid without being going
to liquid state.
29 Function of neuron:- Neurons are the highly specialised cells star being
stimulated and transmit the stimulus very rapidly from one place to another
within the body impulses allow us to move our muscles when we want to.
OR (a) Blood has a fluid matrix called plasma in which red blood cells white
blood cells and platelets are suspended the plasma contains protein salt and
hormones blood flows and transports gas is digested food hormones and
waste material.
(b) Cartilage is another type of connective tissue cartilage smoothens bone
services head joins hands also present in nose , ear trachea and larynx.
( c) Two bones can be connected to each other by another type of connective
tissue called the ligament . This tissue is very elastic it has considerable
strength
(d) Bone is another example of connective tissue it forms the framework that
supports the body it also anchors the muscles and support the main organs of
the body it is a strong and non- flexible tissue.
30 (a) Work done by a force acting on an object is equal to the magnitude of the
force multiplied by the distance moved in the direction the force.
(b) Mass (m) = 10 kg, displacement (s) = 1.5 m, work done, W = F× s = mg × s
= 10 kg × 10 m s -2 × 1.5 = 150 J
KENDRIYA VIDYLAYA SANGATHAN::: RANCHI REGION

31 (a) Mass (m) = 10 kg , initial velocity(u) = 5 m s -1 , final velocity (v) = 9 m s-1


time taken (t) = 8 second. Initial momentum= 10×5 kg m s -1 = 50 kg m/s
Final momentum = 10× 9 = 90 kg m /s Force = m (v-u) /t = 10 (9-5) /8 = 5
N.
(b) In doing so fielder increases the time during which the high velocity of a
moving ball decreases to 0.
32 Importance of universal law of gravitation to explain the phenomenon :- I.
The force that binds us to the earth. II. The motion the moon around the
earth. III. The tides due to the moon and the sun.
g = G M/R 2 = 9.8m / s 2
Whenever any object falls towards the earth acceleration is involved. This
acceleration is called acceleration due to gravity.
OR (a) Weight of an object is the force with which it is attracted towards the
earth.
(b) 1/6 of 30 N
33 The SER helps in the manufacture of fat molecules, or repeats, important for
cell function some of the proteins prepared by RER and protein by SER help in
building the cell membrane this process of building cell membrane is known
as membrane Biogenesis.
RER:- The rough endoplasmic reticulum is involved in the protein synthesis
folding of protein and the sending the protein according to need of the cell.
SER:- The smooth endoplasmic reticulum helps in the manufacture of fat
molecules are lipid important parcel function.
34 (a) The name of the phenomena is Tyndall effect. This is the scattering light by
the small sized colloidal particles.
(b) Because of the small size Of salt particles they cannot scatter a beam of
visible light so that the light can easily pass through the solution.
35 (a) There are 16 nutrients which are essential for plants. The soil supplies the
13 nutrients to plants. ,amongst these 13 nutrients 6 are required in large
quantities and are called macro nutrients. The other 7 nutrients which are
required by the plant in a small quantity are there for cold micro nutrients.
(b) Deficiency of these nutrients affects physiological processes in plants
including reproduction growth and susceptibility to diseases to increase the
yield the soil can be increased by supplying these nutrients in the form of
manure and fertilizers. OR

(a) Animal husbandry eastern to improve the cattle breeds animal husbandry
r cattle husbandry eastern for 2 purposes milk and broad labour for
agriculture work milk production depends, to some extent on the duration of
the lactation., meaning the period of milk productions, milk production can be
increased by increasing the lactation period.
(b) In composite fish culture system both imported and the local fish disease
are used a combination of 5 or 6 piece species used in a single fish pond
species are selected so that they do not compete for food among them having
different types of food habits and as the food available in the parts of pond is
used decant use all the food in the pond without competing with each other
this increases the fish yield.
One problem with such composite fish culture is that many of these be
spread only during monsoon it is very hard to separate the wide variety from
these so the lack of availability of good quality seed is the major problem.
36 collenchyma (b) Parenchyma (c) Sclerenchyma
KENDRIYA VIDYLAYA SANGATHAN::: RANCHI REGION

(a) (b) Simple permanent tissue (c) Chlorenchyma (d) The


husk of coconut is made up of Sclerenchyma tissue , cells of these tissue are
dead.

37 (a) approximately 2-3 angstrom (b) negative charge


( c) Hydrogen (d) 10-10 m.
38 (a) reduces to the ¼ of the previous Gravitational force.
(b) To get a four times force we have to reduce the distance to half of the
previous distance.
39 (a) Osmosis is a special case of diffusion through a selectively permeable
membrane. Or Osmosis is the passage of water from a region or high water
concentration through a semipermeable membrane to a reason of low water
concentration.
(b) The cell membrane is called selectively permeable membrane because it
allows the entry and exit off some material in and out of the cell and also
provides movement of some other materials.
( c) Any isotonic solution there is no movement of water.
OR If the medium surrounding the cell has a higher water concentration than
the cell meaning that the outside solution is very dilute and the cell will gain
water by osmosis such a solution is known as hypotonic solution.
……………
KENDRIYA VIDYLAYA SANGATHAN::: RANCHI REGION

KENDRIYA VIDYALAYA SANGATHAN:::RANCHI REGION


Blue Print
Class : IX Sub : science. Max marks : 80
1 02 03 05 04
A-R
mark mark mark mark mark
S No. Chapter name questio Total
questi questio questio questio questi
n
on n n n on

Matter in our
1 1(1) - 1(2) OR 1(3) - - 3(6)
surrounding

Is matter around
2 1(1) - - - 1(5) OR - 2(6)
us pure

Atoms and
3 1(1) 1(1) 2(4) - - 4(6)
molecules

4 Structure of atom 3(3) - - - - 1(4) 4(7)

Cell : The basic


5 2(2) - 1(2) OR 1(3) - 1(4) 5(11)
unit of life

6 Tissues 2(2) 1(1) - 1(3) OR 1(5)OR - 5(11)

7 Motion 1(1) - - 1(3) - - 2(4)

Forces and laws


8 1(1) - 2(4) - - 6(10)
of motion

Gravitation and
9 2(2) - - 1(3) 1(4) 8(18)
flotation

Work,energy and
10 1(1) 1(1) - 1(3)OR - - 3(5)
power

11 Sound 1(1) - - 1(3) - - 2(4)

Improvement in
12 - 1(1) - - 1(5) - 2(6)
food resources

39
Sub total 16(16) 4 (4) 6 (12) 7 (21) 3 (15) 3 (12)
(80)
KENDRIYA VIDYLAYA SANGATHAN::: RANCHI REGION

CLASS: IX M.M. 80
Subject : Science Time: 3 hrs
General Instructions:
(i) The question paper comprises five sections A, B, C D and E. There are 39 questions
in
the question paper. All questions are compulsory.
(ii) Section–A - consists of 20 objective type questions carrying 1 marks each.
(iii) Section–B - consists of 6 very short questions carrying 02 marks each.Answers to
these questions should in the range of 30 to 50 words.
(iv) Section–C - consists of 7 short answer type questions carrying 03 marks each.
Answers to these questions should in the range of 50 to 80 words.
(v) Section–D – consists of 3 long answer type questions carrying 05 marks each.
Answer to these questions should be in the range of 80 to 120 words.
(vi) Section E consists of 3 source based/ case based units of assessment of 04 marks
each with sub - parts.

S. Mark
Question
No. s

Which of the following phenomena would increase on raising the


temperature?
(a) Diffusion, evaporation, compression of gases
1. 1
(b) Evaporation, compression of gases, solubility
(c) Evaporation, diffusion, expansion of gases
(d) Evaporation, solubility, diffusion, compression of gases

Mixture of two or more metals , or metals and non-metals called


as…………
2. 1
a) Alloy b) Solution c) Mixture d)
Metallic mixture

The chemical symbol for nitrogen gas is


3. 1
(a) Ni (b) N2 (c) N+ (d) N

Which of the following correctly represents the electronic distribution in


4. the Mg atom? 1
(a) 3, 8, 1 (b) 2, 8, 2 (c) 1, 8, 3 (d) 8, 2, 2

Which of the following are true for an element?


(i) Atomic number = number of protons + number of electrons
(ii) Mass number = number of protons + number of neutrons
5. (iii) Atomic mass = number of protons = number of neutrons 1
(iv) Atomic number = number of protons = number of electrons
(a) (i) and (ii) (b) (i) and (iii) (c) (ii) and (iii) (d) (ii)
and (iv)

Identify the Mg2+ ion from the figure where, n and p represent the
6. 1
number of neutrons and protons respectively.
KENDRIYA VIDYLAYA SANGATHAN::: RANCHI REGION

Which of these options are not a function of ribosomes?


(i) It helps in manufacture of protein molecules.
(ii) It helps in manufacture of enzymes.
7. (iii) It helps in manufacture of hormones. 1
(iv) It helps in manufacture of starch molecules.
(a) (i) and (ii) (b) (ii) and (iii) (c) (iii) and (iv) (d)
(iv) and (i)

Chromosomes are made up of


8. (a) DNA (b) Protein (c) DNA and protein (d) 1
RNA

Which muscles act involuntarily?


(i) Striated muscles
(ii) Smooth muscles
9. (iii) Cardiac muscles 1
(iv) Skeletal muscles
(a) (i) and (ii) (b) (ii) and (iii) (c) (iii) and (iv) (d)
(i) and (iv)

Which is not a function of epidermis?


(a) Protection from adverse condition (b) Gaseous
10. 1
exchange
(c) Conduction of water (d) Transpiration

A particle is moving in a circular path of radius r. The displacement after


11. half a circle would be: 1
(a) Zero (b) πr (c) 2r (d) 2πr

According to the third law of motion, action and reaction


(a) always act on the same body
12. (b) always act on different bodies in opposite directions 1
(c) have same magnitude and directions
(d) act on either body at normal to each other

Law of gravitation gives the gravitational force between


(a) the Earth and a point mass only
13. (b) the Earth and Sun only 1
(c) any two bodies having some mass
(d) two charged bodies only

The force of attraction between two unit point masses separated by a


14. 1
unit distance is called
KENDRIYA VIDYLAYA SANGATHAN::: RANCHI REGION

(a) gravitational potential (b) acceleration due to


gravity
(c) gravitational field (d) universal gravitational
constant

When a body falls freely towards the earth, then its total energy
(a) increases (b) decreases
15. 1
(c) remains constant (d) first increases and then
decreases

In SONAR, we use
16. (a) ultrasonic waves (b) infrasonic waves 1
(c) radio waves (d) audible sound waves

Direction: In each of the following questions, a statement of Assertion is


given followed by a corresponding statement of Reason. Of the
statements, mark the correct answer as
(a) Both assertion and reason are true and reason is the correct
explanation of assertion.
(b) Both assertion and reason are true but reason is not the correct 1
explanation of assertion.
(c) Assertion is true but reason is false.
(d) Assertion is false but reason is true

Assertion: Atomicity of ozone is three while that of oxygen is two.


17 Reason: Atomicity is the number of atoms constituting a molecule.

Assertion – Apical meristem is present at the growing tips of stems and


18. roots 1
Reason-Apical meristem is always located upper side of plant

Assertion : The work done during a round trip is not zero.


19 1
Reason : No force is required to move a body in its round trip

Assertion-Our natural resources are damaged due to photosynthesis of


20. plant. 1
Reason- Our natural resources damaged by green house effect

Why do we see water droplets on the outer surface of a glass


containing ice cold water?
OR
Analyse the temperature versus time graph of water, given below.
21.

Which region contains all liquids?

22. Define law of conservation of mass.


KENDRIYA VIDYLAYA SANGATHAN::: RANCHI REGION

23. Write two drawbacks of Dalton’s atomic theory.

If you are provided with some vegetables to cook, you generally add
salt into the vegetables. After adding salt, vegetables release water.
Why?
24.
OR
Name two cell organelles that have their own genetic material/DNA and
ribosomes?

25. Define 1 newton force.

Plot a graph between force applied on a body and the acceleration


26. produced in the given mass, assuming that the magnitude of force is
constantly changing.

A glass tumbler containing hot water is kept in the freezer compartment


of a refrigerator (temperature <0°C). If you could measure the
temperature of the content of the tumbler, which of the following graphs
would correctly represent the change in its temperature as a function of
27. time?

Describe the functions of the three organelles, viz Golgi bodies,


28.
chloroplasts, and mitochondria

(a) What is the lining of blood vessels made up of?


(b) What is the lining of small intestine made up of?
(c) What is the lining of kidney tubules made up of?
29.
(d) Where are the epithelial cells with cilia found?
OR
Why are xylem and phloem called complex tissues?

In a long distance race, the athletes were expected to take four rounds
of the track such that the line of finish was same as the line of start.
Suppose the length of the track was 200 m.
30. What is the total distance to be covered by the athletes ?
What is the displacement of the athletes when they touch the finish line
?
Is the motion of athletes uniform or non-uniform ?

Identical packets are dropped from two aeroplanes—one above the


equator and other above the north pole, both at height h. Assuming all
31.
conditions to be identical, will those packets take same time to reach
the surface of earth?

Look at the activities listed below.


32. Reason out whether or not work is done in the light of your
understanding of the term ‘work’
KENDRIYA VIDYLAYA SANGATHAN::: RANCHI REGION

(i) Suma is swimming in a pbnd.


(ii) A donkey is carrying a load on its back.
(iii) A wind-mill is filling water from a well.
(iv) A green plant is carrying out photosynthesis.
(v) An engine is pulling a train.
(vi)Food grains are getting dried in the sun.
(vii) A sailboat is moving due to wind energy.
OR
A cyclist comes to a skidding stop at 50 m. During this process, the
force on the cycle due to the road is 1000 N and is directed opposite to
the motion. How much work does the road do on the cycle?

(a) The sound of which of the following frequencies are audible to


33. human ears : 2 Hz, 5 Hz, . 20 Hz, 200 Hz, 2000 Hz.
(b) How [paths of, certain families are able to escape capture ?

Explain about the following statements:


(a) Evaporation causes cooling.
(b) Rate of evaporation of an aqueous solution decreases with increase
in humidity.
(c) Sponge though compressible is a solid.
34.
(d) Ice is solid at 0°C, while water is liquid at room temperature.
(e) Sugar crystals dissolve faster in hot water than cold water.
OR
What is evaporation? Why does evaporation cause cooling? Also
explain factors affect the rate of evaporation.

What is a neuron? Write the structure and functions of a neuron.


OR
35.
Differentiate between sclerenchyma and parenchyma tissues. Draw
well labelled diagram.

36. Enumerate the advantages of mixed framing.

The composition of two atoms A and B is given :

37.

(a) What are the mass numbers and atomic numbers of A and B ?
(b) What is the relation between the two chemical species ?
(c) Which element or elements do they represent ?

All living Organisms are made up of cells and these cells perform all the
functions essential for the survival of the Organism eg. Respiration,
38. digestion, excretion etc. In Unicellular organisms, a single cell carries
out all these functions and in multicellular organisms different group of
cells carry out different functions.
KENDRIYA VIDYLAYA SANGATHAN::: RANCHI REGION

Cells were first discovered by Robert Hooke in 1665. He observed the


cells in a cork slice with the help of a primitive microscope.
Leeuwenhoek (1674), with the improved microscope, discovered the
free living cells in pond water for the first time. It was Robert Brown in
1831 who discovered the nucleus in the cell. Purkinje in 1839 coined
the term ‘protoplasm’ for the fluid substance of the cell.
(1) Who discovered the cell?
(2) Who discovered the nucleus in the cell?
(3) Who coined the term ‘Protoplasm’?
(4) What is protoplasm?

There is no atmosphere on the moon. This is because gas molecules


need a certain amount of force of attraction to be retained on a
heavenly body. The force of attraction of the moon is less than the
required force, hence no atmosphere can exist.

(i) The value of g on moon is …….. times that of earth


(a) 1/3 (b) 1/4 (c) 1/5 (d) ⅙
39. (ii) Mass of the moon is ………. that of earth
(a) more than (b) less than
(c) equal to (d) can’t Day
(iii) If the weight of an object is 60 kg f on earth then, its weight on
moon is …….
(a) 10 kg f (b) 20 kg f
(c) 30 kg f (d) 40 kg C
(iv) who coined the term gravitation?
KENDRIYA VIDYLAYA SANGATHAN::: RANCHI REGION

Marking Scheme
Class : IX Sub:
Science
S No Answers Marks

1. (c) Evaporation, diffusion, expansion of gases 1

2. Alloy 1

3. N2 1

4. (b) 2, 8, 2 1

5. (d) (ii) and (iv) 1

6. (d) 1

7. (c) (iii) and (iv) 1

8. (c) DNA and protein 1

9. (b) (ii) and (iii) 1

10. (c) Conduction of water 1

11. (c) 2r 1

12. (b) always act on different bodies in opposite directions 1

13. (c) any two bodies having some mass 1

14. (d) universal gravitational constant 1

15. (c) remains constant 1

16. (c) radio waves 1

17. (a) 1

-b) Both Assertion and Reason are correct, and Reason is not the correct
18. 1
explanation for Assertion.

19. (d) 1

20. D 1

The water vapour present in the air comes in contact with cold surface
of the glass, loses its energy and gets converted into droplets of water.
21. 2
Or
Region C

22. It states that, ‘Mass is neither created nor destroyed in a chemical 2


KENDRIYA VIDYLAYA SANGATHAN::: RANCHI REGION

reaction.’ In other-words, the mass of the reactants must be equal to


the mass of products.

Drawbacks of Dalton’s Atomic Theory :


(i) According to modern theory, atom is not the ultimate indivisible
particle of matter. Today, we know that atoms are divisible, i.e., they
23. are themselves made-up of particles (protons, electrons, neutrons, 2
etc.).
(ii) In case of isotopes of an element, the assumption that the atoms of
the same element have same mass does not hold good

When salt is added, a hypotonic medium is created, i.e., the


concentration of salt molecules is more outside the vegetables than
24. inside. Hence, due to osmosis water from the vegetables come out. 2
Or
Mitochondria, Plastids.

1 newton is the magnitude of force which produces an acceleration of 1


25. 2
m/s2 in a body of mass 1 kg.

26. 2

(a). The water will cool initially till it reaches 0°C, the freezing point. At
27. this stage, the temperature will remain constant till all the water will 3
freeze. After this, temperature would fall again

(a) Functions of Golgi bodies:

Carbohydrate synthesis of mucopolysaccharides


Formation of acrosome
Formation of the lysosome.
Formation of the plasma membrane.
Formation of the cell wall.
Absorption of compounds.
Production of hormones.
28. Formation of pigments. 3
Yolk deposition.
(b) Functions of chloroplast:

Their main function is to trap the sun’s energy and to convert it into the
chemical energy of food by photosynthesis.
Storage of starch,
Chloroplasts in fruits and flowers change into chromoplasts.
(c) Functions of Mitochondria:
KENDRIYA VIDYLAYA SANGATHAN::: RANCHI REGION

Powerhouses the cell and stores energy as ATP.


Several respiratory enzymes are found in mitochondria.
DNA is also contained in mitochondria.
They regulate the concentration of calcium ions in the cells.

(a) Squamous epithelium


(b) Columnar epithelium
29. 3
(c) Cuboidal epithelium
(d) Respiratory tract

Xylem and phloem are called as complex tissues as they are made up of
more than one type of cells. Following are the differences between
xylem and phloem:

Xylem Phloem
30. 3
1. Xylem mainly consists of dead cells (except xylem parenchyma). 1.
Phloem consists of living cells (except phloem fibre).
2. It conducts water and minerals from roots to aerial parts of the plant.
2. It translocates prepared food from leaves to storage organs
and growing parts of the body

Total distance to be covered in four rounds = 4 x length of track = 4 x


200 = 800 m.
Since initial and final positions of athletes are same, so displacement of
31. 3
athletes = 0.
Motion of athletes is non-uniform as they cover unequal distances in
equal intervals of time.

The value of ‘g’ at the equator of the earth is lesser than that at poles.
Therefore, the packets fall slowly at equator in comparison to the poles.
32. 3
Thus, the packets will remain in air for longer time interval, when it is
dropped at the equator.

i) Work is done because the displacement of swimmer takes place in the


direction of applied force.
(ii) If the donkey is not moving, no work is done as the displacement of
load does not take place in the direction of applied force.
(iii) Work is done, as the displacement takes place in the direction of
force.
(iv) No work is done, because no displacement takes place.
33. (v) Work is done, because displacement takes place in the direction of 3
applied force.
(vi)No work is done, because displacement does not take place.
(vii)Work is done because displacement takes place in the direction of
the force
Or
Given,
Displacement, s = 50 m
KENDRIYA VIDYLAYA SANGATHAN::: RANCHI REGION

Force, F = – 1000 N
Work done, W = F x s
W = -1000 x 50J
W = 50000J

(a) Evaporation produces cooling as the particles at the surface of the


liquid gain energy from the
surroundings and change into vapour, thereby producing a cooling
effect.
(b) Air around us cannot hold more than a definite amount of water
vapour at a given temperature which is known as humidity. So, if the air
is already rich in water vapour, it will not take up more water;
therefore, rate of evaporation of water will decrease.
(c) A sponge has minute holes in which air is trapped. Also the material
is not rigid. When we press it, the air is expelled out and we are able to
compress it. But it is a solid because it has definite shape and volume
and does not change its shape unless compressed.
(d) Ice is solid at 0°C because it has a definite volume and definite shape
due to strong intermolecular forces. Water is liquid at room
temperature because it has definite volume and no definite shape due
to weak intermolecular forces of attraction.
(e) Sugar crystals dissolve faster in hot water than cold water because
hot water molecules have more kinetic energy. Due to this, they strike
34. faster on the particles of sugar than cold water molecules. As a result, 5
hot water will dissolve them faster than cold water.
Or
The process in which a liquid changes into its vapour state at a
temperature below the boiling point is called evaporation. Evaporation
is an endothermic process i.e., the liquid absorbs heat during
evaporation. This heat may be provided either by the surrounding or by
the liquid itself. When the evaporating liquid takes the required heat
from other parts of the liquid, the rest of the liquid cools down.

On the other hand, if the liquid takes heat from the surroundings, it
causes cooling of the surroundings. For example, on a hot day (sunny
day) we perspire. When this sweat evaporates, it absorbs the required
heat from our body, and we feel cool.Factors that affect the rate of
evaporation are:

Temperature: Evaporation increases with increase in temperature.


Humidity: Evaporation decreases with an increase in humidity.
Wind speed: Evaporation increases with an increase in wind speed.

Nervous tissue contains highly specialised unit cells called nerve cells or
neurons. Each neuron has the following three parts:
35. 5
The cyton or cell body: It contains a central nucleus and cytoplasm with
KENDRIYA VIDYLAYA SANGATHAN::: RANCHI REGION

characteristic deeply stained particles, called Nissl granules.


The dendrites: These are short processes arising from the cyton.
The axon: It is a single, long, cylindrical process of uniform diameter It
carries impulses away from the cell body.
(Proper diagram)
Or

Following are the main advantages of mixed farming:

Farmyard manure is made available from livestock which is used again


in agricultural farms.
Organic waste materials like straw, husks and chaffs of grains,
household kitchen waste, etc., are converted into human food through
the agency of cattle, sheep, poultry, pigs, etc., as per the choice of
36. 5
farmer.
It provides work to all the members of a family throughout the year,
thus providing subsidiary occupation without the need of employing
special labour.
Adopting exact combination in mixed farming, income can be increased,
e.g., the number of animals can be increased (as per the food/crop
available) to enhance milk production

(a) Mass number of A = 17 + 18 = 35 u


Mass number of B = 17 + 20 = 37 u
37. (b) The two chemical species exist as pair of isotopes since they have 4
the same number of protons and electrons.
(c) They represent the element chlorine (Cl).

1. Robert Hooked
2. Robert Brown's
38. 4
3. Purkinje
4. Fluid substance of the cell

(d)
(b)
39. 4
(a)
Newton.
KENDRIYA VIDYLAYA SANGATHAN::: RANCHI REGION

KENDRIYA VIDYALAYA SANGATHAN RANCHI REGION


CLASS IX
SAMPLE QUESTION PAPER
(2022-2023)
Max.Marks:-80 Time allotted:- 3hours
General Instruction:-
• All questions are compulsory.
• The question paper has five sections and 39 questions.
• Section A- has 20 questions carrying 1 marks each;
• Section B- has 6 questions carrying 2 marks each;
• Section C- has 7 questions carrying 3 marks each;
• Section D- has 3 questions carrying 5 marks each;
• Section E- has 3 questions carrying 4 marks each;

SECTION- A

1. What is the physical state of water at:


(a) 25oC (b) 0oC
2. Define the true solution?
3. What is valency?
4. What is an ion? Give an example.
5. The mass number of chlorine atom is 35 and its atomic
number is 17. How will this chlorine atom be
represented?
6. Write valency of Sulphur and Oxygen?
7. Name the longest animal cell?
8. Define cell?
9. What are the function of the stomata?
KENDRIYA VIDYLAYA SANGATHAN::: RANCHI REGION

10. A ball is moving over a horizontal smooth surface


with a constant velocity. What type of forces are acting
on the ball?
11. Give the value of universal gravitational
constant(G) in SI units?
12. A piece of ice floats on water. Why?
13. Name and define the SI unit of work?
14. What do you mean by sound?
15. Name the tissue which joins:
(a) Muscles to bones
(b) Bones to bones.
16. What is the SI unit of acceleration?
Directions: For question numbers 17 to 20, two statements
are given- one labeled Assertion (A) and the other labeled
Reason (R). Select the correct answer to these questions
from the codes (a), (b), (c) and (d) as given below:
(a) Both (A) and (R) are true and (R) is the correct
explanation of the assertion (A).
(b) Both (A) and (R) are true but (R) is not the correct
explanation of the assertion (A).
(c) (A) is true but (R) is false.
(d) (A) is false but (R) is true.
17. Assertion (A): Atoms of inert gases exist
independently.
Reason (R) : It is possible to feel and see atoms.
18. Assertion (A): Heart has voluntary muscles called
cardiac muscles.
Reason (R) : The muscles of heart show rhythmic
contraction and relaxation throughout the life.
KENDRIYA VIDYLAYA SANGATHAN::: RANCHI REGION

19. Assertion (A): A light body and a heavy body have


same momentum. They also have same kinetic energy.
Reason (R) : Kinetic energy does not depend on
mass of the body.
20. Assertion (A): Capture fishery is the method of
obtaining fish from natural resources.
Reason (R) : Culture fishery is the method of
obtaining fish by fish farming.

SECTION-B

21. Why is ice at 273K more effective in cooling than


water at the same temperature?
OR
What produces more severe burns, boiling water or
steam?
22. Define atomicity? Give an example each of
monoatomic and diatomic molecules.
23. What is difference between 2H and H2?
24. What is the momentum of a man of mass 75kg,
when he walks with a uniform velocity of 2m/s?
25. When a carpet is beaten with a stick, dust comes
out of it. Explain.
26. Why is the cell called the structural and functional
unit of life?
OR
What is endocytosis? Give one example.
KENDRIYA VIDYLAYA SANGATHAN::: RANCHI REGION

SECTION-C

27. What is matter? List three characteristics of


particles of matter.
28. What is endoplasmic reticulum? Write its main
functions.
29. List four elements of phloem. Which of them is
most important and why?
OR
What is xylem? Which one of its component is very
important and why?
30. Define the term speed. What are its various units?
Is it a scalar or vector quantity?
31. An iron nail sinks in water, while a large ship floats
on the surface of water. Give reason.
32. Name the different forms of energy. What is
mechanical energy? What are its two forms?
OR
A bullet of mass 10g is fired with a velocity of
80m/s. What is its kinetic energy?
33. Define the terms amplitude, time period and
frequency. Give their SI units.
KENDRIYA VIDYLAYA SANGATHAN::: RANCHI REGION

SECTION-D

34. (i) Compare metals and non- metals based on their


physical properties.( any two points)
(ii) What are metalloids? Give two examples.
(iii) Identify metals from the following:
Boron, Sodium, Mercury, Carbon.
OR
(i)Differentiate between a saturated solution and
an unsaturated solution.
(ii) A solution contains 40g of common salt in 320g
of water. Calculate the concentration in terms of mass
by mass percentage of the solution.
35. Draw a labelled diagram of shoot to show location
of meristematic tissues in the plant body and write the
functions of each of them.
OR
(i)What is nervous tissue?
(ii) Draw a labelled diagram of neuron and write
the functions of dendrites and axon.
36. (a) What do you mean by mixed cropping. Give an
example.
(b)What are weeds? Give example.
KENDRIYA VIDYLAYA SANGATHAN::: RANCHI REGION

SECTION-E

Read the paragraph and answer the following


questions(i-iv):-

37. (I) We know that protons are present in the


nucleus of an atom. It is the number of protons of an
atom which determines its atomic number and hence
identity. After studying the properties of subatomic
particles it is concluded that protons together with
neutrons reside in the nucleus and this determines the
mass of the atom. The number of protons and neutrons
together are known as nucleus.

Answer the following questions:


(i)Where do the electrons reside?
(ii) Where is the positive charge of the atom situated?
(iii) What are such atoms called which have same
number of protons but different number of nucleons?
(iv) What are such atoms called which have same
number of nucleons but different number of protons?
OR

(II) Atom is a neutral particle. The study of static


electricity and the fact that electricity is conducted by
charged particles revealed that an atom is divisible and
consists of charged particles. In 1900 J.J.Thomson
identified the presence of one sub-atomic particle. It
was called electron. Even before the electron was
KENDRIYA VIDYLAYA SANGATHAN::: RANCHI REGION

identified, E.Goldstein in 1886 discovered the


emanation of positively charged radiations from the
atoms. This led to the discovery of another sub-atomic
particles which had a charge equal in magnitude but
opposite in sign to that of the electron. It was given the
name proton. It was also found that the electron could
be easily peeled off but not protons.

Answer the following questions:


(i)What makes removal or peeling of electrons easier
but not of protons?
(ii) Positive charge is formed when an electron or more
electrons are removed from an atom. Is the negative
charge formed by removal of a proton?
(iii) What sub-atomic particles are involved in the
conduction of electricity through metals?
(iv) What particles make the conduction through
solutions possible?

38. (I) Cork is a substance which comes from the bark


of a tree. This was in the year 1665 when Robert Hooke
made this chance observation through a self- designed
microscope. He called honeycomb like boxes “cells”. Cell
is a latin word for “a little room”.

Answer the following questions:


(i)What is the source of cork? Is it living or dead? What
is the difference between cork and bark?
KENDRIYA VIDYLAYA SANGATHAN::: RANCHI REGION

(ii) Who discovered cells for the first time and in which
year? How did Robert Hooke observe cells?
(iii) What is the latin word for “a little room”?
(iv) How is cork formed and what is its function?
OR

(II) Cell organelles are enclosed by membranes. In


prokaryotes, beside the absence of a defined nuclear
region, the membrane-bound cell organelles are also
absent. On the other hand, the eukaryotic cells have
nuclear membrane as well as membrane enclosed
organelles.

Answer the following questions:


(i)What encloses cell organelles?
(ii) Give two features of prokaryotes. Write examples of
prokaryotes.
(iii) Write two characters of eukaryotes. Give their
examples.
(iv) How prokaryotes are different from eukaryotes?

39. (I) When a body is placed on a surface, it exerts a


force on the surface equal to its own weight. This total
force is called thrust. When we press a drawing pin, we
apply a force on the surface area of its heat. The force
acting on the pin perpendicular to its surface is the
thrust. When we stand on loose sand, our feet go deep
into the sand. When we lie down on the sand, our body
will not go deep in the sand. In both cases, force exerted
KENDRIYA VIDYLAYA SANGATHAN::: RANCHI REGION

on the sand is same but the force acting per unit area or
pressure is different. So we find different effects.

Answer the following questions:


(i)Define the term thrust?
(ii) Define the term pressure?
(iii) What is the SI unit of pressure?
(iV) A sharp knife cuts better than a blunt one. Why?
OR

(II) It is a common experience that a mug filled


with water appears to be heavier when it is lifted above
the surface of water in a bucket. Inside water, the mug
appears to lose some weight. This apparent loss in
weight is due to the upward force exerted by water. This
force is known as buoyancy. The magnitude of the force
of buoyancy is equal to the weight of fluid displaced by
it. This statement is known as Archimedes’ principle.

Answer the following questions:


(i)What do you mean by buoyancy?
(ii) When does an object float when placed on the
surface of water?
(iii) State Archimedes’ principle.
(iv) Why does a block of plastic released under water
come up to the surface of water?
KENDRIYA VIDYLAYA SANGATHAN::: RANCHI REGION

KENDRIYA VIDYALAYA RANCHI REGION


SAMPLE QUESTION PAPER(2022-23)
SUBJECT: SCIENCE
CLASS- X
ANSWER KEY MM:80

Q. ANSWER MARKS
NO.
SECTION-A
1 (a)Liquid (b) solid 1
2 Write definition 1
3 Valency is the combining capacity of an 1
element.
4 An ion is formed when an atom loses or 1
gains one or more electrons. Eg- cation
and anion
5 35
Cl(mass) , 17Cl(atomic no) 1
6 Electronic configuration of sulphur is 1
2,8,6;
Valency- 2. Oxygen- 2,6; valency-2
7 Neuron 1
8 Correct explanation 1
9 Exchange of gases in leaf (CO2 & O2) 1
10 Balanced force 1
11 6.67x10-11Nm2/kg2 1
12 The density of ice is less than that of 1
water. So ice floats on water.
13 Define work, SI unit of work is joule(J) 1
KENDRIYA VIDYLAYA SANGATHAN::: RANCHI REGION

14 Sound is a form of energy which 1


produces a sensation of hearing in our
ears.
15 (a)Tendon (b)Ligament 1
16 SI unit of acceleration is m/s2 1
17 (c) 1
18 (d) 1
19 (d) 1
20 (b) 1
21 Ice at 273K is less energetic than water, 2
the difference being the latent heat of
fusion. OR
Steam is more energetic than boiling
water, can produce severe burns.

22 The number of atoms combined together 2


to form a stable molecule of an element
is called its atomicity.
Eg- Monoatomic- He
Diatomic- O2
23 2H means two atoms of hydrogen 2
whereas H2 means on molecule of
hydrogen.
24 m=75kg, v=2 m/s 2
momentum, p=mv=75 kg x 2 m/s=
150kg m/s
25 Write correct explanation 2
26 Write correct explanation OR 2
Write correct explanation, eg- Amoeba
27 Matter is anything that has mass and 3
occupies space. Write characteristics
KENDRIYA VIDYLAYA SANGATHAN::: RANCHI REGION

28 Write correct explanation and function 3


29 Elements of phloem: 3
(i)Sieve tubes (ii) Companion cells
(iii) Phloem parenchyma (iv) Phloem
fibres
Most imp. Element- Sieve tubes
OR
Xylem is a complex plant tissue which
transports water and dissolved minerals
from roots to all other plant parts.
Most imp. Element of xylem is vessel.
30 Speed= Distance travelled/ Time taken 3
SI unit of speed is ms-1; speed is scalar
quantity
31 Write correct answer with reason 3
32 Mechanical energy, sound energy, heat 3
energy, light energy, chemical energy;
Mechanical energy- (i) Kinetic energy
(ii) Potential energy OR

m=10g=1/100 kg, v=80 m/s


K.E.= 1/2mv2=1/2 x 1/100 x (80)2= 32 J
33 Write correct explanation; SI unit of 3
amplitude is metre(m); SI unit of time is
second(s); SI unit of frequency is
hertz(Hz)
34 (i)Write any 2 points 2+2+1=5
(ii) Metalloids are the elements which
have properties intermediate between
those of metals and non-metals. Eg-
boron, silicon
KENDRIYA VIDYLAYA SANGATHAN::: RANCHI REGION

(iii) metal- sodium & mercury


OR
(i) Write correct answer
(ii)mass of solute (salt)= 40g 2+3=5
Mass of solvent(water)=320g
Mass solution= 40g + 320g= 360g
Mass percentage of
solution=40/360x100= 11.1%
35 Draw diagram and label it; Write their 3+2=5
function OR
(i) Write correct answer
(ii)Draw labelled diagram and write 2+3=5
function

36 (a)mixed cropping- growing two or more 2.5+2.5=5


crops simultaneously in the same piece
of land.eg- wheat+gram; groundnut+
sunflower
(b)weeds- undesirable plants. Eg-
Xanthium, cyperinus rotundus

37 (I)-(i)In the extranuclear part 1+1+1+1=4


(ii)In the nucleus
(iii)Isotopes
(iv)Isobars OR
(II)-(i)The protons are densely populated
in the interior of the atom whereas
electrons are sparsely located around the
nucleus.
1+1+1+1=4
KENDRIYA VIDYLAYA SANGATHAN::: RANCHI REGION

(ii)Protons are never removed from the


atom. Negative charge is formed by
addition of electrons.
(iii)Electrons
(iv)Movement of positive and negative
ions in the solution.
38 (I)-(i)Bark of a tree. It is dead tissue. 1+1+1+1=4
Write difference
(ii)Robert Hooke, in 1665. He observed
cells through a self designed microscope.
It was chance observation.
(iii)Cell
(iv)Corks is formed during secondary
growth from cork cambium. It protects
the internal parts of the plant.
OR
(II)-(i)Cell membrane
(ii)Prokaryotes lack membrane- bound 1+1+1+1=4
cell organelles;
Nuclear region is poorly defined in
prokaryotes. Eg- Bacteria and
Cyanobacteria.
(iii)Eukaryotes have nuclear membrane
around chromatin material so nucleus is
well defined;
Cell organelles are enclosed in
membrane in eukaryotes. Eg- man, tulsi,
banyan tree, etc.
(iv) write differences
KENDRIYA VIDYLAYA SANGATHAN::: RANCHI REGION

39 (I)-(i)The total force exerted by a body on 1+1+1+1=4


any surface in contact with it is called
thrust.
(ii)The thrust acting per unit area is called
pressure.
(iii)SI unit of pressure is newton per
square metre Nm-2.
(iv)sharp edge- less area and more force
and pressure ; blunt edge- more area and
less force and pressure. Hence a sharp
edge cuts better.
OR
(II)-(i)The upward force acting on a body
immersed in a fluid is called buoyancy. 1+1+1+1=4
(ii)An object floats on water when its
density is less than that of water.
(iii)When a body is immersed fully or
partially in a fluid, it experiences an
upward thrust equal to the weight of the
fluid displaced by it is called Archimedes’
principle.
(iv)The upthrust or the buoyant force
exerted by water brings the plastic block
to the surface of water.
KENDRIYA VIDYLAYA SANGATHAN::: RANCHI REGION

KENDRIYA VIDYALAYA SANGTHAN


RANCHI REGION
Sub – Science Class – IX.
Max. Marks – 80 Time – 3Hrs.
(SET-1)
General Instructions:
i. This question paper consists of 39 questions in 5 sections.
ii. All questions are compulsory. However, an internal choice is provided in some
questions. A student is expected to attempt only one of these questions.
iii. Section A consists of 20 objective type questions carrying 1 mark each.
iv. Section B consists of 6 Very Short questions carrying 02 marks each. Answers
to these questions should in the range of 30 to 50 words.
v. Section C consists of 7 Short Answer type questions carrying 03 marks each.
Answers to these questions should in the range of 50 to 80 words
vi. Section D consists of 3 Long Answer type questions carrying 05 marks each.
Answer to these questions should be in the range of 80 to 120 words.
vii. Section E consists of 3 source-based/case-based units of assessment of 04 marks each
with sub-parts.

SECTION – A
Select and write one most appropriate option out of the four options given for each of
the questions 1 to 20.
Q. No Question Marks
1 Which among the following can termed as fluid? 01
a) sulphur b) carbon c) oxygen d) phosphorus
2 Which one of the following is not a metalloid? 01
a) boran b) silicon c) gallium d) germanium
3 The correct symbol for silver is 01
a) Ag. b)Si c) Ar d) Al
4 The charge on an electron is equal to 01
a) 1.6 ×10-19 C of negative charge
b) 1.6×10-19 C of negative charge
c) 1.6×10-19 C of negative charge
d) 1.6×10-19 C of negative charge
5 The first model of an atom was given by 01
a) N. bohr b) E. Goldstein c) Rutherford d) J.J Thomson
6 An atom with three protons and 4 neutrons will have a valency of 01
a) 3 b) 7 c) 1 d) 4
7 Who proposed the cell theory 01
a) Schleiden and Schwann
b) Watson and Crick
c) Dorwin and Wallace
d) Mendel and Morgan
8 Nucleus was discovered by 01
KENDRIYA VIDYLAYA SANGATHAN::: RANCHI REGION

a) Robert brown b) Robert hooke c) A.V. leeuwenhoek


d) Schwann
9 Synapse is the junction of 01
a) dendrites b) nerve cells c) nucleus d)
Axon

10 Which of the following helps in translocation of food in plants 01


a) xylem
b) phloem
c) sclerenchyma
d) collenchyma
11 If displacement of a particle is zero distance covered by it 01
a) may be zero or may not be zero
b) must be zero
c) must not be zero
d) all of the above
12 Newton's second law measures the 01
a) acceleration b) force c) momentum d) angular
momentum
13 The atmosphere is held to the earth by 01
a) winds b) clouds c)gravity d) none
14 Which is not a unit of energy 01
a) watt second
b) kilo watt hour
c) watt
d) joule
15 Infra sound can be heard by 01
a) dog
b) bat
c) rhinoceros
d) human beings
16 Upthrust on a body depends on 01
a) density of the liquid
b) volume of the body
c) density of the body
d) both b and c
Q. no 17 to 20 are Assertion - Reasoning based questions.
These consist of two statements – Assertion (A) and Reason (R). Answer these questions
selecting the appropriate option given below:
(a) Both A and R are true and R is the correct explanation of A
(b) Both A and R are true and R is not the correct explanation of A
(c) A is true but R is false
(d)A is False but R is true
17 Assertion: an atom is electrically neutral. 01
Reason: Atom contains equal number of protons and neutrons.
18 Assertion higher plants have meristematic regions for indefinite growth. 01
Reason: Higher plants have root and shoot apices possible for this growth.
19 Assertion: power developed in circular motion is always zero. 01
Reason: Work done in case of uniform circular motion is not zero.
KENDRIYA VIDYLAYA SANGATHAN::: RANCHI REGION

20 Assertion: Agriculture output increased several times after introduction of 01


DDT.
Reason: DDT was the first insecticide use on a wide scale.
SECTION - B
Question number 21 to 26 are very short answer questions.
21 Name the three States of matter. Gives one example of each. 02
Or
What is sublimation? Give examples.
22 Name the following compounds PCl3 and SO2. 02
23 What is Avogadro's number? 02
24 Give two examples of prokaryotic cell. 02
Or
Define cell. Who coined the term cell?
25 It is easier to roll a barrel than to pull it along the road. Why? 02
26 A retarding force is applied to stop a motor car. Is the speed of the motor car 02
is doubled, how much more distance will it cover before stopping under the
same retarding force?

SECTION-C
Question number 27 to 33 are short answer questions and each question carry
three marks.
27 What is condensation? How is the condensation of a gas carried out? 03
28 a) Is the cell wall in a plant cell living or nonliving? 03
b) Is the cell wall in a plant cell permeable or selectively permeable?
c) What is its chemical composition?
29 Give three features of cardiac muscle. 03
Or
Give three differences between bone and cartilage.
30 What is the acceleration of a particle moving with uniform velocity? 03
31 A driver increases the speed of his car on approaching a hilly road. Why is it 03
done?
OR
Water at the bottom of waterfall is warmer than at the top. Why?
32 State the factors on which upthrust or buoyant force depends. 03
33 Before playing the Orchestra in a musical concert, a Sitarist tries to adjust the 03
tension and pluck the string suitably. By doing so, what is he adjusting?
SECTION- D
Question number 34 to 36 are long answer questions each question carry 5 marks.
34 How are true solution colloidal solution and suspension different from each 05
other?
OR
Explain in detail about classification of solution on the basis of amount of
solute.
35 What is the skeletal connective tissue? Give its functions. 05
OR
Explain tendons and ligaments.
36 What happens due to deficiency of nutrients? 05
Or
KENDRIYA VIDYLAYA SANGATHAN::: RANCHI REGION

Can increasing grain production alone solved the problem of malnutrition


and hunger?
SECTION-E
Question number 37 to 39 are case-based/data-based questions with 2 to 3 short sub-parts.
Internal choice is provided in one of these sub parts. Each question carry 4 marks.
37 Dalton's atomic theory suggested that the Atom was indivisible and 04
indestructible. But the discovery of two fundamental particles electron and
protons inside the atom, led to the failure of this aspect of Dalton's atomic
theory. It was then considered necessary to know how electrons and protons
are arranged within an atom. For explaining this, many scientists proposed
various atomic models. J. J. Thomson was the first one to propose a model
for the structure of an atom.

Thomson was a British physicist he was awarded the Nobel prize in physics
for his work on the discovery of electrons. Thomson proposed the model of
an atom to be similar so to that of a Christmas pudding. The electrons in a
sphere of positive charge. We can also think of a watermelon, the positive
charge in the atom is spread all over like the red edible part of the watermelon,
while the electrons are studied in the positively charged sphere, like the seeds
in the watermelon. Thomson proposed that: an atom consist of a positively
charged sphere and the electrons are embedded in it. The negative and
positive charges are equal in magnitude. So, the atom as a whole is electrically
neutral.

1) Identify the correct statement.

Statement 1- Dalton's atomic theory suggested that the Atom was indivisible
and indestructible.

Statement 2- electrons and protons are present inside the atom.


Statement 3- Thomson was the first one to propose a model for the structure
of an atom.

Statements 4- protons are positively charged particle.

a) Only 2
b) both 3 and 4
c) both 1 and 2
d) all of the above

2) According to Dalton's atomic theory matter consist of indivisible

a) molecules
b) atoms
c) ions
d) mixtures

3) Who was the first to propose atomic theory?

a) Thomson
KENDRIYA VIDYLAYA SANGATHAN::: RANCHI REGION

b) John Dalton
c) Rutherford
d) bohr

4) "Atom is indivisible and indestructible" why this aspect of Dalton's atomic


theory leads to the failure?
OR
Explain the Thomson's model for the structure of an atom?
38 Chloroplasts are green coloured plastid. The green colour is due to the 04
presence of chlorophyll pigment. Each chloroplast is bounded by two unit
membranes chlorophyll traps the solar energy which is used in manufacturing
of food.

1) Where is the chloroplast located?


2) Write function of chloroplast.
3) Name the site where solar energy is trapped?
OR
In chloroplast protein synthesis will occur or not. Give reason to support your
answer.
4) Where chloroplast are present?
39 Mass is defined as the amount of matter in an object. The mass remains the 04
same whether the object is on the earth, moon or outer space. The weight of
body is the force with which it is attracted towards the centre of earth. Weight
depends on location of object from the centre of earth.

1) What is the weight of a 2kilogram object on the earth. ( g=9.8 m/s2).


a) 9.8 N
b) zero
c) infinite
d) 19.6 N

2) A body has a weight 60 Newton on the surface of earth. What will be its
weight when taken to the moon?
a) 10N
b) zero
c) infinite
d) 5 N

OR
On which factor the weight of the object will depend upon?

***************
KENDRIYA VIDYLAYA SANGATHAN::: RANCHI REGION

SCIENCE (086)
CLASS IX
MARKING SCHEME (2022-23)
SECTION-A
Q. Questions Marks
No
1 1) c) oxygen 01
2 c) gallium 01
3 a) Ag 01
4 a) 1.6 × 10-19 C of negative charge. 01
5 d) J.J Thomson 01
6 c) 1 01
7 a) Schleiden and Schwann 01
8 a) Robert brown 01

9 b) nerve cells 01
10 b) phloem 01
11 a) when displacement is zero particle may be at rest ,therefore, distance 01
travelled zero, again when displacement is zero final position coincides
with the initial position distance travel is not zero.
12 b) force 01
13 c) gravity 01
14 c) watt 01
15 c) rhinoceros 01
16 d) both b and c 01
17 c) an atom is electrically neutral atoms necessarily contain equal number of 01
protons and electrons but not neutrons.
18 b) both assertion and reason are true but reason is not the correct explanation 01
of the assertion.
19 c) if assertion is correct but reason is incorrect. Work done and power 01
developed is zero in uniform circular motion only.
20 b) both assertion and reason are true and reason is not the correct 01
explanation of assertion.
SECTION -B
21 Solids, liquids and gases are three States of matter and their examples are 02
ice, water and steam respectively.
OR
Sublimation is a phenomenon of interconversion of solid into gases and vice
versa. Example: Solid CO2 gets directly converted into gases form.
22 Phosphorus trichloride and Sulphur dioxide. 02
23 Gram atomic mass of an element contains the same number of atoms of that 02
element as their carbon atoms in exactly 12 g of carbon-12. This number is
Avogadro's number that is 6.023×10^23.

24 Bacteria and blue green algae. 02


OR
Cell is the structural and functional unit of life. 'Robert Hooke' coined the
terms cell.
KENDRIYA VIDYLAYA SANGATHAN::: RANCHI REGION

25 The rolling friction is lesser as compared to the sliding friction. 02


26 Since distance is directly proportional to the square of velocity, motorcar 02
will cover the distance four times longer than before.
SECTION-C
27 The process due to which a gas changes into liquid state by giving out heat 03
energy is called condensation. When a gas is compressed it loses its heat
energy there by consequently decreasing intermolecular distance between
molecules and increasing intermolecular forces of attraction between
molecules. Thus, resulting into liquefaction of gas.

28 a) plant cell wall is considered as non-living due to the presence of 03


cellulose.
b) It is permeable because it is made of cellulose which is permeable to
water, solutes and gases.
c) plant cell wall is chiefly composed of insoluble fibrous polysaccharide,
cellulose carbohydrate, compounds hemi- cellulose ,pectin and protein also
occur in the cell wall.

29 i) cells- they are small cylindrical, uni-nucleate is striated with short lateral 03
branches.
ii) intercalated disc-in the area of union between the two adjacent cardiac
muscle fibres, zig-zag junction is present, called intercalated disc. The
intercalated disc function as impulse booster.
iii) rhythmic contraction-the muscles are involuntary and non-fatigued
which continue to contract and relax tirelessly throughout the life.
OR
Bones- a) hard, in flexible and porous.
b) blood vessel present.
c) matrix made up of proteins.
Cartilage
a) flexible, not very hard and porous.
b) blood vessels absent.
c)Matrix made up of proteins and minerals salt.
30 zero is the acceleration of a particle moving with uniform velocity. 03
31 when a car is moving on a flat road, it has to do work to overcome the 03
friction of the road and air resistance but no work is done against the force
of gravity. On the other hand when the car is going up the hill, then in
addition to friction and air resistance it has to do work against the force of
gravity. Thus, driver increases the speed of his car on an approaching a hilly
road to give more kinetic energy to the car so that it may go up against
gravity.

OR
The mechanical energy (K.E + P.E) of the falling water is converted into
heat energy when waterfalls on the ground. Due to this heat energy the
temperature of water at the bottom of waterfall increases.
32 a) size or volume of body immersed in a liquid. 03
b) density of the liquid in which the body is immersed. c) acceleration due
to gravity at the given place.
KENDRIYA VIDYLAYA SANGATHAN::: RANCHI REGION

33 He is adjusting frequency of the sitar string with the frequency of the other 03
musical instrument.
SECTION-D
34 True solution 05
a) they are clear and transparent appearance.
b) particles are not visible to naked eye.
c) particle size is less than 1 nanometer.
d) they diffuse quickly.
e) they do not settle.
f) they can pass through ordinary filter paper as well as animal membrane.

Colloidal solution
a) they are translucent appearance.
b) scattering of light by particle is observed under ultra microscope
visibility.
c) particle size is between 1 nanometer and 100nanometer.
d) the diffuse slowly.
e) they do not settle.
f) the pass through filter paper but not through animal membrane.

Suspension
a) they are opaque appearance.
b) particles are visible to naked eye.
c) particle size is more than hundred nanometer.
d) they do not diffuse.
e) they shuttle down on standing.
f) they do not pass through filter paper or membrane.

OR

Solutions can be classified into two types on the basis of the amount of
solute present in the solution we can classify them into the following types.
a) unsaturated solution- unsaturated solution is a solution in which more of
the solute can be dissolved at a given temperature. In this the addition of
solute is possible till the solution which is the point of saturation. Example
5 gram or 10 gram or 20 gram of NaCl in 100 g of water.
b) saturated solution- a solution in which no more solute can be dissolved
in a definite amount of solvent that a given temperature is called a saturated
solution. Example 36 gram of NaCl and 100 gram of water at room
temperature forms of saturated solution.

c) Super saturated solution-a solution which has more amount of solute than
the saturated solution at a given temperature is called super saturated
solution.
35 skeletal connective tissue is that connective tissue in which the matrix is 05
solid and living cells occur inside fluid filled spaces called lacunae. It is of
two types that is cartilage and bone.
Functions of skeletal connective tissues are as follow-
KENDRIYA VIDYLAYA SANGATHAN::: RANCHI REGION

a) endoskeleton-it forms the internal supporting framework of the animal


body.
b) protection-the tissue protects the vital organs like brain, spinal cord,
heart, lungs etc.
c) joints-the tissue forms joints which allow for growth and movement of
body parts.
d) muscles-it provide surface was attachment to the muscles.
e) blood cells-the firm red marrow of bones.
f) minerals-bony skeleton stores minerals sum of which are withdrawn by
the body in case of emergency.

OR
Connective tissues of animals solve the functions of binding and joining
one tissues to another and form protective sheath and pack material around
the various organs separating them. They do not interfere with each others
activities and carry materials from one part to another in the body and
formers supporting framework of cartilage and bones for the body etc.
Tendon- tendons are cord like, very tough inelastic bundles of white
collagen fibres bound together by areolar tissue. The cells present in the
tendons are elongated fibroblast which lie in almost continuous Rows here
and there. The tendons connect the skeletal muscles with the bones.
Ligaments-ligaments are cords formed by yellow elastic tissue in which
make collagen fibres are bound together by areolar tissue. The fibroblast
are irregularly scattered. This tissue combines with grade flexibility. The
ligaments of to bind the bones together. Both tendons and ligaments are
examples of dense regular connective tissue.
36 nutrients are required by the plants for maintaining their health and every 05
living process occurring in their bodies. Efficiency of nutrients affects the
various visual logical process in plants like reproduction growth,
susceptibility to disease etc. And can ultimately lead to the death of plants.
All over it effects all the life on the earth related to their survival needs as
plants will affect by the nutrients then after on the earth all the living
organism will affect and it will cause the survival of all the living organism
due to the deficiency of nutrients.

OR

No, increasing grain production only for storage in houses cannot solve the
problem of malnutrition and hunger. Food security depends both on other
availability of food and access to it. As the majority of our population
depends on agriculture for their livelihood, increasing the incomes of
people working in agriculture thus, becomes necessary to combat the
problem of hunger.

SECTION-E
37 1) d) all of the above 04

2) b) atoms
KENDRIYA VIDYLAYA SANGATHAN::: RANCHI REGION

3) b) John Dalton

4) Dalton's atomic theory suggested that the Atom was indivisible and
indestructible, but the discovery of two fundamental particles electrons and
protons inside the atom led to the failure of the aspect of Dalton's atomic
theory.

OR

Thomson was the first want to propose a model for the structure of an atom.
38 1) chloroplast is located in the mesophyll region of the leaves. 04
2) chloroplast performs the function to synthesize food by the process of
photosynthesis.

3) grana
OR
In chloroplast protein synthesis occurs because in it DNA and ribosomes
are present. Due to this it is able to synthesis its own proteins. Therefore it
is also called a semi- autonomous cell organelle.
4) chloroplast is present in plant cell only.
39 1) d) 19.6 N 04
Weight= m× g
= 2× 9.8
=19.6 N

2) a) weight of the body on the surface of moon will be 10 N as value of g


on the moon is one-sixth that of the earth.

OR
Weight depends on location of object from the center of earth.
KENDRIYA VIDYLAYA SANGATHAN::: RANCHI REGION

KENDRIYA VIDYALAYA SANGTHAN


RANCHI REGION
SAMPLE PAPER (2022-23)

Sub – Science Class – IX


Max. Marks – 80 Time - 3 Hrs.
General instructions:
i. This question paper consists of 39 questions in 5 sections.
ii. All questions are compulsory. However, an internal choice is provided in some
questions. A student is expected to attempt only one of these questions.
iii. Section A consists of 20 objective type questions carrying 1 mark each.
iv. Section B consists of 6 Very Short questions carrying 02 marks each. Answers
to these questions should in the range of 30 to 50 words.
v. Section C consists of 7 Short Answer type questions carrying 03 marks each.
Answers to these questions should in the range of 50 to 80 words
vi. Section D consists of 3 Long Answer type questions carrying 05 marks each.
Answer to these questions should be in the range of 80 to 120 words.
vii. Section E consists of 3 source-based/case-based units of assessment of 04 marks each
with sub-parts.

Q. No Question Marks

SECTION-A 1
Select and write one most appropriate option out of the flow options given
for each of the questions 1 to 20
1 Which of the following does not undergo sublimation process? 1
a) sodium chloride
b) ammonium chloride
c) Iodine
d) Naphthalene
2 Milk of magnesia is 1
a) a colloid
b) a true solution
c) a homogeneous mixture
d) a suspension
3 In water the proportion of Oxygen and hydrogen by mass is 1
a) 1:4
b) 1:8
c) 4:1
d) 8:1
4 Which of the following is not allotropes of carbon? 1
a) graphite
b) graphene
KENDRIYA VIDYLAYA SANGATHAN::: RANCHI REGION

c) fullerene
d) none
5 The atomic nucleus consists of which of the following pair? 1
a) electrons and protons
b) neutrons and electrons
c) protons and neutrons
d) nucleus and electrons
6 The element having Tetra atomic atomicity is 1
a) helium
b) nitrogen
c) Phosphorus
d) Chlorine
7 In the mitochondrion energy is stored in the form of 1
a) ATP
b) citric acid
c) AMP
d) ADP
8 The Golgi bodies are related to 1
a) respiration
b) excretion
c) secretion
d) circulation
9 Average lifespan of human RBC is 1
a) 100 days
b) 90 days
c) 120 days
d) none
10 Which tissue does black blood supply and hills slowly? 1
a) Nerve cells
b) muscles
c)Cartilage
d) bones

11 The displacement of a body is zero. The distance covered 1


a) zero
b) is not zero
c) may or may not be zero
d) depends upon the acceleration
12 Swimming is possible on account of 1
a) Newton's first law of motion
b) Newton’s second law of motion
c)Newton's third law of motion
d)Newton's law of gravitation
KENDRIYA VIDYLAYA SANGATHAN::: RANCHI REGION

13 Newton's universal law of gravitation applies to 1


a) small bodies only
b) planets only
c)both small and big bodies
d)only valid for solar system
14 According to work energy theorem the work done by the net force on a 1
particle is equal to the change in its
a) kinetic energy
b) potential energy
c) linear momentum
d) angular momentum
15 To find the loss of weight in a liquid one has to immersed the body 1
a) completely
b) partly
c) in salted water only
d) to one fourth of its volume
16 When a body vibrates it compresses the air surrounding and forms a high- 1
density and known as
a) refraction
b) reflection
c) rarefaction
d) Compression
Q. no 17 to 20 are Assertion - Reasoning based questions.
These consist of two statements – Assertion (A) and Reason (R). Answer
these questions selecting the appropriate option given below:
(a) Both A and R are true and R is the correct explanation of A
(b) Both A and R are true and R is not the correct explanation of A
(c) A is true but R is false
(d) A is False but R is true
17 Assertion: atoms can neither be created nor destroyed. 1
Reason: atom is made up of electrons, protons and neutrons.
18 Assertion: specialization of cells in advantages for the organism. 1
Reason: It increases the operational efficiency of an organism.
19 Assertion: The work done in moving a body over a closed loop is zero for 1
every force in nature.
Reason: Work done depends on nature of force.
20 Assertion: Improvement in food production is essential for the survival of 1
living organism on the earth.
Reason: as the food is the major source for surviving of living organism on
the earth.
SECTION-B
Question number 21 to 26 are very short answer questions.
21 Give one example each of homogeneous and heterogeneous mixture. 2
OR
KENDRIYA VIDYLAYA SANGATHAN::: RANCHI REGION

How can we check whether the given solid substances is pure or not.
22 What is meant by the term chemical formula? 2
23 Why is it not possible to see an atom with naked eye? 2
24 What is the energy source for active transport? 2
OR
What do you mean by selectively permeable membrane?
25 Under what circumstances is Newton's first law same as Newton's second 2
law.
26 Define Inertia of rest with example. 2
SECTION-C
Question number 27 to 33 are short answer questions.
27 Explain the diffusion of copper sulphate into water. 3
28 Write the names of the following organelles 3
a) powerhouse of the cell
b) digestive bag of the cell
c) protein factory of the cell
29 Name the tissues which show the following features 3
a) cells are living, show thickening, provide mechanical support to plants.
b) cells are dead, show thickening, provides mechanical support to the plants,
made up of one types of cell.
c) cells are living thing, content green colored chloroplast, poses intercellular
spaces.
OR
What are the functions of areolar tissue?
30 for a particle in one dimensional motion the instantaneous speed is always 3
equal to the magnitude of instantaneous velocity. Why?
31 No work is done when we push immovable object like a huge stone or a wall 3
etc. however we feel tired in doing so. explain why?
OR
Identify energy transformation in the following
a) hydroelectric power
b) battery
c)Stretched bow with arrow
32 Explain the factors which determine whether an object floats or sinks when 3
placed on the surface of water.
33 Define frequency. What is its symbol and its SI unit? 3
SECTION-D
Question number 34 to 36 are long answer questions
34 Classify the following as physical change or chemical change 5
a) burning of magnesium ribbon in air
b) burning of Sulphur in air
c) electrolysis of water
d) purification of copper sulphate from impure copper sulphate by
crystallization
e) preparation of sugar solution.
OR
12 grams of potassium sulphate dissolves in 75 grams of water at 60 degree
Celsius. What is its solubility in water at that temperature?
35 Draw a neat and labelled diagram of plant tissue. 5
KENDRIYA VIDYLAYA SANGATHAN::: RANCHI REGION

OR
Draw a neat and labelled diagram of animal tissue.
36 36) What do you mean by organic farming? 5
OR
What are the types of food requirements of dairy animals? Why do external
and internal parasites live on and in the cattle can be fatal?
SECTION-E
Question number 37 to 39 are case-based/data-based question with two short
sub- parts. Internal choice is provided in one of these sub- parts.
37 Protons are present in the nucleus of an atom. It is the number of protons of 4
an atom which determines its atomic number. It is denoted by 'Z'. All atoms
of an element have the same atomic number 'Z'. In fact elements are defined
by the number of protons possess. For hydrogen Z= 1, because in hydrogen
atom have only one proton is present in the nucleus. Therefore , the atomic
number is defined as the total number of protons present in the nucleus of an
atom.

The mass of an atom is practically due to protons and neutrons alone. These
are present in the nucleus of an atom. And protons and neutrons are also
called nucleons. Therefore, the mass of an atom resides in its nucleus. For
example, mass of carbon is 12 u because it has 6 protons and 6 neutrons ,6 u
+ 6u is equal to 12u. Similarly, the mass of aluminium is 27u(13 protons+14
neutrons). The mass number is defined as the sum of the total number of
protons and neutrons present in the nucleus of an atom. It is denoted by 'A'.

1) atomic number is denoted by


a) X
b) Y
c) Z
d) z

2) the sum of the total number of protons and neutrons present in the nucleus
of an atom is
a) atomic number b) mass number c)atomic weight
d) none of the above

3) mass number is denoted by


a) A
b) a
c) Z
d) z

4) Why is mass of carbon 12? Give the reason?


38 Chromosomes are rod like structures present in the nucleus. They become 4
visible only during cell division. Chromosome is composed of protein and
DNA. DNA contains specific information which is transferred from one
generation to next generation.

1) Define centromere.
KENDRIYA VIDYLAYA SANGATHAN::: RANCHI REGION

2) list function of chromosome.


3) How chromosome and DNA are linked?
4) name two types of chromosome based on the position of centromere.
OR
In humans how many pair of chromosomes are found?
39. The quantity of matter contained in an object is called mass. It remains 4
constant whether the object is on earth, moon or even in the outer space.
Weight on the other hand is the force of attraction of earth with which an
object is attracted towards the earth. Now, suppose a man weighs 600 Newton
on earth his weighs on moon would be 100 Newton.

1) the mass of man on Earth, if g is 10 meters per second square is


a) 60 kg
b) 10kg
c) 6000kg
d) 1000kg

2) the mass of man on moon is


a) 60 kg
b) 10 kg
c) 6000kg
d) 1000kg
OR
A solution due to gravity on moon is
a) 10m/s^2
b) 9.8m/s^2
c) 1.66m/s^2
d) 1m/s^2

****************
KENDRIYA VIDYLAYA SANGATHAN::: RANCHI REGION

SCIENCE (086)
CLASS IX
MARKING SCHEME (2022-23)
Q. Questions Marks
No
Section A
1 a) sodium chloride 1
2 d) a suspension 1
3 d) 8:1 1
4 d) none 1
5 c) protons and neutrons 1
6 c) phosphorus 1
7 a) ATP 1
8 c) secretion 1
9 c) 120 days 1
10 c) cartilage 1
11 c) may or may not be zero 1
12 c) when the swimmer pushes some water in backward direction then he get 1
some momentum in forward direction from water and starts to swim. This
is according to Newton's third law of motion.
13 c) it is applicable to both small and big bodies. 1
14 a) work done by the net force is equal to change in kinetic energy of the 1
particle this is according to work energy theorem.
15 a) completely 1
16 d) compression 1
17 b) atoms can be created or destroyed atom is made up of sub atomic 1
particles.
18 a) both assertion and reasoning are true and reason is correct explanation of 1
the assertion.
19 d) work done in the motion of a body over a closed loop is zero for 1
conservation forces only. Work done depends upon the nature of force.
20 a) 1
Section-B
21 homogeneous mixture- aqueous solution of table salt 2
Heterogeneous mixture-milk
OR
By checking apps melting point whether it's agree with the value given in
troubles or not if it agree then it is pure.
22 the chemical formula of a compound represent the composition of molecule 2
of the compound in terms of symbols of the elements present in it.
23 we cannot see an atom with naked eye because of its very small size. The 2
radius of an atom is in the order of 10 to the power minus 10 metre and such
a small object cannot be seen with a naked eye.
24 ATP is the energy source of active transport. 2
OR
Selectively permeable membrane allows the passage of solvent and summer
selected solids in and out of the cell.
KENDRIYA VIDYLAYA SANGATHAN::: RANCHI REGION

25 When the force applied on a body is zero Newton's first law becomes a 2
special case of Newton's second law.
26 Inertia of rest is the tendency of the body to continue in state of rest even 2
when some external unbalanced force is applied on it inertia of rest. for
example when a carpet is suddenly jerked the dust fly off because due to
the sudden moment the carpet moves but the dust on account of inertia of
rest is left behind.

Section-C
27 copper sulphate crystals are blue in color. When we put some crystals of 3
copper sulphate at the bottom of the beaker containing water, the water
slowly turns blue, this is because particles of copper sulphate mix with the
particles of water. This mixing of particles will continue till the whole
solution turns blue. This phenomenon is called diffusion.
28 a) mitochondria 3
b) lysosome
c) ribosome
29 a) collenchyma 3
b) Sclerenchyma
c) Chlorenchyma
OR
Areola tissue fills the space inside the organs supports internal organs and
helps in repair of tissue it also gives the fat tissue to the organism for the
development of the bodies shape.
30 In an accelerated motion the velocity of an object always keeps on 3
changing. One has to measure the instantaneous velocity. However, when
accelerated motion takes place along a straight line the velocity of the body
changes only due to change in magnitude of velocity. The instantaneous
speed is always equal to the magnitude of instantaneous velocity of the
particle in one dimensional motion.
31 when we push immovable object like a huge stone or a Wall, we feel tired 3
quickly all those the work done appears to be zero as there is no
displacement. There is no doubt that the work done by us on the object is 0.
However, the work done on our own body is not zero, however muscles are
stretched our blood is displaced to the to the straining muscles and it is in
making this displacement that energy is lost. It is because of this loss of
energy that we feel tired.
OR
a) P.E to K.E and electrical energy
b) chemical to electrical energy
c) potential energy to kinetic energy
32 a) volume of the object 3
b) density of the water
33 the number of complete waves or cycles produced per second is called 3
frequency of sound waves.
It is denoted by f. SI is hertz (Hz).

SECTION-D
Question number 34-36 are long answer questions
KENDRIYA VIDYLAYA SANGATHAN::: RANCHI REGION

34 i) chemical change (oxide of magnesium is formed) 5

ii) chemical change (oxide of Sulphur formed)

iii) chemical change (hydrogen and oxygen are formed)

iv) physical change (no new substance formed)

v) physical change

OR

Here we have given that 75 grams of water dissolves want to grams of


potassium sulphate. We have to find how much potassium sulphate will
dissolved in 100 grams of water. Now
75 gram of water will dissolve =12 gram of potassium sulphate.
so,
100 gram of water will dissolve
= 12 / 75×100g of potassium sulphate= 16g of potassium sulphate
Does the solubility of potassium sulphate in water is 16 gram at 60 degree
Celsius.

35 5

OR

36 Organic farming is a farming system in which there is minimal or no use 5


of chemical such as fertilizers, herbicides, pesticides etc. There is maximum
input of organic manure recycled farms waste that is straw and livestock
excreta use of bioad such as culture of blue green algae in preparation of
bio fertilizers. Also Neem leaves or turmeric is used as specifically in grain
storage which act as biopesticides. It employs healthy cropping system
(mixed cropping, intercropping and crop rotation).
OR
There are two types of food requirements of dairy animals.
a) maintenance requirement, which is the food required to support the
animal to live a healthy life.
b) milk producing requirement which is the food required during the
lactation period. The external parasites live on skin and mainly cause skin
disease the internal parasite like warms affect stomach and intestine while
flukes damage liver.
Section-E
KENDRIYA VIDYLAYA SANGATHAN::: RANCHI REGION

Question number 37 to 39 are case-based/ data - based questions with 2 to


3 short sub parts. Internal choice is provided in one of these sub parts.
37 1) c 4
2) b
3)a
4) mass of carbon is 12 u because it has 6 protons and 6 neutrons, 6 u + 6u
= 12u.
38 1) Centromere is region on a chromosome that join sister chromatids are 4
which are formed during cell division. Chromosome contents genes which
are responsible for the particular character in next generation. DNA is a
thread like structure which condensed to form chromosome.

2) chromosome is essential for the process of cell division. Chromosome


are often called the 'Packaging material' because it tightly holds the DMA
and proteins together in the eukaryotic cells.

3) DNA is a thread like material that is present in loose form in the matrix
of the cell. During cell division it becomes condensed and form rod shape
chromosome.

4) i) metacentric-centromere is present in the middle of chromosome.


ii) telocentric- centromere is present at the end of the chromosome.

5) In human beings, 23 pairs of chromosomes that is that is 46 chromosomes


are present.
39 1) a) W= m × g 4
600=m×10
m= 60kg

2) a) mass remain scene everywhere. So, much on moon= 60 kg.

OR

c) W= m × g
100=60×g
g= 100/60=1.66m/s^2
KENDRIYA VIDYLAYA SANGATHAN::: RANCHI REGION
KENDRIYA VIDYLAYA SANGATHAN::: RANCHI REGION
KENDRIYA VIDYLAYA SANGATHAN::: RANCHI REGION
KENDRIYA VIDYLAYA SANGATHAN::: RANCHI REGION
KENDRIYA VIDYLAYA SANGATHAN::: RANCHI REGION
KENDRIYA VIDYLAYA SANGATHAN::: RANCHI REGION
KENDRIYA VIDYLAYA SANGATHAN::: RANCHI REGION
KENDRIYA VIDYLAYA SANGATHAN::: RANCHI REGION
KENDRIYA VIDYLAYA SANGATHAN::: RANCHI REGION
KENDRIYA VIDYLAYA SANGATHAN::: RANCHI REGION
KENDRIYA VIDYLAYA SANGATHAN::: RANCHI REGION
KENDRIYA VIDYLAYA SANGATHAN::: RANCHI REGION
KENDRIYA VIDYLAYA SANGATHAN::: RANCHI REGION
KENDRIYA VIDYLAYA SANGATHAN::: RANCHI REGION

Blueprint of science class 9


Unit/ chapter name 1 mark A-R que 2 marks 3 marks 4 5 Subtot Total
que (Assertion- que que marks marks al unit
(MCQ) Reason) que( questi chapte wise
source on rwise
based/
case
based)
No. of No. of No. of No. of No. of No. of No. of
questi questions( question question questio questi questi
ons(m marks s(marks s(marks ns(mar ons(m ons(m
arks allotted) allotted) allotted) ks arks arks
allotte allotte allotte allotte
d) d) d) d)
Matter: its nature 13(25)
and behaviour
(i) matter in our 1(1) 1(2)(OR) 1(3) 3(6)
surrounding
(ii) Is matter around 1(1) 1(5)(O 2(6)
us pure( excluding R)
the topic of
separation)
(iii) Atoms and 1(1) 1(1) 2(4) 4(6)
molecules
(iv) Structure of 3(3) 1(4)(O 4(7)
atom R)
Organisation in the 10(22)
living world
(i) cell: basic unit of 2(2) 1(2)(OR) 1(3) 1(4)(O 5(11)
life R)
(ii) Tissue 2(2) 1(1) 1(3)(OR) 1(5)(O 5(11)
R)
Motion, Force and 14(27)
work
(i) Motion 1(1) 1(3) 2(4)

(ii) Force and 1(1) 2(4) 3(5)


Newton’s laws
(iii) Gravitation 1(1) 1(4)(O 2(5)
R)
(iv) Floatation 1(1) 1(3) 2(4)

(v) Work, energy 1(1) 1(1) 1(3)(OR) 3(5)


and power
KENDRIYA VIDYLAYA SANGATHAN::: RANCHI REGION

(vi) Sound 1(1) 1(3) 2(4)


Food production 2(6)

Improvement of 1(1) 1(5) 2(6)


food resources
Subtotal 16(1) 4(1) 6(12) 7(21) 3(12) 3(15) 39(80)
KENDRIYA VIDYLAYA SANGATHAN::: RANCHI REGION

KENDRIYA VIDYALAYA SANGATHAN::: RANCHI REGION


CLASS IX
SCIENCE
SAMPLE PAPER
(SESSION: 2022-2023)
MAX MARKS: 80
General Instruction:
• All questions are compulsory
• The question paper has five sections and 39 questions.
• Section - A has 20 questions carrying 1 mark each.
• Section - B has 6 questions carrying 2 marks each.
• Section - C has 7 questions carrying 3 marks each.
• Section - D has 3 questions carrying 4 marks each.
• Section - E has 2 questions carrying 5 marks each.

SECTION A

1. A gas can be liquefied, when the following conditions will be 1


provided:
(a) Low temperature, low pressure
(b) High temperature, low pressure
(c) Low temperature, high pressure
(d) High temperature, high pressure

2. In a water - sugar solution 1


(a) Water is solute and sugar is solvent
(b) Water is solvent and sugar is solute
(c) Water is solvent and water is solute
(d) None of the above

3. What is the chemical formula of sodium carbonate? 1


(a) NaHCO3
(b) NaOH
(c) Na2CO3
(d) Na2HCO3
KENDRIYA VIDYLAYA SANGATHAN::: RANCHI REGION

4. The atomic number of an element is equal to – 1


(a) Number of the protons
(b) Number of neutrons
(c) Number of electrons
(d) None of the above

5. An alpha Particle is – 1
(a) Sub atomic particle
(b) An unionized particle
(c) A neutral particle
(d) A doubly charged helium atom

6. The element which has only one proton and neutron is 1


(a) Helium
(b) Hydrogen
(c) Potassium
(d) Magnesium

7. A cell will swell up if 1


(a) The concentration of water molecules in the cell is higher than
the concentration of water molecules in the surrounding medium.
(b) The concentration of water molecules in the surrounding medium
is higher than water molecules concentration in the cell.
(c) The concentration of water molecules is same in the cell and in the
surrounding medium.
(d) Concentration of water molecules does not matter.

8. Chromosomes are made up of 1


(a) DNA
(b) Protein
(c) DNA and protein
(d) RNA

One of the following is not true about Cardiac muscles


9. 1
(a) They can be controlled
(b) They lack mitochondria
(c) Cells are bi-nucleate
(d) Found outside the heart such as the digestive tract
KENDRIYA VIDYLAYA SANGATHAN::: RANCHI REGION

(e) All of the above

10. Which of the following tissues has dead cells?


(a) Parenchyma
(b) Sclerenchyma
(c) Collenchyma
(d) Epithelial tissue

11. From the given v – t graph, it can be inferred that the object is in 1

(a) in uniform motion


(b) at rest
(c) in non-uniform motion
(d) moving with uniform acceleration

12. A goalkeeper in a game of football pulls his hands backwards after 1


holding the ball shot at the goal. This enables the goalkeeper to
(a) exert larger force on the ball
(b) reduce the force exerted by the balls on the hands
(c) increase the rate of change of momentum
(d) decrease the rate of change of momentum

13. The value of acceleration due to gravity 1


(a) is same on equator and poles
(b) is least on poles
(c) is least on equator
(d) increases from pole to equator

14. Archimedes’s principle holds good for 1


(a) liquids only
(b) gases only
(c) both liquids and gases
(d) neither liquid nor gases
KENDRIYA VIDYLAYA SANGATHAN::: RANCHI REGION

15. Assertion: Atomicity of ozone is three while that of oxygen is two. 1


Reason: Atomicity is the number of atoms constituting a molecule.
a) Both assertion and reason are true and reason is the correct
explanation of assertion.
(b) Both assertion and reason are true but reason is not the correct
explanation of assertion.
(c) Assertion is true but reason is false.
(d) Assertion is false but reason is true

16. Assertion: A cell of meristem tissue is very active. 1


Reason: Meristem tissue is helps to reproduction of plants
a) Both assertion and reason are true and reason is the correct
explanation of assertion.
(b) Both assertion and reason are true but reason is not the correct
explanation of assertion.
(c) Assertion is true but reason is false.
(d) Assertion is false but reason is true

17. Assertion: No work is done when a woman carrying a load on her 1


head, walks on a level road with a uniform velocity.
Reason: No work is done if force is perpendicular to the direction of
displacement.
a) Both assertion and reason are true and reason is the correct
explanation of assertion.
(b) Both assertion and reason are true but reason is not the correct
explanation of assertion.
(c) Assertion is true but reason is false.
(d) Assertion is false but reason is true

18. Assertion: Fungicides act against fungal pathogen. 1


Reason: Fungicides are not harmful to human beings.
a) Both assertion and reason are true and reason is the correct
explanation of assertion.
(b) Both assertion and reason are true but reason is not the correct
explanation of assertion.
(c) Assertion is true but reason is false.
(d) Assertion is false but reason is true.

19. If speed of a car becomes 2 times, its kinetic energy becomes 1


(a) 4 times
(b) 8 times
(c) 16 times
(d) 12 times
KENDRIYA VIDYLAYA SANGATHAN::: RANCHI REGION

20. In SONAR, we use 1


(a) ultrasonic waves
(b) infrasonic waves
(c) radio waves
(d) audible sound waves

SECTION - B

21. Define: (a) Latent heat of fusion (b) Latent heat of 2


vaporization
(OR)

What is the Tyndall effect? Which kinds of solutions show it?

22. Define Atomicity. Categorize the following species into monatomic, 2


diatomic, triatomic and polyatomic molecules:
HCl, H2, O3, PCl5.
An element has 17 electrons and 17 protons.
23. (a) Name the element and write the symbol of it. 2
(b) What is the atomic number of the element?

24. A force of 5 N gives a mass m1, an acceleration of 10 ms-2 and a mass 2


m2, an acceleration of 20 ms-2. What is acceleration would it give if
both the masses were tied together?

25. State Newton’s second law of motion. What is the S.I unit of Force? 2

26. What do you mean by Inertia? Give any two examples. 2

SECTION - C
27. Differentiate between Plant cell and Animal cell. 3
KENDRIYA VIDYLAYA SANGATHAN::: RANCHI REGION

28.

In the above figure two onion bulbs are placed in jar1 and jar 2. The
3
roots have been grown for few days and then root tip of the onion of
jar 2 has been cut. It has been observed after 10 days that jar 1 has the
long roots while the growth of roots of onion in jar 2 has been
stopped. Explain.

(OR)
Draw a neat and clean diagram of nervous tissue. Which part of the
nervous tissue receives signal/stimuli from the environment?

A bus starting from rest moves with a uniform acceleration of 5 ms-2


29. for 10 minutes. Find (a) the speed acquired, (b) the distance travelled. 3

(a) Why does a block of wood held under water rise to the surface
30. when released? 3
(a) State the factors on which upthrust or buoyant force depend.

(a) Differentiate between physical and chemical change. 2


31. (b) State any two characteristics of colloid. 1

A. Give one example each of potential energy


32. (i) due to position
(ii) due to shape. 3
B. What is the commercial unit of energy?
(OR)
Calculate the work done against the gravity. Give any two uses of
kinetic energy.
(a) What is intensity of sound?
33. (b) On what factor does the pitch of a sound depend? 3

(c) What is the wavelength of the wave in the above given picture?

SECTION - D
KENDRIYA VIDYLAYA SANGATHAN::: RANCHI REGION

(a) Will 35Cl and 37Cl have different valencies? Justify your
34. answer. 4
(b) Why did Rutherford select a gold foil in his α–ray scattering
experiment?
(c) Draw the electron dot structure of element having 11 and 18
number of electrons.
(d) Write down the electron distribution of chlorine atom. How
many electrons are there in the L shell? (Atomic number of
chlorine is 17).
What happens when-
35. (a) Lysosome will burst 1
(b) There will be no plasma membrane 1
(c) There is no mitochondria 1
(d) There is no nucleus 1

Reeta was playing with a tennis ball. She threw the ball with certain
36. velocity and noticed that after reaching a certain height the ball was
coming downwards. She wondered why the ball did not go upward.
A. Doubt of Reeta will be cleared by giving her the concept of
(i) Gravitation 1
(ii) Attraction
(iii) Buoyancy
(iv) Thurst
B. When the ball reached a certain height, it is coming
downward. The ball is said to be at
(i) Motion 1
(ii) Free fall
(iii) Rest
(iv) None of the above
C. State universal law of Gravitation. Why is it known as
universal law? Derive an expression for it.
2

SECTION - E

Give reason:
37. (a) Naphthalene balls disappear with time without leaving any 3
solid.
(b) What produces more severe burns, boiling water or
steam? Why?
(c) A diver is able to cut through water in a swimming pool.
Which property of matter does this observation show?

(d) To make a saturated solution, 36 g of sodium chloride is 2


dissolved in 100 g of water at 29K. Find its concentration at
this temperature.
KENDRIYA VIDYLAYA SANGATHAN::: RANCHI REGION

(a) What minerals is the bone matrix rich in?


38. (b) Which tissue in plants provides them flexibility? 5
(c) Which blood cells deal with immune reaction?
(d) What stimulates the movement of muscles?
(e) Water hyacinth floats on water surface. How?
(a) What do we get from cereals, pulses, fruits and vegetables? 1
39. (b) Compare the use of manure and fertilizers in maintaining the soil 1
fertility.
(c) Why is animal husbandry essential? Which method is commonly 3
used for improving cattle breeds and why?
KENDRIYA VIDYLAYA SANGATHAN::: RANCHI REGION

Marking Scheme of Science of class 9


Section-A
1. (C)
2. (b)
3. (c)
4. (a)
5. (d)
6. (b)
7. (b)
8. (C)
9. (c)
10. (b)
11. (a)
12. (d)
13. (b)
14. (c)
15. (a)
16. (c)
17. (a)
18. (c)
19. (a)
20. (c)

Section – B
21. (a) The amount of heat energy that is required to change 1 kg of solid into liquid at
atmospheric pressure at its melting point is known as latent heat of fusion.
(b) The amount of heat energy that is required to change 1 l of liquid into vapour at
atmospheric pressure at its boiling point is known as latent heat of vaporisation.
(OR)

The phenomenon by which the colloidal particles scatter light is called Tyndall effect. If light is
passed through a colloid the light is scattered by the larger colloidal particles and the, beam
becomes visible. This effect is called Tyndall effect. Colloid and suspension solution show tyndall
effect.

22. Atomicity refers to the number of atoms present in a single molecule of an element,
substance or compound.
HCl- diatomic, H2 – diatomic, O3 – triatomic, PCl5 – polyatomic

23. (a) Chlorine, Cl


(b) Z = 17
24. 6m/s2
KENDRIYA VIDYLAYA SANGATHAN::: RANCHI REGION

25. The rate of change of momentum of an object is proportional to the applied


unbalanced force in the direction of force. S.I. unit is Newton (N).
26. Inertia is the tendency of an object to resist change in its state of motion or rest.
Examples: Dust coming out of mat when beaten.Falling forward in transport when
sudden breaks are applied.

Section – C

27.
Plant Cell Animal Cell
• Plant cells have cell wall • Animal cells do not have cell
• Plant cells have plastids wall
• Plant cells lack centriole • Animal cells do not have plastids
• Plant cells have large vacuole • Animal cells have centriole
• Animal cells have small or no
vacuole

28. Because meristematic tissue of the root tip has been removed.
(OR)

Dendrites of the neuron receives the


stimuli from the environment.
29. (a) 12m/s (b) 720 m
30. (a) Because the weight of the wood is less than the weight of an equal volume of
water. So when it's completely submerged, the upward buoyant force on it is greater
than the downward gravitational force on it.
(b) The buoyant force depends on the mass of the object, the weight of the object
and its density. Also, the buoyant force depends on the volume of the liquid displaced.
31. (a)
Physical Change Chemical Change
• Change take place in physical • Change take place in chemical
properties properties
• No new substance is formed • New substance is formed
• Reversible in nature • Irreversible in nature
• Ex – change in state of water • Rusting of iron

(b) Two properties of colloid are:


KENDRIYA VIDYLAYA SANGATHAN::: RANCHI REGION

(i) A colloid is a heterogenous mixture.


(ii) Colloid is stable in nature.
32. A. (i) Water stored in dam
(ii) A stretched rubber band
B. kilowatt hour(kWh)
(OR)
The work done on the mass is then W = Fd = mgh.
Two uses of kinetic energy are:
(i) To run windmills.
(ii) To generate electricity

33.(a) is defined as the power carried by sound waves per unit area in a direction
perpendicular to that area.
(b) its frequency
(c) AE.
Section - D
34. (a) no, as they isotopes and isotopes have same atomic number
(b) Gold has highest malleability and hence can form the thinnest sheet(1000
atoms) and hence alpha scattering can be studied very efficiently.

(c)
(d) K= 2 electrons, L= 8 electrons, M= 7 electrons.
35. (a) cell will be digested
(b) There will be no selective transport of of materials across the PM
(c) The will be no production of energy currency which will affect the metabolic
activities of cell.
(d) There will be no control centre of the cell then the activities of the cell will not be
managed.
36. A. (i) B.(ii) C. Every object in the universe attracts every other object with the force
which is directly proportional to the product of the mass and inversely proportional to the
square of the distance between the object. F= G Mm/d2.
Section – E
37. (a) Sublimation (b) Steam as it has more energy in the form of latent heat of
vaporization
(c) liquid, as the particles of liquid have intermolecular space between them
(d) 26.47%
38. (a) Calcium and phosphorous (b) Chollenchyma tissue (c) WBC (d) An impulse (e)
Due to the presence of Aerenchyma tissue
39. (a) Carbohydrate, proteins vitamin and minerals
KENDRIYA VIDYLAYA SANGATHAN::: RANCHI REGION

(b)
Fertilizer: Manure:

They provide humus to


They do not provide humus to the soil.
the soil.
It is relatively less in
They are rich in nutrients.
nutrient content.
They can be made in
They are synthesized in factories.
fields.

(c) Animal husbandry is the branch of agriculture concerned with animals that are raised
for meat, fibre, milk, or other products. It includes day-to-day care, selective breeding, and
the raising of livestock.
The method of cross-breeding is widely used to improve cattle breeds. Cross-breeding two
good cattle varieties will lead to a new, improved variety.
KENDRIYA VIDYLAYA SANGATHAN::: RANCHI REGION

Blue Print as per CBSE Sample Paper

Unit / Chapter Name 01 Mark A-R 02 Marks 03 Marks 05 Marks 04 Marks Sub-Total Total Unit
Questions Questions Questions Questions Questions Questions Chapter wise
MCQ [Assertion- Source wise
Reasoning] based /
Case
based

No. of No. of No. of No. of No. of No. of No. of


questions questions questions questions questions questions questions
[Marks [Marks [Marks [Marks [Marks [Marks [Marks
allotted] allotted] allotted] allotted] allotted] allotted] allotted]

Unit-I Matter - Its Nature and 13(25)


Behaviour

Matter in Our Surroundings 1(1) 1(2)OR 1(3) 3(6)

Is Matter Around Us Pure 1(1) 1(5)OR 2(6)


(Excluding Separating the
components of a mixture)

Atoms and Molecules 1(1) 1(1) 2(4) 4(6)

Structure of Atom 3(3) 1(4)OR 4(7)


KENDRIYA VIDYLAYA SANGATHAN::: RANCHI REGION

Unit-II Organisation in the 10(22)


Living World

Cell: Basic unit of Life 2(2) 1(2)OR 1(3) 1(4)OR 5(11)

Tissues, Organs, Organ 2(2) 1(1) 1(3)OR 1(5)OR 5(11)


system, Organism

Unit-III Motion, Force and 14(27)


Work

Motion 1(1) 1(3) 2(4)

Force and Newton’s laws 1(1) 2(4) 3(5)

Gravitation 1(1) 1(4)OR 2(5)

Floatation 1(1) 1(3) 2(4)

Work, Energy & Power 1(1) 1(1) 1(3)OR 3(5)

Sound 1(1) 1(3) 2(4)

Unit IV Food Production 2(6)

(Improvement in Food 1(1) 1(5) 2(6)


Resources)

Sub-Total 16(16) 4(4) 6 (12) 7(21) 3(15) 3(12) 39 (80)

“OR” means that there is an alternate question in that particular question.


# Please refer to Sample paper of Class X while preparing the question paper of class IX.
KENDRIYA VIDYLAYA SANGATHAN::: RANCHI REGION

Class IX
Sample Question Paper (2022-23)
SUBJECT- SCIENCE
Max. Marks: 80
Time Allowed: 3 hours
General Instructions:
I This question paper consists of 39 questions in 5 sections.
ii. All questions are compulsory. However, an internal choice is provided in some questions.
A student is expected to attempt only one of these questions.
iii. Section A consists of 20 objective type questions carrying 1 mark each.
iv. Section B consists of 6 Very Short questions carrying 02 marks each. Answers to these
questions should in the range of 30 to 50 words.
v. Section C consists of 7 Short Answer type questions carrying 03 marks each. Answers to
these questions should in the range of 50 to 80 words
vi. Section D consists of 3 Long Answer type questions carrying 05 marks each. Answer to
these questions should be in the range of 80 to 120 words.
Vii . Section E consists of 3 source-based/case-based units of assessment of 04

SECTION-A
1) When heat is constantly supplied by a burner to boiling water, then the temperature of
water during vaporization:
[A]. Rises very slowly
[B]. Rises rapidly until steam is produced
[C]. First rises and then becomes constant
[D]. Does not rise at all.

2) Which of the following are physical changes?


[A]. Melting of iron metal, rusting of iron metal, bending of an iron rod.
B]. Melting of iron metal, rusting of iron metal, drawing a wire of iron metal.
[C]. Melting of iron metal, bending of an iron rod, drawing a wire of iron metal.
[D]. Rusting of iron metal, bending of an iron rod, drawing a wire of iron metal.

3) All samples of carbon dioxide contain carbon and oxygen in the mass ratio 3 : 8. This is in
agreement with the law of
(a) conservation of mass
(b) constant proportions
(c) multiple proportions
(d) gaseous volumes

4) An atom with 3 protons and 4 neutrons will have a valency of


(a) 3
KENDRIYA VIDYLAYA SANGATHAN::: RANCHI REGION

(b) 7
(c) 1
(d) 4

5) Observe the given figure and answer the question that follows

(I) Most of the fast moving a-particles passed straight through the gold foil.
(II) Some of the a- particles were deflected by the foil by small angles.
(III) Surprisingly one out of every 12000 particles appeared to rebound.
Identify the correct observations.
(a) (I) and (II)
(b) (II) and (III)
(c) (I) and (III)
(d) All are correct.

6) Information about two atoms, X and Y are shown below :

Which of the following is correct about these two atoms?


KENDRIYA VIDYLAYA SANGATHAN::: RANCHI REGION

7) The undefined nuclear region of prokaryotes is also known as


[A]. Nucleus
[B]. Nucleolus
[C]. Nucleic acid
[D]. Nucleoid

8) The dead element present in the phloem is


[A] Companion cells
[B]. Phloem fibers
[C]. Phloem parenchyma
[D]. Sieve tubes

9) Given below are four operations for preparing a temporary mount of human cheek cells:
(i) Taking a scraping from the inner side of the cheek and spreading it on a clean slide
(ii) Putting a drop of glycerine on the material
(iii) Adding two or three drops of methylene blue
(iv) Rinsing the mouth with fresh water and disinfectant solution
Identify the correct sequence of these operations.
(a) (i)—(ii)—(iii)—(iv)
(b) (iv)—(i)—(iii)—(ii)
(c) (iv)—(i)—(iii)—(ii)
(d) (i)—(iii)—(ii)—(iv)

10) Cells with evenly thickened, hard, lignified wall are

a)striated muscle cells


b)parenchyma
c)sclerenchyma
d)collenchyma
KENDRIYA VIDYLAYA SANGATHAN::: RANCHI REGION

11.) Four cars A, B, C and D are moving on a levelled road. Their distance versus time graphs
are shown in the adjacent figure. Choose the correct statement.

(a) Car A is faster than car D.


(b) Car B is the slowest.
(c) Car D is faster than car C.
(d) Car C is the slowest.

12) An athlete does not come to rest immediately after crossing the winning line due to the
(a) inertia of motion
(b) inertia of rest
(c) inertia of direction
(d) none of these

13) The value of the universal gravitational constant is _______


(a) 6.67 x 10-11 Nm2/kg2
(b) 66.7 x 10-11 Nm2/kg2
c) .667x10-11Nm2/kg2
(d) None of these

14) What is the upward force experienced by a body, when it is immersed in a fluid, known
as?
A) Tangential force
b) Buoyant force
c)Centripetal force
d)Centrifugal force

15) Water stored in a dam possesses


(a) no energy
(b) electrical energy
(c) kinetic energy
(d) potential energy

16) If a wave complete 20 vibration in 2.5s, then its frequency is


(a) 200 Hz
(b) 50 Hz
(c) 20 Hz
(d) 8 Hz
KENDRIYA VIDYLAYA SANGATHAN::: RANCHI REGION

Q. no 17 to 20 are Assertion - Reasoning based questions.


These consist of two statements – Assertion (A) and Reason (R). Answer these questions
selecting the appropriate option given below:
(a) Both A and R are true and R is the correct explanation of A
(b) Both A and R are true and R is not the correct explanation of A
(c) A is true but R is false
(d) A is False but R is true.

17) Assertion :Pure water obtained from different sources such as river, well, spring,sea
etc. always contains hydrogen and oxygen combined in the ratio of 1 : 8 by mass.
Reason :A chemical compound always contains same elements combined in same fixed
proportion by mass.

18) Assertion : The inner lining of intestine has tall epithelial cells.
Reason : Columnar epithelium facilitates absorption and secretion.

19) Assertion :The change in kinetic energy of a particle is equal to the work done on it by
the net force.
Reason :Change in kinetic energy of particle is equal to the work done only in case of a
system of one particle.

20) Assertion : Usage of manure is advantageous for our environment.


Reason : Manure contains chemical substances like nitrogen, phosphorus and potassium.

. SECTION – B
Q. no. 21 to 26 are very short answer questions.

21) Name A, B, C, D, E and F in the following diagram showing change in its state-- 2
Name A, B, C, D in the following diagram showing change in its state.
KENDRIYA VIDYLAYA SANGATHAN::: RANCHI REGION

22) Write down the formulae of


(i) Sodium oxide
(ii) Aluminium chloride

23) Give the names of the elements present in the following compounds:
(a) Quick lime
(b) Potassium sulphate.

24) How is prokaryotic cell different from a eukaryotic cell?


OR
Make a comparison and write down ways in which plant cells are also different from
animal cell.

25) What is momentum? Write its SI unit.

26) Explain Newton’s second law of motion and with the-help of an example show how it
is used in sports.
.

SECTION - C
Q.no. 27 to 33 are short answer questions

27) State the differences between solid, liquid and gas.

28) Name the cell organelle for the following:


(a) Present only in plant cell, provides strength and rigidity to the cell.
(b) It is the site for lipids synthesis and helps in detoxification of drugs.
(c) The inner membrane is folded to form cristae, it has its own DNA and proteins.

29) Differentiate between striated, unstriated and cardiac muscles on the basis of their
structure and location in the body.
KENDRIYA VIDYLAYA SANGATHAN::: RANCHI REGION

30. A motorcyclist drives from place A to B with a uniform speed of 30 km h−1 and returns
from place B to A with a uniform speed of 20 km h−1. Find his average speed.

31. A body of mass 100 kg is lifted up by 10 m. Find :


(i) the amount of work done
(ii) potential energy of the body at that height (value of g = 10 m/s2)
OR
. (a) Explain by an example what is meant by potential energy. Write down the expression
for gravitational potential energy of a body of mass m placed at a height h above the surface
of the earth.
(b) What is the difference between potential energy and kinetic energy?
(c) A ball of mass 0.5 kg slows down from a speed of 5 m/s so that of 3 m/s. Calculate the
change in kinetic energy of the ball. State your answer giving proper units.

32. (a) Define density. What is the SI unit of density?


(b) Define relative density. What is the SI unit of relative density?
c) The density of turpentine is 840 kg/m3. What will be its relative density? (Density of
water = 1000 kg/m.

33) (a) Define the terms 'frequency', 'wavelength' and 'velocity' of a sound wave. What is
the relation between them?
(b) A body vibrating with a time-period of 1/256s produces a sound wave which travels in air
with a velocity of 350 m/s. Calculate the wavelength.

SECTION-D
Q.no. 34 to 36 are Long answer questions.
34) a) What are the difference between true solution , colloids and suspensions .
b) Classify the following into true solutions and colloidal solutions and suspension--
-
Ink, Salt solution, Starch solution, muddy water.
OR-
i) List the points of differences between homogeneous and heterogeneous
mixtures.Give suitable examples.

ii)During an experiment the students were asked to prepare a 10% (Mass/Mass)


solution of sugar in water. Ramesh dissolved 10g of sugar in 100g of water while Sarika
prepared it by dissolving 10g of sugar in water to make 100g of the solution.
(a) Are the two solutions of the same concentration
(b) Compare the mass % of the two solutions.
--
KENDRIYA VIDYLAYA SANGATHAN::: RANCHI REGION

35) Distinguish between the following pairs in one or two sentences for each-

(a) RBCs and WBCs;


(b) bone and cartilage;
(c) blood and lymph;
(d) striated and unstriated muscles;
(e) tendon and ligament
OR
Differentiate between parenchyma ,Collenchyma and sclerenchyma tissues. Draw well
labelled diagram of them.

36) Differentiate between fertilisers and manures. How is the use of manure beneficial for
our environment?

SECTION - E
Q.no. 37 to 39 are case - based/data -based questions with 2 to 3 short sub - parts. Internal
choice is provided in one of these sub-parts.

37) Read the passage given below and answer the following questions.The table shows the
number of sub-atomic particles in arbitrary elements, A to H.
KENDRIYA VIDYLAYA SANGATHAN::: RANCHI REGION

1)The pair of isotopes from the table is/are


(i) C and D (ii) E and F (iii) B and C (iv) G and H
OPTIONS--(a) (ii) only
(b) (iv) only
(c) (ii) and (iv) only
(d) (i), (ii), (iii) and (iv)

2) . Which of the given elements attains noble gas configuration by gaining an electron?
(i) A (ii) E (iii) C (iv) H
(a) (iii) only
(b) (iv) only
(c) (i) and (iv) only
(d) (i) only

3) 43. Identify pair of isobars from the table.


(a) C and D
(b) B and E
(c) G and H
(d) E and F
OR
. The atom _____ has nucleon number 13 and atom _____ has valency 3.
(a) G and F
(b) F and D
(c) C and E
(d) F and

38) Plasma membrane or Cell membrane is the outermost covering of the cell that
separates the contents of the cell from its external environment. The plasma membrane is
KENDRIYA VIDYLAYA SANGATHAN::: RANCHI REGION

flexible and is made up of organic molecules called lipids and proteins. The flexibility of the
cell membrane also enables the cell to engulf in food and other material from its external
environment. Such processes are known as endocytosis.The plasma membrane allows or
permits the entry and exit of some materials in and out of the cell. It also prevents
movement of some other materials. The cell membrane, therefore, is called a selectively
permeable membrane.
Some substances like carbon dioxide or oxygen can move across the cell membrane by a
process called diffusion. There is spontaneous movement of a substance from a region of
high concentration to a region where its concentration is low. Similar thing happens in cells
– some substance like CO2 (carbon dioxide is cellular waste and requires to be excreted out
by the cell) accumulates in high concentrations inside the cell. In the cell’s external
environment, the concentration of CO2 is low as compared to that inside the cell. As soon as
there is a difference of concentration of CO2 inside and outside a cell, CO2 moves out of the
cell, from a region of high concentration, to a region of low concentration outside the cell by
the process of diffusion.
Water obeys the law of diffusion. The movement of water molecules through such a
selectively permeable membrane is called osmosis. The movement of water across the
plasma membrane is also affected by the amount of substance dissolved in water. Thus,
osmosis is the net diffusion of water across a selectively permeable membrane toward a
higher solute concentration.

(1) The plasma membrane is made up of ___________


(a) Proteins
(b) Lipids
(c) Proteins and Lipids (Lipoproteins)
(d) none of above

(2) Why cell membrane is known as selectively permeable membrane?


(3) What is mean by diffusion.
OR
Define Osmosis

39. Every object in the universe attracts every other object with a force which is
proportional to the product of their masses (m1 x m2) and inversely proportional to the
square of the distance (d2) between them. The force is along the line joining the centers of
two objects.
KENDRIYA VIDYLAYA SANGATHAN::: RANCHI REGION

1)What is universal gravitational constant? What is its SI unit?


(2) Two objects of masses 10kg and 20kg separated by distance 10m. What is
gravitational force between them?
3) When mass of one body is doubled then what will be force of gravitation ?
OR
State the universal law of gravitation.
KENDRIYA VIDYLAYA SANGATHAN::: RANCHI REGION

KENDRIYA VIDYALYA SANGATHAN ,RANCHI REGION


CLASS-IX
SCIENCE
MARING SCHEME
SAMPLE PAPER

1) D ------- ------ 1
2) C ----------------1
3) B --------------- 1
4) C------------------ 1
5) D ------------------1
6) C------------------ 1
7) D----------------- 1
8) B---------------- 1
9) C--------------------1
10) C-------------------1
11) B--------------------1
12) A--------------------1
13) A--------------------1
14) B--------------------1
15) D--------------------1
16) D------------------- 1
17) A--------------------1
18) A------------ ---1
19) C--------------------1
20) C--------------------1

21) ANS- A —> Liquefication/melting/fusion B —> Vapourisation/evaporation—1/2x4=2


C-->Solidification D--->Condensaion

22) ANS-i)Na2O--------1
ii)Al2O3--------1

23) Ans—a)CaO------1
b)K2SO4-----1

24) ANS-Prokaryotic cell is generally smaller in size (1-10 pm), nuclear region is poorly
defined, the cell organelles are not membrane-bound and has a single chromosome.
1
Eukaryotic cell is generally larger in size (5-100 pm), nuclear region is well defined with
nuclear membrane. Membrane-bound cell organelles are present and has more than one
chromosome-----1
KENDRIYA VIDYLAYA SANGATHAN::: RANCHI REGION

OR--

1/2X4=2

25) Ans-The momentum (P) of an object is defined as the product of its mass (m) and
velocity (υ).
P = mυ-------------------------------------------------------------------------------------------------------------1
SI unit of momentum is kg ms-1-----------------------------------------------------------------------------1

26) Newton’s second law of motion: The rate of change of momentum of an object is
proportional to the applied unbalanced force in the direction of the force. -----------------------
-------------------1
Use of second law of motion in sports: -----------
In cricket field, the fielder gradually pulls his hands backward while catching a ball. The
fielder catches the ball and gives swing to his hand to increase the time during which the
high velocity of the moving ball decreases to zero. The acceleration of the ball is decreased
and therefore the impact of catching the fast moving balls reduced. If not done so, then the
fast moving ball will exert large force and may hurt the fielder.- --------------------------------------
---------------------------------------1

27)

28) - (a) Cell wall


(b) Smooth endoplasmic reticulum (c) Mitochondria-----------------------------------------------------
-1x3=3
KENDRIYA VIDYLAYA SANGATHAN::: RANCHI REGION

29)

30) We have to find the average velocity of the entire journey. For this, we have the
following information:
Speed from A to B = (v1) = 30 m/s
Let the distance from A to B be (d).
Also, let the time taken to travel from A to B be (t1).
Time =Distance travelled/Speed-----------------------------------------------------------1
We have: t1=d/30
Speed from B to A (v2) = 20 m/s
Let the time taken to travel from B to A be(t2).
Thus, we have: t2=d/20 ,
Total time of journey:=t1+t2=d/30+d/20=d/12------------------------------------------1
Total distance travelled is 2d.
Therefore,
Average speed=Total distance travelled/Time
On putting the values to obtain the average speed of the motorcyclist, we get:
=(2d)/d/12=24 km/hr --------------------------------------------------------------------------1

31)
(i) Mass of the body, (m) = 100 kg
Height above the ground, (h) = 10 m
Acceleration due to gravity, (g) = 10 m/s2--------------------------------1/2
We can calculate the work done against gravity as,
Work done in lifting a body = (Weight of body) × (Vertical distance)
So, W = (m) × (g) × (h)---------------------------------------------------------------1/2
So work done,
W = (100) × (10) × (10) J-----------------------------------------------------------1
= 10 KJ
(ii) Potential energy of a body kept at the same height is equal to the amount of work done
against gravity in moving the body to that position. Therefore, the potential energy of the
body at that height is -10 KJ----------------------------------------------------------------------1
OR----
(a) An example to explain the fundamental meaning of potential energy is as follows:
KENDRIYA VIDYLAYA SANGATHAN::: RANCHI REGION

A brick lying on the ground has no energy, so it cannot do any work. If we lift the brick to the
roof of a house, the work done in lifting the brick against gravity gets stored in it in the form
of potential energy. Thus, the energy in the brick is due to its higher position with respect to
the ground, which is known as potential energy. ----------------------------------------------------------
---------- 1
We can calculate the potential energy of a body of mass (m) lifted by a height (h) as,
Potential energy = (Weight of body) × (Vertical distance)
So,P.E = m × g × h
Where, (P.E) - Potential energy
(m) - Mass of body
(g) - Acceleration due to gravity
(h) - Height by which the body is lifted
(b) The difference between potential and kinetic energy are:
Potential Energy Kinetic Energy
It is the energy of a body by virtue of its It is the energy of a body by virtue of
change in shape, size or configuration.
its motion .

It is independent on the speed of the body. It is directly proportional to the square of the
speed . of the moving body
1

(c) Mass of the ball, (m) = 0.5 kg


Initial velocity, (v1) = 5 m/s
Final velocity, (v2) = 3 m/s
So, initial kinetic energy can be calculated as,
K.E=12mv2
Therefore, initial kinetic energy,
(K.E)1=12×0.5×52 J =6.25 J
Similarly, final kinetic energy,
(K.E)2=12×0.5×32 J =2.25 J
So, Work done = Change in kinetic energy
Therefore work done,
Work done = (KE)2 – (KE)1
= (2.25 – 6.25) J
=–4J
A negative sign shows that the force applied is opposite to the direction of motion of the
body.-----1

32)a) Density is defined as the mass per unit volume of a substance. The symbol for density
is ρ. Mathematically, density is defined as -- ρ=mV
The SI unit of density is kg/m3.-----------------------------------------------1
(b) Relative density of a substance is the ratio of density of the substance to the density of
water. Relative density has no unit.-----------------------------------------------------1
(c)Relative density=Density of turpentine/Density of water =840 kg/m31,000 kg/m3
=0.84 Ans: Relative density of turpentine = 0.84.------------------------------1
KENDRIYA VIDYLAYA SANGATHAN::: RANCHI REGION

33) (a) Frequency: The number of complete waves or cycles produced in one second.
Wavelength: The minimum distance in which a sound wave repeats itself.
Velocity of a wave: The distance travelled by a wave in one second.
We know the relation between velocity, frequency, and wavelength
ν = f × λ,------------------------------------1
where ν is the velocity of sound, F the frequency, and λ the wavelength.
(b) We have to find the wavelength of the sound wave.
Given: Velocity of sound wave ν = 350 m/s
We have to first find the frequency of oscillation.
Time period (T)=1/256s
We know the relation between frequency and time period f=1/T,
where f is the frequency and T the time period.
So, f=256/1Hz =256 Hz ------------------------------------1
Now we can calculate wavelength as λ=v/f,
where ν is the velocity, f the frequency, and λ the wavelength.
So, λ=350/256m =1.367 m
Therefore, velocity of sound is 1.37 m. -----------------------------1

34)

3
KENDRIYA VIDYLAYA SANGATHAN::: RANCHI REGION

true solutions –Salt solution


colloidal solutions -Ink,Starch solution
suspension—muddy water
OR –ANSWER:

i)

ii)(a) No, Sarika has a higher mass by mass percentage.


(b) Solution made by Ramesh:
Mass by mass % = 10/(10+100)×100=10/110×100=9.09%

Solution made by Sarika:


Mass by mass %=10/100×100=10%

35)
1x5=5
a) RBCs WBCs

They are red in colour. They are colourless.


Their number is 4–6 million/mm3. Their number is 6,000–9,000 million/mm3.
The cells contain haemoglobin. The cells lack haemoglobin.

(b)
Bone Cartilage
They are hard and flexible. They are flexible.
They are porous. They are non-porous.
They contain blood vessels. They lack blood vessels.

(c)
Blood Lymph
It is red in colour. It is colourless.
RBCs are present. RBCs are absent.
It transports nutrients, hormones, and vitamins. It transports nutrients (oxygen and
to the tissues and glucose) that may have been
filtered.
out of blood capillaries back into the
KENDRIYA VIDYLAYA SANGATHAN::: RANCHI REGION

bloodstream
(d)
Striated Muscles Unstriated Muscles
They are cylindrical. They are spindle-shaped.
They help in locomotion. They help in rhythmic contraction of the internal
organs.

(e)
Tendon Ligament
It is inelastic. It is elastic.
It connects muscles to bones. It connects two bones.
It is made up of white fibres. It is made up of white fibres.

OR—
KENDRIYA VIDYLAYA SANGATHAN::: RANCHI REGION

36)

Manure is beneficial because they help in:


2
a) protecting the damage of environment from chemicals such as pesticides and fertilisers.
B )recycling the biological wastes, i.e., animal excreta and plant wastes, thus preventing the
accumulation of these things.

37)1-c 2-c 3-a or 3-b-----------------------------------------------------------------1x3=3

38) 1-c--------------------------------------------------------------------1x3=3
- 2)Cell membrane allows or permits the entry and exit of some materials in and out of
the cell. It also prevents movement of some other materials. Hence it is called as selectively
permeable membrane.

3) It is spontaneous movement of a substance from a region of high concentration to a


region where its concentration is low. For example, some substances like carbon dioxide or
oxygen can move across the cell membrane by a process called diffusion.
Or
Osmosis is the passage of water molecules across a semi-permeable membrane from a
solution with a high concentration to a solution with a lower concentration.

39) 1) The force of attraction between any two unit masses separated by a unit distance is
called universal gravitational constant denoted by G measured in Nm2/kg2 1
2) Mathematically
KENDRIYA VIDYLAYA SANGATHAN::: RANCHI REGION

1
3) Force will become double 1
OR
Every object in the universe attracts every other object with a force which is proportional to
the product of their masses (m1*m2) and inversely proportional to the square of the
distance (d2) between them
KENDRIYA VIDYLAYA SANGATHAN::: RANCHI REGION

KENDRIYA VIDYALAYA SANGATHAN, RANCHI REGION


SESSION ENDING EXAMINATION (2022 – 2023)
BLUE PRINT
CLASS : IX
SET - A

Unit / Chapter Name 01 A-R 02 03 04 Sub- Total


Mark Questio Mar Mar Marks Total Unit
Questio ns ks ks Quest Chapt wise
ns & [Asserti Que Que ions er
MCQ on- stion stion Sourc wise
Reasoni s s e
ng] based
/ Case
based

No. of No. of No. No. No. of No. of


questio questio of of questi questi
ns ns ques ques ons ons
[Marks [Marks tions tions [Mark [Mark
allotted allotted [Mar [Mar s s
] ] ks ks allott allott
allot allot ed] ed]
ted] ted]

Unit-I Matter - Its 11(21)


Nature and Behaviour

Matter in Our 2(2) 1(2) 3(4)


Surroundings

Is Matter Around Us 1(3) 1(3)


Pure (Excluding
Separating the
components of a
mixture)

Atoms and Molecules 3(3) 1(3) 4(6)

Structure of Atom 1(1) + 1(3) 1(4) 3(8)


1(1)OR +1(4)
KENDRIYA VIDYLAYA SANGATHAN::: RANCHI REGION

OR

Unit-II Organisation in 08(17)


the Living World

Cell: Basic unit of Life 1(1) 1(1) 1(2) 1(3) 4(07)

Tissues, Organs, Organ 1(1)+ 3(9) 4(10)


system, Organism 1(1)OR +
2(6)
OR

Unit-III Motion, Force 17(35)


and Work

Motion 2(2) 1(1) 1(3) 4(6)

Force and Newton’s 1(2) 1(3) 2(5)


laws

Gravitation 1(1) 1(1) OR 2(4) 1(3) 4(8)

Floatation

Work, Energy & Power 1(1) OR 2(4) 2(6) 4(10)

Sound 2(2) 1(4) 3(6)


+1(4)
OR

Unit IV Food 6(7)


Production

(Improvement in Food 4(4) 1(1) 1(2) 6(7)


Resources) +1(1)O
R

Sub-Total 16(16) 4(4) 8 12 2(08) 42 42(80)


(16) (36) (80)
KENDRIYA VIDYLAYA SANGATHAN::: RANCHI REGION

KENDRIYA VIDYALAYA SANGATHAN, RANCHI REGION


SESSION ENDING EXAMINATION (2022-23)
CLASS –IX
SUBJECT: SCIENCE (086)
(SET-A)
TIME-03:00 Hrs MM-80
INSTRUCTIONS.
1. The question paper comprises 4 sections – A, B ,C & D. Attempt all sections.
2. All questions are compulsory however Internal choice is given in some questions of
some sections.
3. All questions in section A are carrying 1 mark comprising MCQ, VSA and Assertion –
Reason type. They are to be answered in one word or in one sentence.
4. All question in section B are of 02 marks each, short answer type questions.
5. All question in section B are of 03 marks each, short answer type questions.
6. All questions in section C are of 04 marks each.
SECTION-A

Q1) On Kelvin scale 0 0 C is equal to 1


a) 273K b) -273K c) 0 K d) 100 K
Q2) Molecular mass of CH3COOH is ….. 1
a) 60u b) 60.5u c) 65.5u d) 70.0u
Q3) The undefined nuclear region of prokaryotes are also known as 1
a) Nucleus b) nucleous c) nucleic acid d) nucleoid
Q4) (i) S.I unit of speed is 1
(a) m/s (b) s/m (c) m/s2 (d) none of these
OR
The work done by a man who lifts an object with a force of 20N to a distance 10m.
1
(a) 100J (b)200J (c)20J (d)2000J
Q5) 10 kgwt is equal to 1
a) 9.8 N b) 98N c) 0.98N d) 980N
KENDRIYA VIDYLAYA SANGATHAN::: RANCHI REGION

Q6) Preventive and control measures adopted for the storage of grains include 1
(a) strict cleaning (b) proper disjoining (c) fumigation (d) all of the
above
Q7) Which of the following is an Italian bee variety 1
a) Apis cerana indicia b) Apis mellifera
c) Apis dorsata d) Apis florae
OR
Pasturage is related to 1
(a) Cattle (b) Fishery (c) Apiculture (d) Poultry

Q8) An atom with 3 proton and 4 neutrons will have a valency of 1


a) 3 b) 7 c) 1 d) 4
1
Q9 Flexibility in plant is due to 1
a) Collenchyma b) Parenchyma c) Sclerenchyma d) aerenchyma
OR
Sieve tubes and companion cells are present in

(a)Xylem (b) Phloem (c) Cambium (d) Cork

Q10) When we change feeble sound into loud sound, we increase its 1
(a) frequency (b) amplitude (c) velocity (d) wavelength
Q11) Name foreign cattle breeds selected for long lactation period? 1
Q12) Why do liquid takes the shape of container in which they are kept? 1
Q13) A body is moving with a velocity of 10m/s .If the motion is uniform, what will be the
velocity after 10s? 1
Q14) What are the two forms of elemental oxygen found in the atmosphere? 1
Q15) What is a GM crop? 1
Q 16) What is reverberation? 1
Direction (Q17-20):-In the following questions, a statement of assertion (A) is followed by a
statement of Reason(R).Mark the correct choice as:
KENDRIYA VIDYLAYA SANGATHAN::: RANCHI REGION

(A) Both assertion (A) and reason(R) are true and reason(R) is the correct explanation of
assertion (A).
(B) Both assertion (A) and reason(R) are true and reason(R) is not the correct explanation
of assertion (A).
(C) Assertion (A) is true but reason(R) is false
(D) Assertion (A) is false but reason(R) is true

Q17) Assertion : Thomson’s atomic model is known as ‘raisin pudding’ model.


Reason : The atom is visualized as a pudding of positive charge with electrons (raisins)
embedded in it. 1
OR
Assertion : The size of the nucleus is very small as compared to the size of the atom.
Reason : The electrons revolve around the nucleus of the atom.
Q18) Assertion : Plasma membrane is selectively permeable. 1
Reason : Plasma membrane allows some molecules to pass through it more easily than others.
Q 19) Assertion : The displacement of an object can be either positive, negative or zero. 1
Reason : Displacement has both the magnitude and direction.
OR
Assertion: During a journey from the earth to the moon and back, maximum fuel is spent to
overcome the earth’s gravity at take-off.
Reason : Earth’s mass is much greater than that of the moon.

Q 20) Assertion- Wheat is a rabi crop. 1


Reason- Wheat completely grown in the month of November to April.
SECTION-B

Q 21) Define power & Write its formula. 2


Q 22) Why lysosomes are also known as suicide bag of cell? 2
Q 23) What are the difference between mass & weight? 2
Q 24) What do you mean by free fall? Briefly explain. 2
Q 25) How crop rotation helps in nutrient management? 2
Q 26) Why do steam cause more burn in comparison to boiling water? 2
Q 27) What is the first law of motion? 2
KENDRIYA VIDYLAYA SANGATHAN::: RANCHI REGION

Q 28) a) What type of energy is stored in the spring of a watch?


1+1
b) Give one example each of potential energy i) due to position ii) due to shape
SECTION-C

Q 29) Rain water stored in a tank containing sand grains, unfiltered clay particles, calcium
carbonate salt and pieces of paper and some air bubbles.
1+2
a) Find out a solvent, a solute, a colloidal and a suspension
b) Define any two of the above terms
Q 30) Write the functions of following cell organelles
1+1+1
A) Nucleus B)Ribosomes C) Smooth Endoplasmic Reticulum
OR
What are skeletal muscles? Write their frunction.
Q31) Give three differences between Simple and Complex permanent tissue
3
OR
In brief state what happens when
a) Number of WBC get decreased b) No bone present in body c) Nervous system got
damaged
Q32) A ball thrown up vertically returns to the thrower after 6 seconds find 1+1+1
a) the velocity of with which it was thrown up b) the maximum height it reaches
c) its position after 4 seconds
Q33) Deduce the equation showing the relation between g & G.
3

Q 34) In multicellular organism there are millions of cells. Most of the cells are specialized
to perform a selected few function each specialized cell function is taken up by a different
group of cells. Since these cells of a tissue carry out only a particular function, they do it
very efficiently. For example, in human beings, muscle cells contract and relax to cause
movement, nerve cells carry messages and blood flows to support oxygen, food, hormone
and waste material. In plants vascular tissues conduct water and food from one part of the
plant to another part, meristematic tissues regulate normal growth and development. Thus, it
KENDRIYA VIDYLAYA SANGATHAN::: RANCHI REGION

can be said that multicellular organisms show division of labour


3
a) Define term tissue
b) Name the tissue that
i) forms inner lining of the mouth in human
ii) store fat in our body
c) tissue A and B constitutes part C of plant body Here, A Transport water and B
transport food; identify tissue A, B,
Q 35) A moving object can do work. An object moving faster can do more work than an
identical object moving relatively slow. A moving bullet, blowing wind, a rotating wheel, a
speeding stone can do work. How does a bullet pierce the target? How does the wind move
the blades of a windmill? Objects in motion possess energy. We call this energy kinetic
energy. Thus, the kinetic energy possessed by an object of mass, m and moving with a
uniform velocity, v is KE = ½ *mv2 The energy possessed by an object is thus measured in
terms of its capacity of doing work. The unit of energy is, therefore, the same as that of work,
that is, joule (J). 1×3
(i) Which of the following has same unit?
(a) Potential energy and Force (b) Kinetic energy and work (c) Both a and b (d) None of
these
(ii) Kinetic energy depends
(a) Inversely on velocity of body (b) Directly on square of velocity of body
(c) Directly on velocity of body (d) None of these
(iii) Define kinetic energy of body. Give its SI unit
Q36) Write down the key points of :
1.5×2
a) Dalton’s atomic theory.
b) Neil Bohr’s experiment
Q 37) Label the a, b and c in the following diagram of areolar tissue: 3
KENDRIYA VIDYLAYA SANGATHAN::: RANCHI REGION

Q 38) a)The potential energy of freely falling object decreases progressively does this
violate the law of conservation of energy? Explain briefly.
1.5
b) A mass of 5Kg is taken from the ground to the height of 100m. Find the potential
energy of the object.
1.5
Q39). Define Momentum and find the momentum of an object of mass 5kg moving
with a velocity of 10m/s.
1+2
Q40) a) What are polyatomic ions? Give examples.
1+2
c) Calculate the formula unit mass of K2CO3 and Na2O (K=39u, C=12u, O=16u)

SECTION-D
Q41) What SONAR stands for. Briefly explain the working and two applications of
SONAR.
1+1+2
OR
a) Why sound waves are called mechanical waves 1
b) Define frequency 1
c) A sound Wave travels at a speed of 399m/s. Its wave length is 1.5cm. What is the
frequency of the wave? Will it be audible? 2

Q42) What is gold foil experiment? Name the scientist who performed this
experiment. Write the conclusions of gold foil experiment.
1+1+1+1

OR
The given figure depicts the atomic structure of an atom of an element “X “
KENDRIYA VIDYLAYA SANGATHAN::: RANCHI REGION

Write the following information about the element X


a) atomic number of X b) atomic mass of X
c) valency of X d) X should be metal or non-metal

------------------End of Paper----------------
KENDRIYA VIDYLAYA SANGATHAN::: RANCHI REGION

KENDRIYA VIDYALAYA SANGATHAN, RANCHI REGION


SESSION ENDING EXAMINATION (2022 – 2023)
MARKING SCHEME
CLASS : IX
SET - A

Q1 A 1
Q2 A 1
Q3 D 1
Q4 C OR B 1
Q5 B 1
Q6 D 1
Q7 B OR C 1
Q8 C 1
Q9 A OR B 1
Q10 B 1
Q11 Jersey and brown Swiss 1
Q12 Force of attraction between the particles & the 1
movement of particles cause the liquid to be variable
in shape or takes the shape of container.
Q13 Uniform , velocity remain same (10m/s) 1
Q14 Molecular oxygen(O2) ,ozone (O3) ½+1/2

Q15 Definition 1
Q16 Definition 1
KENDRIYA VIDYLAYA SANGATHAN::: RANCHI REGION

Q17 A or B 1
Q18 A 1
Q19 A OR A 1
Q20 A 1
Q21 Definition + P=W/t 1+1
Q22 Because when cell gets damaged lysosomes release 2
excessive digestive enzyme which kill cell it self.
Q23 Mass is scalar, weight vector, mass – kg, weight- 2
Newton
Q24 When any object falls towards earth only under the 2
influence of force of gravity.
Q25 Crop rotation is the practice of planting different crops 2
sequentially on the same plot of land to improve soil
health, optimize nutrients in the soil, and combat pest and
weed pressure.
Q26 steam has more heat energy than water due to its 2
latent heat of vaporisation and additionally latent heat of
condensation.
Q27 Newton's first law states that if a body is at rest or 2
moving at a constant speed in a straight line, it will
remain at rest or keep moving in a straight line at
constant speed unless it is acted upon by a force
Q28 When we wind a watch, the configuration of its spring is 1+1
changed. The energy stored in the spring is
obviously potential in nature (elastic potential energy to be
more accurate).
Potential energy due to position: Water stored in dam has
potential energy. Potential energy due to shape: In a toy
car, the wound spring possesses potential energy, and as
the spring is released, its potential energy changes into
kinetic energy due to which the car moves.or any other
related example.
Q29 a) Solvent—water, solute –salt, colloidal –air bubble 1+2
suspension—clay particles
KENDRIYA VIDYLAYA SANGATHAN::: RANCHI REGION

b)correct defination

Q30 Nucleus—control room of cell, Ribosome –Protein 3


synthesis, Smooth ER-- Lipid formation etc function
OR
Because they are connected with bones with the help
of tendon. They help in body movement.
Q31 Any three differences 3
OR OR
a) It would affect the functioning of antibody
formation, fighting against foreign bodies.etc. 1+1+1
b) Body won’t have shape and it will not
move.etc
c) We will not get any sense, Stimuli won’t
affect us, we loose thinking abilities.etc.
Q32 1+1+1
U=30m/s, h=45m (Approx.),
Its height above the ground, h′=45−5=40 m
Hence after 4 s, the ball is at a height of 40 m above
the ground.

Q33 Complete derivation g=GM/r2 2+1

Q34 a)Correct definition 1+1+1


b) ,Epithelial tissue, adipose tissue
c) A- Xylem, B-phloem ,
Q35 i) (b) 1+1+1
ii) b)
d) Definition of K.E. and its unit Joule(J)

Q36 a) Any points from daltons’s postulates 1.5


b) Importnats points of Neil bohr. 1.5
Q37 a- Fibroblast 1+1+1
b-Macrophages
KENDRIYA VIDYLAYA SANGATHAN::: RANCHI REGION

c-Mast cell

Q38 (A) No, sum of potential and kinetic energy equal 1.5
to constant with explanation.
(B) P.E=mgh , 5*10*100=5000J 1.5
Q39 Correct definition and P=m*v, 5*10=50kgm/s 1+2

Q40 Correct definition with example 1+2


Potassium carbonate 138u, Sodium oxide 62u
Q41 a) Correct full form 1+1+2
b) Explanation of working
c) Application (2)
OR
a) Correct answer, 1+1+2
b) Amplitude and frequency
c) correct definition
d)Frequency 22.6kHz
As frequency is more than 20kHz it will not audible
Q42 Correct explanation of gold foil experiment, name of 1+1+1+1
scientist ,
Conclusion and short comings
OR
a) At.no =8, b) at mass =18u , c)valency 2
d) non metal
KENDRIYA VIDYLAYA SANGATHAN::: RANCHI REGION

Blue Print as per CBSE Sample Paper

Unit / Chapter Name 01 A-R 02 03 05 04 Sub- Tota


Mark Ques Mark Marks Mark Marks Total l
Questi tions s Questi s Questi Chapt Unit
ons [Asse Ques ons Ques ons er wise
MCQ rtion tions tions Source wise
- based
Reas / Case
onin based
g]

No. of No. No. No. of No. No. of No. of


questi of of questio of questi questi
ons ques quest ns quest ons ons
[Mark tions ions [Marks ions [Marks [Mark
s [Mar [Mar allotte [Mar allotte s
allotte ks ks d] ks d] allotte
d] allott allott allott d]
ed] ed] ed]

Unit-I Matter - Its 13(2


Nature and Behaviour 5)

Matter in Our 1(1) 1(2)O 1(3) 3(6)


Surroundings R

Is Matter Around Us 1(1) 1(5)O 2(6)


Pure (Excluding R
Separating the
components of a
mixture)
KENDRIYA VIDYLAYA SANGATHAN::: RANCHI REGION

Atoms and Molecules 1(1) 1(1) 2(4) 4(6)

Structure of Atom 3(3) 1(4)OR 4(7)

Unit-II Organisation in 10(2


the Living World 2)

Cell: Basic unit of Life 2(2) 1(2)O 1(3) 1(4)OR 5(11)


R

Tissues, Organs, Organ 2(2) 1(1) 1(3)OR 1(5)O 5(11)


system, Organism R

Unit-III Motion, Force 14(2


and Work 7)

Motion 1(1) 1(3) 2(4)

Force and Newton’s 1(1) 2(4) 3(5)


laws

Gravitation 1(1) 1(4)OR 2(5)

Floatation 1(1) 1(3) 2(4)

Work, Energy & Power 1(1) 1(1) 1(3)OR 3(5)

Sound 1(1) 1(3) 2(4)

Unit IV Food Production 2(6)

(Improvement in Food 1(1) 1(5) 2(6)


Resources)

Sub-Total 16(16) 4(4) 6 (12) 7(21) 3(15) 3(12) 39


(80)

“OR” means that there is an alternate question in that particular question.


KENDRIYA VIDYLAYA SANGATHAN::: RANCHI REGION

# Please refer to Sample paper of Class X while preparing the question paper of class IX.

KENDRIYA VIDYALAYA SANGATHAN


RANCHI REGION
SAMPLE QUESTION PAPER
Class - IX TIME ALLOTTED - 3 Hrs
SUB - SCIENCE FULL MARKS - 80

General instructions :-
1. The question paper consists of 42 questions in four sections. All
questions are compulsory.
2. Section A comprises 20 questions carrying 1 mark.
3. Section B comprises 6 questions of 2 marks each.
4. Section C comprises 7 questions of 3 marks each.
5. Section D comprises 3 questions of 5 marks each.
6. Section E comprises 3 questions of 4 marks each.

SECTION A

1 All samples of water contain hydrogen and oxygen in the mass ratio 1
1:8. This is in agreement with the law of
a) Conservation of mass
b) Constant proportions
c) Multiple proportions
d) Gaseous volumes

2 Name different types of simple permanent tissues of plants. 1

3 A particle is moving in a circular path of radius r. The displacement 1


after half a circle would be
a) 0
b) πr
KENDRIYA VIDYLAYA SANGATHAN::: RANCHI REGION

c) 2r
d) 2πr

4 Main cellular site of ATP generation is ............ 1

5 Name the solute and solvent present in a solution of carbon dioxide in 1


water.

6 In Rutherford's gold foil experiment most of the alpha - particles pass 1


through the gold foil without any indicates that deviation from their
paths. This indicates that
a) the atom is spherical
b) there is a positively charged nucleus at the centre of the atom.
c) the entire mass of the atom is concentrated at the nucleus of
the atom.
d) most of the atom is an empty space

7 Write the units of G and g. 1

8 The process for the change of a solid directly into its vapour is called 1
a) Diffusion
b) Sublimation
c) Osmosis
d) Evaporation

9 In SONAR we use: 1
a) ultrasonic waves
b) infrasonic waves
c) radio waves
d) Audible sound waves

10 Two elements A and B have the same atomic mass but their atomic 1
numbers are 19 and 20 respectively. A and B are
a) isotopes
b) isomers
c) isobars
d) polymers

11 The unit of power is 1


a) Watt
b) Joule
c) Newton
d) kg

12 The undefined nuclear region of prokaryotes is also known as 1


a) nucleus
b) nucleic acid
c) nucleolus
d) nucleoid
KENDRIYA VIDYLAYA SANGATHAN::: RANCHI REGION

13 When a bus starts suddenly, the passengers standing on it lean 1


backwards in the bus. This is an example of
a) Newton's first law
b) Newton's second law
c) Newton's third law
d) none of the above

14 The particles present in the nucleus of an atom are 1


a) The proton and the electron
b) The electron and the neutron
c) The proton and the neutron
d) None of these

15 A body floats in a liquid if the buoyant force is 1


a) zero
b) greater than its weight
c) less than its weight
d) equal to its weight

16 Girth of stem increases due to 1


a) Apical meristem
b) lateral meristem
c) Intercalary meristem
d) vertical meristem

DIRECTION : For question numbers 16 to 20, two statements are given- one labelled
Assertion (A) and the other labelled Reason (R). Select the correct answer to these
questions from the codes (a), (b), (c) and (d) as given below.
a) Both A and R are true and R is the correct explanation of the A.
b) Both A and R are true but R is not the correct explanation of the A.
c) A is true but R is false.
d) Both A and R are false

17 Assertion : Different type of crops require different photoperiods. 1

Reason : photoperiods means duration of sunlight.

18 Assertion: Atomicity of ozone is three while that of oxygen is two. 1

Reason: Atomicity is the number of atoms constituting a molecule.

19 Assertion : Sometimes Parenchyma does photosynthesis hence it is 1


called chlorenchyma.

Reason : Parenchyma Is the most common simple permanent


tissue.
KENDRIYA VIDYLAYA SANGATHAN::: RANCHI REGION

20 Assertion : No work is done when a woman carrying a load on her 1


head, walks on a level road with a uniform velocity.

Reason : No work is done if force is perpendicular to the direction


of displacement.

SECTION B

21 A) State Newton's first law of motion. 1+1


B) A body of mass 25 kg has a momentum of 125 kg.m/s. Calculate
the velocity of the body.

22 A) Define valency of an element. 1+1


B) 5.2 g of substance A reacts with 2.7 g of substance B to produce
7.9 g of the substance AB. How would you justify that the result is in
agreement with the law of conservation of mass?

23 A) Why are lysosomes are known as suicide bags? 1+1


B) Identify the given figure and label X , Y.
OR

OR
A) Distinguish between prokaryotic and eukaryotic cells.

24 A)) state the law of constant proportions. 1+1


B) Define molecular mass. Calculate the formula unit mass of K2CO3.

25 What produces more severe burns, boiling water or steam having 1+1
the same temperature (100°C)? Give a reason for your answer?
OR OR
What is the reason that the smell of hot sizzling food reaches you at
a distance, but to get the smell of cold food you have to go near it. 2

26 Which would require a greater force accelerating a 2 kg mass at 5 2


m/s² or accelerating 4 kg mass at 2 m/s²?
KENDRIYA VIDYLAYA SANGATHAN::: RANCHI REGION

SECTION C

27 A) State the principle of flotation. 1+1+1

B) A floating boat displaces water weighing 600 kg.


i) What is the buoyant force on the boat? ( g = 10 m/s2)
ii) What is the weight of the boat?

28 What is the difference between parenchyma and collenchyma? 3


OR
What do you understand by the term permanent tissues in plants. OR
Name any two simple and two complex permanent tissues in plants.
2+1

29 A) State law of conservation of energy. 1+2


B) A man does 200 J of work in 10 seconds and a boy does 100 J of
work in 4 seconds. OR
a) Who is delivering more power?
b) Find the ratio of the power delivered by the man to that by
the boy.
OR
A) Define 1 watt of power.
B) Five 100-W bulbs are used for 10 hours every day for 30 days.
Find the cost of electricity if the rate is Rs 4.00/unit.

30 A) Define the term boiling and evaporation. What are the main 2+1
differences between the two?
B) Convert the following temperatures to the celsius scale.
a) 573 K
b) 270 K

31 A) Write the three equations of uniformly accelerated motion. 1+2

B) A car acquires a velocity of 72 km per hour in 10 seconds starting


from rest. Find (1) the acceleration, (ii) the average velocity, and (iii)
the distance travelled in this time.

32 A)) Draw a well labelled diagram of a plant cell. 2+1


B) Write the function of any two parts which differentiate it from an
animal cell.

33 A)) In which of the three media, air, water or iron, does sound travel 1+2
the fastest at a particular temperature?
B) Distinguish between loudness and intensity of sound.

SECTION D
KENDRIYA VIDYLAYA SANGATHAN::: RANCHI REGION

34 A) What are meristems? What happens to the plants if their tips are 2+3
removed?
B) What are the different types of meristematic tissues? Explain its OR
functions on the basis of their position with the help of a well
labelled diagram. 1+2+2
OR
A) Define muscle tissues.
B) Classify and explain function of different types of muscles.
C) distinguish between squamous and stratified squamous epithelial
tissue.

35 A) What are the main differences between elements, compounds 3+2


and mixtures, with the help of suitable examples.
B) Classify the following into elements, compounds and mixtures:
Air , Zinc , Water , Silver , Sugar solution, Carbon dioxide,

OR

A) What are the main differences between homogeneous mixture


and heterogeneous mixture ? Give suitable examples to justify your
answer.
B) A solution is made by dissolving 50 g of glucose in 250 g of water.
Calculate the concentration of the solution in mass percentage.

36 A) Give merits and demerits of fish culture. 2+2+1


B) Distinguish between fertilisers and manure.
C) What pattern of cropping does the field shows?

SECTION E

CASE BASED QUESTIONS:-


Instructions:- Question no. 34 to 36 consist of 4 questions. Answer these questions on the
basis of your understanding of the following paragraph and the related studied concepts.

37 The earth attracts every object with a certain force and this force
depends on the mass (m) of the object and the acceleration due to
the gravity (g). The weight of an object is the force with which it is
attracted towards the earth.
Mathematically,
KENDRIYA VIDYLAYA SANGATHAN::: RANCHI REGION

W=mxg

A) Direction of weight of any object is


a) Always towards centre of earth
b) Always away from centre of earth
c) Weight don’t have direction
d) None of these

B) Which of the following has same unit


a) Mass and weight
b) Weight and force
c) Velocity and acceleration
d) None of these

C) Whether weight is scalar quantity or vector quantity? Justify your


answer.

D) Differentiate between mass and weight.

OR

The universal law of gravitation was given by Newton. So, it is also


known as Newton's law of gravitation. The 'gravitational force' or '
gravity' is always a force of attraction between two objects (or two
bodies).
A) state universal law of gravitation.
B) what is the SI unit of universal gravitational constant?
C) What is universal gravitational constant?
D) Two objects of masses 10kg and 20kg separated by distance 10m.
What is gravitational force between them?

38 Arvind sometimes goes for a late evening walk with his father. While 1+1+1+
walking in the colony park, he observed many plants having colour 1
flowers. He also saw a few plants having white flowers. Most of these
flowers emit pleasant fragrance (i.e aroma). Out of curiosity, Arvind
asked his father the following questions:
A) What are the function of variously coloured flowers of plant?
B) What are the different types of plastids?
C) …………… is the site of photosynthesis.
D) Colourless plastids are known as…………

OR
Plant cells, in addition to the plasma membrane, have another rigid
outer covering called the cell wall. The cell wall lies outside the
plasma membrane. The plant cell wall is mainly composed of
cellulose.
A) Which of the following is the main constituent of the cell wall
KENDRIYA VIDYLAYA SANGATHAN::: RANCHI REGION

of plant
a) Protein
b) Lipids
c) Lipoproteins
d) Cellulose
B) What do you mean by plasmolysis?
C) Distinguish between cell wall and plasma membrane.
D) Membrane which allows passage of only solvent through it is
a) differentially permeable
b) semipermeable
c) Impermeable
d) both (a) and (b)

39 Many scientists contributed in revealing the presence of charged


particles in an atom.
It was known by 1900 that the atom was not a simple, indivisible
particle but contained at least one sub - atomic particle - the electron
identified by J.J. Thomson.

A) Four elements W, X, Y and Z contain 8, 11, 9 and 17 protons per


atom respectively. The element which cannot form an anion is most
likely to be: -
a) W
b) X
c) Y
d) Z
B) Write the electronic configuration of sodium?
C) Distinguish between electron, proton and neutron.

OR
Dalton’s atomic theory suggested that the atom was indivisible and
indestructible. But the discovery of two fundamental particles
(electrons and protons) inside the atom, led to the failure of this
aspect of Dalton’s atomic theory. It was then considered necessary to
know how electrons and protons are arranged within an atom. To
explain this, many scientists proposed various atomic models.

A) Draw a sketch of Bohr's atomic model with three shells.


B) Rutherford's alpha particle scattering experiment led to the
discovery of………
C) who discovered the subatomic particle, electron?
D) distinguish atomic number and mass number. Give examples to
justify your answer
KENDRIYA VIDYLAYA SANGATHAN::: RANCHI REGION

KENDRIYA VIDYALAYA SANGATHAN


RANCHI REGION
SAMPLE QUESTION PAPER
(MARKING SCHEME)
CLASS - IX TIME ALLOTTED - 3 Hrs
SUB - SCIENCE FULL MARKS - 80
SECTION A

1 b) Constant proportions 1

2 Parenchyma, collenchyma and selerenchyma. 1

3 c) 2r 1

4 Mitochondria 1

5 Solute - carbon dioxide 1


Solvent - water

6 d) most of the atom is an empty space. 1

7 Unit of G = Nm2/kg2 1
Unit of g = m/s2

8 b) Sublimation 1

9 a) ultrasonic waves 1

10 c) isobars 1

11 a) Watt 1

12 d) nucleoid 1
KENDRIYA VIDYLAYA SANGATHAN::: RANCHI REGION

13 a) Newton's first law 1

14 c) The proton and the neutron 1

15 d) equal to its weight 1

16 b) lateral meristem 1

17 B 1

18 A 1

19 A 1

20 A 1

SECTION B

21 A) Newton's first law of motion :- A body at rest will remain at rest, and a 1+1
body in motion will continue in motion in a straight line with a uniform
speed, unless it is compelled by an external force to change its state of
rest or of uniform motion.

B) momentum = mass × velocity


p=m×v
125 kg.m/s = 25 kg × v
v = 5 m/s

22 A) The valency of an element is defined as its capacity to combine with 1+1


other elements.

B) Given,
Mass of A = 5.2 g
Mass of B = 2.7 g
Mass of AB = 7.9 g

According to the law of conservation of mass


A + B —> AB
Total mass of substance A + substance B = 5.2 g + 2.7 g = 7.9 g
That is, Total mass of substance A + substance B = Mass of AB
Hence the result is in agreement with the law of conservation of mass.

23 A) Lysosomes contain hydrolytic enzymes ( digestive enzymes) for almost


all types of organic materials. If their covering membrane breaks as it
happens during injury to the cell, the digestive enzymes will spill over
the cell contents and digest the same. As lysosomes are organelles which
on bursting can kill cells possessing them, they are called suicide bags.
KENDRIYA VIDYLAYA SANGATHAN::: RANCHI REGION

B) Figure of nucleus of an eukaryotic cell. X-nucleolus; Y-chromatin.


OR

Prokaryotic cell Eukaryotic cell

1. Size of a cell is generally 1. Size of a cell is generally


small (1-10 mm). large (5-100 mm).
2. Nucleus is absent (Nuclear 2. Nucleus is present
region or nucleoid is not (Nuclear material is
surrounded by a nuclear surrounded by a nuclear
membrane). membrane).
3. It contains a single 3. It contains more than one
chromosome. chromosome.
4. Nucleolus is absent. 5. 4. Nucleolus is present.
Membrane bound cell 5. Membrane bound cell
organelles are absent. organelles such as
5. Cell division takes place by mitochondria, plastids,
fission or budding (no endoplasmic reticulum,
mitosis). Golgi apparatus,
lysosomes, peroxisomes,
etc., are present.
6. Cell division occurs by
mitotic or meiotic cell
division.

24 A) Law of constant proportions states that "In a chemical substance the


elements are always present in definite proportions by mass".
B) The molecular mass of a substance is the sum of the atomic masses of
all the atoms in a molecule of the substance.

Formula unit mass of K2CO3


= ( 2 × Atomic mass of K) + Atomic mass of C + ( 3 × Atomic mass of O)
= (2×39) + 12 + (3×16) u
= 78 +12 + 48 u
= 138 u

25 The burns caused by steam are more severe due to the fact that steam 2
contains more heat, in the form of latent heat, than boiling water.
OR
Due to the process of diffusion of gases released by food into the air.

26 we have , 2
m1 = 2 kg; a1 = 5 m/s2
m2 = 4 kg; b a₂ = 2 m/s2

F1 = m1 × a1 = 2 kg x 5 m/s² = 10 N
KENDRIYA VIDYLAYA SANGATHAN::: RANCHI REGION

F2 = m2 × a₂ = 4 kg x 2 m/s2 = 8 N.
So, F1 > F2
Thus, accelerating a 2 kg mass at 5 m/s² would require a greater force.

SECTION C

27 A) According to the principle of flotation - An object will float in a liquid


if the weight of the object is equal to the weight of liquid displaced by it.
Weight of object = Weight of liquid displaced by it

B) i) According to the Archimedes principle - The magnitude of buoyant


force acting on an object immersed in a liquid is equal to the weight of
liquid displaced by the immersed object.
Given, m = 600 kg ; g = 10 m/s2
Weight of water displaced , W = m × g = 600 × 10 N = 6000 N

Buoyant force acting on an object = weight of liquid displaced by that


object,
So , buoyant force on the boat = 6000 N

ii) Weight of object = Weight of liquid displaced by it


so , weight of the boat = 6000 N

28 Differences between parenchyma collenchyma and sclerenchyma


OR
Differences between meristematic and permanent tissues.
Simple permanent tissues - parenchyma, collenchyma and sclerenchyma
Complex permanent tissues - xylem and phloem

29 A)Principle of conservation of energy - Energy can neither be created


nor destroyed.It can only be changed from one form to another.

B) a) Power delivered by the man , Pman = 200 J / 10 s = 20 W


Power delivered by the boy , Pboy = 100 J / 4 s = 25 W
So, the boy is delivering more power.

(b) The ratio = 20/25 = 4/5


OR
KENDRIYA VIDYLAYA SANGATHAN::: RANCHI REGION

A) 1 watt is the power of an appliance which does work at the rate of 1


joule per second (or which consumes energy at the rate of 1 joule per
second). That is:

1 watt of power = 1 joule work / 1 second

B) For each bulb, power = P = 100 W = 100/1000 kW


The energy consumed by a bulb every day = U = Pt = 100/1000kW x 10
h = 1 kWh

so, the energy consumed by 5 bulbs in 30 days = 5 x 1 kWh x 30 = 150


kWh.

The cost of electricity at the rate of Rs 4.00/kWh is

(Rs 4.00/kWh) x (150 kWh)= Rs 600.00.

30 A)
Evaporation Boiling

1) It is a spontaneous process 1) It occurs at a particular


occurring at all temperatures. temperature, called the
boiling point of the liquid.

2) It occurs only at the surface 2) It occurs throughout the


of the liquid. mass of the liquid with the
formation of bubbles.

3) It causes cooling effect. 3) It does not produce cooling


effect.

B) a) 573 K = 300°C
b) 270 K = -3°C

31 A) first equation, v = u + at , second equation, s = ut + ½ at2 , third


equation , v2 = u2 + 2as

B) i) 2 m/s2
ii) 10 m/s
iii) 100 m

32 Well labelled diagram of a plant cell. 2+1


Function of any two parts which differentiate it from an animal cell.
KENDRIYA VIDYLAYA SANGATHAN::: RANCHI REGION

33 A) sound travels the fastest in a solid medium. Out of air, water and iron, 1+2
iron is a solid, therefore, sound travels fastest in iron.

B) loudness of sound -
- The sensation produced in the ears which enables us to
distinguish between a faint sound (feeble sound) and a loud
sound is called loudness of sound.
- Loudness of sound is measured in the unit of decibel (dB)
- Loudness of sound depends on the sensitivity of ears

Intensity of sound -
- The average energy transported by a sound wave per second per
unit area (perpendicular to the direction of propagation) is called
intensity of sound.
- Intensity of sound is measured in the unit of watts per square
metre (W/m²)
- Intensity of sound does not depend on the sensitivity of the ears.

SECTION D

34 A) Cells of meristems divide continuously and help in increasing the


length and girth of the plant. If apical meristem is damaged or cut, then
growth in length of the plant will stop.
B)

Function of apical, lateral and Intercalary meristem.


OR
A) The muscle tissues or muscles of the body form the contractile tissue
and are made of muscle cells.
B) On the basis of their location, structure and function, there are
following three types of muscle fibres: (i) Striated muscles; (ii) Smooth
muscles; (iii) Cardiac muscles.
C) difference between squamous and stratified squamous epithelial
tissue

35 A) Element :- 3+2
- An element is a substance which cannot be split into two or
more different substances.
KENDRIYA VIDYLAYA SANGATHAN::: RANCHI REGION

- An element is formed from atoms of the same kind.


- Examples - gold , iron etc.

Compound :-
- A compound can be split into two or more different substances.
- A compound is formed from atoms of different kinds.
- It is always homogeneous.
- Examples - water, carbon dioxide, ammonia etc.

Mixture :-
- It is made up of two or more pure substances mixed in any
proportion.
- The constituents of a mixture can be separated by simple
physical processes.
- It may be homogeneous or heterogeneous.
- Examples - salt in water solution, air etc.

B) elements - Silver, zinc. Compound - water, carbon dioxide,. Mixture -


sugar solution, Air .
OR

A) homogeneous mixture:-
- Those mixtures in which the substances are completely mixed
together and are indistinguishable from one another, are called
homogeneous mixtures.
- Uniform composition
- All homogeneous mixture are solution.
- Examples - sugar solution, salt solution etc.
Heterogeneous mixture :-
- Those mixtures in which the substances remain separate and
one substance is spread throughout the other substance as small
particles, droplets or bubbles, are called heterogeneous
mixtures.
- Non uniform composition
- All the suspension and colloids are heterogeneous.
- Examples - sugar - sand mixture, chalk in water etc.

B) Mass of solute = 50g


Mass of solvent = 250g
Mass of solution = 50 + 250 = 300
Concentration of solution = 50×100/300 = 16.6%

36 A) Merits -
a) Economically important desired fishes are made available.
b) A large number of fishes are raised in a small area.
c) Fishes are made to breed in different seasons.
d) There is a little mortality in the younger stages of the fishes.
KENDRIYA VIDYLAYA SANGATHAN::: RANCHI REGION

e) Through selective hybridisation, yield and quality of fishes are


improved.

Demerits -
a) Only some selected high yielding and economically important
breeds are reared.
b) Fish culture is a threat to biodiversity, since other forms of
aquatic organisms are being ignored. Even natural waters are
being seeded with economically important fishes; this too is
affecting the natural biodiversity of the water bodies.

B) Difference between fertilisers and manure.


C) Intercropping of maize and soyabean.

SECTION E

37 A) a)
B) b)
C) Weight is a vector quantity as it has magnitude as well as direction
which is always towards the centre of the earth.
D) Difference between mass and weight
OR
A) Every body in the universe attracts every other body with a force
which is directly proportional to the product of their masses and
inversely proportional to the square of the distance between them.
B) Universal gravitational constant denoted by G measured in
Nm2/kg2.
C) The force of attraction between any two unit masses separated by a
unit distance is called universal gravitational constant.
D) 13.34 × 10-11 N

38 A) Plants have variously coloured flowers to attract insects and other


animals for pollination.
B) plastids are of three types -
1) Chromoplasts - Coloured plastids (except green colour).
2) Chloroplasts - Green-coloured plastids.
3) Leucoplasts - The colourless plastids.
C) Chloroplast
D) Leucoplast
OR
A) c) lipoproteins
B) When a living plant cell loses water through osmosis there is
contraction of the contents of the cell away from the cell wall. This
phenomenon is called plasmolysis.
C) difference between cell wall and plasma membrane
D) b) semipermeable.
KENDRIYA VIDYLAYA SANGATHAN::: RANCHI REGION

39 A) b)
B) 2, 8, 1
C) difference between proton, neutron and electron

OR
A) diagram with labelled name.
B) nucleus
C) J. J. Thomson
D) difference between atomic number and mass number with example

KENDRIYA VIDYALAYA SANGATHAN RANCHI REGION


SAMPLE QUESTION PAPER (2022-23)
CLASS IX SCIENCE (086)
Max. Marks: 80 Time allowed: 3 hrs
General instructions:-
i. The question paper consists of 39 questions in 5 sections.
ii. All questions are compulsory. However internal choices provided in some questions. A
student is expected to attempt only one of these questions.
iii. Section A consists of 20 objective type questions carrying 1 mark each.
iv. Section B consists of 6 very short questions carrying 02 marks each. Answer to these
questions should be in the range of 30 to 50 words.
v. Section C consists of 7 short answer type questions of 03 marks each. Answer to these
questions should be in the range of 50 to 80 words.
vi. Section D consist of 3 long type questions of 05 marks each. Answer to these
questions should be in the range of 80 to 120 words.
vii. Section E comprises 3 source-based/case-based questions of 04 marks each with sub-
parts.
SECTION A
Select and write one most appropriate option out of the four options given for each of the
question 1 -20
1 A solution of urea in water contains 16 grams of it in 120 grams of solution. 1
What will be the mass % of the solution?
(a) 12% (b) 13.33%
(c) 7.5% (d) 15.33%

2 Two chemical species X and Y combine together to form a product P which 1


contains both X and Y
X+Y→P
X and Y cannot be broken down into simpler substances by simple chemical
reactions. Which of the following concerning the species X, Y and P are
correct?
(i) P is a compound
(ii) X and Y are compounds
KENDRIYA VIDYLAYA SANGATHAN::: RANCHI REGION

(iii) X and Y are elements


(iv) P has a fixed composition
(a) (i), (ii) and (iii)
(b) (i), (ii) and (iv)
(c) (ii), (iii) and (iv)
(d) (i), (iii) and (iv)

3 A box contains some identical red colour balls labeled as A each weighing 2 1
g. Another box contains identical blue colored balls, labeled as B, each
weighing 5 g. In the combinations AB, AB2, A2B and A2B3 which is
applicable?
(a) Law of Definite proportion
(b) Law of multiple proportion
(c) Law of conservation of mass
(d) None of the above
4 Why was the Thomson’s Model of an atom failed? 1
i. It could not explain the screening of negative charges from that of positive
ii. It did not tell about the presence of electrons
iii. It did not give an idea about the discrete energy levels
iv. It explained the atom as a whole to be electrically neutral.
Choose the correct option from the following:
(a) Only (iii)
(b) Both (i) & (iii)
(c) Only (i)
(d) Both (ii) & (iv)

5 What prevents an atom from being collapsed? 1


(a) The nuclear forces
(b) The electron-electron repulsions
(c) Movement of electrons in discrete energy levels
(d) All of these

6 Which radioactive element is used in the treatment of cancer? 1


(a) Iodine-131 (b) Uranium-234
(c) Plutonium-239 (d) Cobalt-60

7 Lipids are manufactured by 1


(a) Plasma membrane (b) Golgi apparatus
(c) Rough endoplasmic reticulum (d) Smooth endoplasmic
reticulum

8 A cell will swell up if 1


(a) The concentration of water molecules in the cell is higher than the
concentration of water molecules in the surrounding medium.
(b) The concentration of water molecules in the surrounding medium
is higher than the concentration of water molecules in the cell.
(c) The concentration of water molecules is same in the cell and in the
surrounding medium.
KENDRIYA VIDYLAYA SANGATHAN::: RANCHI REGION

(d) Concentration of water molecules does not matter.

9 Which tissue makes up the husk of coconut? 1


(a) Collenchyma (b) Sclerenchyma
(c) Parenchyma (d) Chlorenchyma

10 Which of the following is not the element of Xylem tissue? 1


(a) Tracheid (b) Companion cell
(c) Vessels (d) Xylem fibers

11 What does the path of an object look like when it is in uniform motion? 1
(a) A curved line (b) A straight line
(c) An inverted line (d) none

12 Which of the following graph show that the body is at rest? 1


(a) (b) (c)

13 An object of mass 10 kg is at a point A on a table. It is moved to a point B 1


by a distance 5 m. If the line joining A and B is horizontal, then what is
the work done on the object by the gravitational force?
(A) 50 J (B) 100 J
(C) 60 J (D) Zero

14 A body experiences an upthrust F1 in seawater and F2 in river water when 1


dipped up to the same level. Which is more, F1 or F2?
(a) F2 = F1 (b) F2 > F1
(c) F2 < F1 (d) Cannot be determined

15 A ball is dropped from a height of 10 m. 1


(a) Its potential energy increases and kinetic energy decreases during the falls
(b) Its potential energy is equal to the kinetic energy during the fall.
(c) The potential energy decreases and the kinetic energy increases during the
fall.
(d) The potential energy is 0 and kinetic energy is maximum while it is
falling.
KENDRIYA VIDYLAYA SANGATHAN::: RANCHI REGION

16 The number of compressions or rarefactions per unit time gives ———– 1

a. Frequency
b. Time period
c. Amplitude
d. Pitch

Q. no 17 to 20 are Assertion-Reasoning based questions. These consist of two statements -


Assertion (A) and Reason (R). Answer these questions selecting the appropriate option given
below:
a) Both A and R are true and R is the correct explanation of the A.
b) Both A and R are true but R is not the correct explanation of the A.
c) A is true but R is false.
d) A is false but R is true.
17 Assertion: A molecule is the smallest particle of an element or a compound 1
which is capable of free existence.
Reason: The number of atoms present in one molecule of the substance is
called its atomicity.
18 Assertion : The inner lining of intestine has tall epithelial cells. 1
Reason : Columnar epithelium facilitates absorption and secretion.
19 Assertion : A spring has potential energy, both when it is compressed or 1
stretched.
Reason : In compressing or stretching, work is done on the spring against the
restoring force.
20 Assertion: Cattle breeds can be improved by superovulation and embryo 1
transplantaion.
Reason : Superovulation in high milk-yielding cows is induced by hormonal
injection.
SECTION -B
Q. no 21 to 26 are very short answer questions.
21 Substance ‘A’ has high compressibility and can be easily liquefied. It can take up 2
the shape of any container. Predict the nature of the substance. Enlist two properties
of this state of matter.
OR
Why does the temperature of a substance remain constant during its melting point
or boiling point?

22 What is the difference between cation and anion? 2

23 In magnesium sulphide, the ratio by mass of Mg and S is 3 : 4. What is the ratio of 2


the number of Mg and S atoms ?
KENDRIYA VIDYLAYA SANGATHAN::: RANCHI REGION

24 1. Identify the above figure. 2

2. Label X and Y

OR
If you are provided with some vegetables to cook, you generally add salt into the
vegetables. After adding salt, vegetables release water. Why?

25 Distance-time graph of a moving body is shown in figure. How much force is 2


acting on the body ?

26 The force of friction between the surface of a floor and the surface of a box 2
in contact with the floor in 200 N. We wish to move the box on the floor with
constant velocity. How much force has to be applied on the box ?

SECTION – C
Q.no- 27 TO 33 are short answer questions.
27 The diagram shows an experiment in which gases hydrogen and carbon 3
dioxide are placed in two jars as shown in the figure. If the lid separating the
two jars be removed, what will the constituents in the gas jar A after a few
minutes and why?
KENDRIYA VIDYLAYA SANGATHAN::: RANCHI REGION

(a) carbon dioxide only


(b) hydrogen only
(c) mixture of carbon dioxide and hydrogen.

28 1. Identify A and B cells. 3

2. What will happen if B cells are kept in hypotonic solution?


3. What will happen if A cells are kept in hypertonic solution?

29 3

Identify the figure. What is its function?


Label X, Y and Z.

OR

What will happen if

1. Apical meristem is damaged or cut ?


2. Cork is not formed in older stems and roots ?
3. Lymph is not returned to blood ?

30 Give an example of a motion in which average velocity is zero, but the 3


average speed is not zero.
KENDRIYA VIDYLAYA SANGATHAN::: RANCHI REGION

31 Loaded test-tube placed in pure milk sinks to a certain mark (M). Now some 3
water is mixed with the milk. Will the test tube sink more or less? Explain.

32 A spring which is kept compressed by tying its ends together is allowed to 3


be dissolved in an acid. What happens to the potential energy of the
spring ?

OR

When is the work done by a force said to be negative? Give one situation
in which one of the forces acting on the object is doing positive work and
the other is doing negative work.
33 Represent the following sound waves, 3
(i) Waves having same amplitude but different frequencies.
(ii) Waves having same frequency but different amplitudes.

(iii) Waves having different amplitudes and different wave lengths


SECTION D
Q.no. 34 to 36 are long answer questions.
34 Iron filings and sulphur were mixed together and divided into two parts ‘A’ and 5
‘S’. Part ‘A’ was heated strongly while Part ‘S’ was not heated. Dilute
hydrochloric acid was added to both the parts and evolution of gas was seen in
both the cases. How will you identify the gases evolved?
OR
Calculate the mass of sodium sulphate required to prepare its 20% (mass per
cent) solution in 100 g of water.
35 Differentiate between striated, unstriated and cardiac muscle fibres. 5

OR
1.Identify figures : A,B and C
2. Which one of them provides both mechanical strength as well as flexibility
3.Which one of them is commercially exploited to obtain Hemp and Jute ?
4. Which one of them can be modified to form air cavities in aquatic plants?
5. Which one of them has heavy deposition of lignin ?

36 Figure shows the two crop fields [plots A and B] have been treated by manures 5
and chemical fertilisers respectively, keeping other environmental factors same.
Observe the graph and answer the following questions.
(i) Why does plot B show sudden increase and then gradual decrease in yield?
(ii) Why is the highest peak in plot A graph slightly delayed?
(iii) What is the reason for the different pattern of the two graphs?
KENDRIYA VIDYLAYA SANGATHAN::: RANCHI REGION

SECTION E
Q.no. 37 to 39 is Case based/ data based questions.
37
Neils Bohr got the Nobel Prize for his work on the structure of atom in 1922. Among
Professor Bohr’s numerous writings, three appearing as books are: (i) The Theory of
Spectra and Atomic Constitution,
(ii) Atomic Theory and, (iii) The Description of Nature.

In order to overcome the objections raised against Rutherford’s model of the atom,
Neil’s Bohr put forward the following postulates about the model of an atom:
• Only certain special orbits known as discrete orbits of electrons, are allowed
inside the atom.
• While revolving in discrete orbits the electrons do not radiate energy. These orbits
or shells are called energy levels. Energy levels in an atom are shown in Fig. A
few energy levels in an atom These orbits or shells are represented by the letters
K,L,M,N,… or the numbers, n=1,2,3,4,….

(1) The orbits or shells are represented by 1

(a) Letters (b) Numbers


(c) Both a & b (d) Special symbols

(2) These orbits or shells are called 1

(a) Energy levels (b) Discrete orbit


(c) Atomic levels (d) None of the above
(3) Which of the following book is written by Professor Bohr’s 1

(a) The Theory of Spectra and Atomic Constitution (b) Atomic Theory
(c) The Description of Nature (d) All of the above
KENDRIYA VIDYLAYA SANGATHAN::: RANCHI REGION

(4) Identify the correct statement. 1

Statement 1 – The orbits or shells are represented by letters only.


Statement 2 – The orbits or shells are represented by numbers only.
Statement 3 – While revolving in discrete orbits the electrons do not radiate
energy.
Statement 4 – Certain special orbits known as discrete orbits of electrons.
(a) Both 1 & 2 (b) Both 3 & 4
(c) Only 3 (d) All of the above
OR

The electrons present in the outermost shell of an atom are known as the valence
electrons. From the Bohr-Bury scheme, we also know that the outermost shell of an
atom can accommodate a maximum of 8 electrons. It was observed that the atoms of
elements,
completely filled with 8 electrons in the outermost shell show little chemical
activity.
In other words, their combining capacity or valency is zero.

(1) Valence electrons present in calcium are 1


(a) 3 (b) 2 (c) 1 (d) 7
(2) The inert gas has valence electron 1
(a) 0 (b) 9 (c) 8 (d) 7

(3) Which is most reactive element 1


(a) Na (b) Mg (c) Zn (d) Hg
(4) The valency of inert gas is 1
(a) 3 (b) 0 (c) 4 (d) 7

38 Plant cells, in addition to the plasma membrane, have another rigid outer covering
called the cell wall. The cell wall lies outside the plasma membrane. The plant cell
wall is mainly composed of cellulose. Cellulose is a complex substance and provides
structural strength to plants. When a living plant cell loses water through osmosis
there is shrinkage or contraction of the contents of the cell away from the cell wall.
This phenomenon is known as plasmolysis.
(1) Which of the following is the main constituent of cell wall? 1

(a) Proteins (b) Lipids


(c) Lipoproteins (d) Cellulose

(2) Which of the following is outer most covering of the plant cell? 1

(a) Cell membrane (b) Plasma membrane


(c) Cell wall (d) Cellulose
KENDRIYA VIDYLAYA SANGATHAN::: RANCHI REGION

(3) What is mean by plasmolysis? 1

(4) What is the reason behind structural strength of plant cell? 1

OR
The Golgi apparatus, first described by Camilo Golgi, consists of a system of
membrane-bound vesicles (flattened sacs) arranged approximately parallel to each
other in stacks called cisterns. These membranes often have connections with the
membranes of ER and therefore constitute another portion of a complex cellular
membrane system. The material synthesised near the ER is packaged and dispatched
to various targets inside and outside the cell through the Golgi apparatus. Its
functions include the storage, modification and packaging of products in vesicles. In
some cases, complex sugars may be made from simple sugars in the Golgi
apparatus. The Golgi apparatus is also involved in the formation of lysosomes
(1) Who discovered endoplasmic reticulum? 1

(a) Porter and Thompson (b) Robert Brown


(c) Robert Hooke (d) Camilo Golgi
(2) A system of membrane-bound flattened sacs arranged approximately parallel 1
to each other in stacks are called as _________
(a) Cisterns (b) Vesicles
(c) Golgi complex (d) Vacuoles
(3) Membrane bound flattened sacs is termed as _________ 1

(a) Cisterns (b) Vesicles


(c) Golgi complex (d) Vacuoles
(4) Enlist the function of Golgi apparatus. 1

39 Archimedes’ principle, stated as follows: When a body is immersed fully or


partiallyin a fluid, it experiences an upward force thatis equal to the weight of the
fluid displacedby it. The upward force is known as up thrust or buoyant force. In
fact, all objects experience a force of buoyancy when they are immersed in a fluid.
The magnitude of this buoyant force depends on the density of the fluid.Objects
having density less than that of the liquid in which they are immersed float on the
surface of the liquid. If the density of the object is more than the density of the liquid
in which it is immersed then it sinks in the liquid. Hence body will float or sink
depends upon difference between density of body and fluid.
(1) The up thrust of the body is equal to the 1

(a) Mass of liquid (b) Weight of liquid


(c) Weight of liquid displaced by body (d) None of these
KENDRIYA VIDYLAYA SANGATHAN::: RANCHI REGION

(2) If the density of the object is more than the density of the liquid in which it is 1
immersed then
(a) It sinks in liquid (b) It floats on liquid
(c) It comes out of liquid (d) None of these

(3) When anybody immersed in liquid it experience a force called as 1

(a) Gravitational force (b) Buoyancy force


(c) Nuclear force (d) None of these

(4) Why does cube of plastic released deep down under the water come up to 1
surface of water?
OR

We know that the earth attracts every object with a certain force and this force depends on
the mass (m) of the object and the acceleration due to the gravity (g). The weight of an
object is the force with which it is attracted towards the earth. As the weight of an object is
the force with which it is attracted towards the earth, the SI unit of weight is the same as
that of force, that is, Newton (N). The weight is a force acting vertically downwards; it has
both magnitude and direction. We have learnt that the value of g is constant at a given
place. Therefore at a given place, the weight of an object is directly proportional to the
mass, say m, of the object, that is, W αm. It is due to this reason that at a given place, we
can use the weight of an object as a measure of its mass.
(1) Unit of acceleration due to the gravity (g) is 1

(a) m/s (b) m/s2


(c) Newton(N) (d) None of these

(2) Direction of weight of any object is 1


(a) Always towards centre of earth (b) Always away from centre of earth
(c) Weight don’t have direction (d) None of these

(3) Which of the following has same unit 1

(a) Mass and weight (b) Weight and force


(c) Velocity and acceleration (d) None of these

(4) Whether weight is scalar quantity or vector quantity? Justify your answer. 1
KENDRIYA VIDYLAYA SANGATHAN::: RANCHI REGION

MARKING SCHEME
KENDRIYA VIDYALAYA SANGATHAN RANCHI REGION
SAMPLE Q. PAPER (2022-23)
CLASS IX SCIENCE (086)
Q.NO ANSWER MARK
. S
SECTION A 1
1 (b) 13.33% 1
2 (d) (i), (iii) and (iv) 1
3 (b) Law of multiple proportion 1
4 (b) Both (i) & (iii) 1
5 (c) Movement of electrons in discrete energy levels 1
6 (d) Cobalt-60 1
7 (d) Smooth endoplasmic reticulum 1
8 (a) The concentration of water molecules in the cell is higher than the 1
concentration of water molecules in the surrounding medium.

9 (b) Sclerenchyma 1
10 (b) Companion cell 1
11 (b) A straight line 1
12 1
13 (D) Zero 1
14 (b) F2 > F1 1
15 (c) The potential energy decreases and the kinetic energy increases 1
during the fall.

16 (d) Pitch 1
17 b) Both A and R are true but R is not the correct explanation of the A. 1
18 a) Both A and R are true and R is the correct explanation of the A 1
KENDRIYA VIDYLAYA SANGATHAN::: RANCHI REGION

19 a) Both A and R are true and R is the correct explanation of the A. 1

20 b) Both A and R are true but R is not the correct explanation of the A 1
SECTION -B
21 A’ is a gas. 2
Properties of gases:

They do not have fixed shape and fixed volume.

They have large interparticle space.

They have least forces of attraction between the molecules.

They are highly compressible.

OR
The temperature of a substance remains constant during its melting and
boiling point because the change in any state of matter, as solid to
liquid or liquid to gas, involves crossing the latent heat of fusion which
causes a difference in the intermolecular spacing of the molecules in
the substance.
22 1.A cation refers to an atom or a group of atoms that bear one or 2
multiple positive electric charge
2. An anion refers to an atom or a group of atoms that bear one or
multiple 2electric charges. ( Any two)
23 2

24 1. It is figure of Nucleus. 2

2.X- Nucleous.Y-Chromatin
OR

OR

When salt is added, a hypotonic medium is created, i.e., the


concentration of salt molecules is more outside the vegetables than
inside. Hence, due to osmosis water from the vegetables come out.
25 From the graph, it is clear that the body is moving with uniform velocity i.e. 2
constant velocity. Hence acceleration of body is zero. Therefore, net force
acting on the body is zero. [∴ F = ma]

26 The box will move with a constant velocity if no net external force acts on the 2
body. Thus, the effect of force of friction has to be balanced. So a force equal
to the force of friction (i.e., 200 N) but opposite in direction has to be applied
KENDRIYA VIDYLAYA SANGATHAN::: RANCHI REGION

on the box to move it with a constant velocity. So the applied force on the box
= 200 N

SECTION -C

27 (c) The gas jar A will contain both the gases carbon dioxide and hydrogen.
Actually, the molecules of the gas present in one jar will move into the empty
spaces present in the other jar and vice versa

28 . A cells- Turgid cells. B cells- Plasmolysed cells.

B cells kept in hypotonic solution will become deplasmolysed if done


so immediately after plasmolysis.

A cells kept in hypertonic solution will become plasmolysed

29 X – Sieve tube cell Y – Sieve plate


Z – Companion cell. OR

1. Apical Meristem Removed: Growth in length will stop.


2. Cork is Not Formed: If cork is not formed in older stems and
roots, the outer tissues will rupture with the increase in girth and
expose the interior to desiccation and infection.
3. Lymph is Not Returned to Blood: Blood volume will decrease
while passage of materials from tissues to blood and vice versa
would be dislocated.

30 . Since, the total distance travelled by a moving body increases with


the time, so average speed of a moving body can not be zero.

Since total displacement of a moving body can be zero, so the


average velocity of a moving body can be zero.
Example. When an athlete completes one round in a circular track in
the given time interval, then his total displacement is zero but total
distance travelled is equal to the length of the circular track. Hence,
his average velocity is zero but his average speed is not zero.
31 As density of milk is greater than density of water, so when some
water is mixed with milk, then amount of buoyant force is decreased
at some extent. Consequently test tube will sink more.
32 The potential energy of the spring gets converted into heat energy
(kinetic energy of acid molecules). Due to this heat, the temperature
of the acid rises. OR

We know that work done W = FS cos 0, where 0 is the angle


between F and S. Clearly, W will be -ve, if 0 is between 90° and
180° because then cos 0 will be -ve. Consider the case of a body
KENDRIYA VIDYLAYA SANGATHAN::: RANCHI REGION

falling under gravity. The body experiences an upward frictional


force and downward force due to gravity. Since the body is moving
downwards, the work done by force to gravity will be +ve but that is
against the upward thrust will be -ve.
33

SECTION D
34 . Part A
Fe(s)+S(s)⟶HeatFeS(s)
FeS(s) + 2HCl(aq) → FeCl2(aq) + H2S(g)
Part B
Fe(i) + S(s) → Mixture of iron filings and sulphur
When dilute HCl is added to it
Fe(s) + S(s) + 2HCl(aq) → FeCl2(aq) + H2(g)
Sulphur remains unreacted.
H2S gas formed has a foul smell and on passing through lead acetate
solution, it turns the solution black. Hydrogen gas burns with a pop
sound. OR

Let the mass of sodium sulphate required be x g.


The mass of solution would be = (x + 100)g

So, the mass of sodium sulphate required is 25 g

35

OR
KENDRIYA VIDYLAYA SANGATHAN::: RANCHI REGION

1. A- T.S sclerenchyma fibres.


B- T.S parenchyma cells.
C- T. S collenchyma.
2. Mechanical strength and flexibility: Collenchyma.
3. Hemp and Jute: Sclerenchyma fibres.
4. Aerenchyma: Modification of parenchyma.
5. Deposition of Lignin: Sclerenchyma.

36 (i) With the addition of chemical fertilisers there is sudden increase in yield
due to release of nutrients N,P,K, etc. in high quantity. The gradual decline in
the graph may be due to continuous use and high quantity of chemicals which
kills microbes useful for replenishing the organic matter in the soil. This
decreases the soil fertility.
(ii) Manures supply small quantities of nutrients to the soil slowly as it
contains large amounts of organic matter [Hint: Importance of organic matter
can be included]. It enriches soil with nutrients; thereby increasing soil
fertility continuously.
(iii) The differences in the two graphs indicate that use of manure is beneficial
for long duration in cropping as the yield tends to remain high when the
quantity of manure increases.

In case of plot B the chemical fertilisers may cause various problems


when used continuously for long time. Loss of microbial activity
reduces decomposition of organic matter and as a result, soil fertility is
lost that affects the yield

SECTION E

37 1) c (2) a (3) d (4) b


Or
1) b (2) c (3) a (4) b

38 .(1) d (2) c
(3). When living plant cell loses water through osmosis there is contraction of
the contents of the cell away from the cell wall. This phenomenon is called as
plasmolysis.

(4)The plant cell wall is mainly composed of cellulose. Cellulose is a


complex substance and provides structural strength to plants.
Or
(1) d (2) a (3) b
(4) Functions of Golgi apparatus:

• Storage, modification and packaging of products


• Involved in formation of lysosomes
KENDRIYA VIDYLAYA SANGATHAN::: RANCHI REGION

39 (1) c (2) a (3) c


(4) Cube of plastic released deep down under the water come up to
surface of water because it has less density than that of water also it
experiences upward buoyancy force on it due to liquid.
OR
(1) b (2) a (3) b
(4) Weight is vector quantity as it has magnitude as well as direction
which is always towards centre of a earth.

You might also like